Nothing Special   »   [go: up one dir, main page]

Cons Study Summary Quiestions

Download as docx, pdf, or txt
Download as docx, pdf, or txt
You are on page 1of 124

MS Muscoloskeletal Disorders

A client is in Buck's skin traction after fracturing the right hip. The nurse should include which action in the plan of care?

A. B. C. D.

Removing the weights once every shift Maintaining the bed in the knee-gatch position Keeping the client in semi-Fowler's position Maintaining correct body alignment

Rationale: Buck's traction produces realignment by exerting a pulling force on the fractured hip. Therefore, the nurse must maintain correct body alignment. Traction should be continuous; if the weights must be removed, the nurse should apply manual traction until the weights are replaced. The knee-gatch position shouldn't be used because it disrupts the constant pulling force needed for alignment. Semi-Fowler's position would cause the client to slide in the direction of the traction, defeating the purpose of traction.

The nurse is teaching a client with osteomalacia how to take prescribed vitamin D supplements. The nurse stresses the importance of taking only the prescribed amount because high doses of vitamin D can be toxic. Early signs and symptoms of vitamin D toxicity include:

A. B. C. D.

GI upset and metallic taste. dry skin, hair loss, and inflamed mucous membranes. flushing and orthostatic hypotension. sensory neuropathy and difficulty maintaining balance.

Rationale: GI upset and metallic taste are early signs and symptoms of vitamin D toxicity. Such toxicity also may cause headache, weakness, renal insufficiency, renal calculi, hypertension, arrhythmias, muscle pain, and conjunctivitis. Dry skin, hair loss, and inflamed mucous membranes suggest vitamin A toxicity. Flushing and orthostatic hypotension (effects of vasodilation) may result from nicotinic acid and nicotinamide supplements, used to correct niacin deficiency. Sensory neuropathy and difficulty maintaining balance suggest pyridoxine toxicity.

A client is undergoing an extensive diagnostic workup for suspected muscular dystrophy. The nurse knows that muscular dystrophy has many forms, but that one assessment finding is common to all forms. Which finding belongs in this category?

A. B. C. D.

Muscle weakness Cardiac muscle involvement Pseudohypertrophy of the calf muscles Muscle pain

Rationale: Muscle weakness is common to all forms of muscular dystrophy. Cardiac muscle involvement and pseudohypertrophy of the calf muscles don't occur in all forms of muscular dystrophy. Muscle pain is rare with any form.

A client is admitted to the orthopedic unit for treatment of a fractured right femur caused by a motor vehicle crash. He is scheduled to undergo an open reduction internal-fixation of the right femur. The night before surgery, the nurse administers 250 mg glutethimide (Doriden) as prescribed. Which statement regarding usage of glutethimide is correct?

A. B. C. D.

The nurse should store the drug in a tight, light-resistant container. The nurse should dilute it in fruit juice to improve absorption. The nurse shouldn't use the liquid if it becomes slightly darkened. The nurse should discard unused portions of the drug immediately after use.

Rationale: Glutethimide should be stored in a tight, light-resistant container to avoid possible deterioration. Diluting the drug in fruit juice doesn't improve its absorption. A slight darkening of the liquid form of glutethimide results from exposure to light and air but doesn't affect the drug's safety or potency. Unused portions of glutethimide may be stored for future use in a tight, light-resistant container.

To help prevent osteoporosis, what should the nurse advise a young woman to do?

A. B. C. D.

Avoid trauma to the affected bone. Encourage the use of a firm mattress. Consume at least 1,000 mg of calcium daily. Keep the serum uric acid level in the normal range.

Rationale: To help prevent osteoporosis, the nurse should encourage the client to consume at least the recommended daily allowance (RDA) of calcium. Before menopause, the RDA is 1,000 mg; after menopause, it is 1,500 mg. Because osteoporosis affects all bones, option A is inappropriate. Options B and D don't relate to osteoporosis. The nurse should encourage a client with ankylosing spondylitis to sleep on a firm mattress. The nurse should advise a client with gouty arthritis to keep the serum uric acid level in the normal range.

A client undergoes a total hip replacement. Which statement made by the client would indicate to the nurse that the client requires further teaching?

A. B. C. D.

"I'll need to keep several pillows between my legs at night." "I need to remember not to cross my legs. It's such a habit." "The occupational therapist is showing me how to use a sock puller to help me get dressed." "I don't know if I'll be able to get off that low toilet seat at home by myself."

Rationale: To prevent hip dislocation after a total hip replacement, the client must avoid bending the hips beyond 90 degrees. Assistive devices, such as a raised toilet seat, should be used to prevent severe hip flexion. Using an abduction pillow or placing several pillows between the legs reduces the risk of hip dislocation by preventing adduction and internal rotation of the legs. Teaching the client to avoid crossing the legs also reduces the risk of hip dislocation. A sock puller helps a client get dressed without flexing the hips beyond 90 degrees.

X-rays reveal a leg fracture in a client who was brought to the emergency department after falling on ice. After a cast is applied and allowed to dry, the nurse teaches the client how to use crutches. Which instruction should the nurse provide about climbing stairs?

A. "Place both crutches on the first step and swing both legs upward to this step." B. "Place the unaffected leg on the first step, followed by the crutches and the injured leg, which should move together." C. "Place the crutches and injured leg on the first step, followed by the unaffected leg." D. "Place the injured leg and the crutch on the unaffected side on the first step; the unaffected leg and crutch on the injured side follow."

Rationale: When climbing stairs with crutches, the client should lead with the unaffected leg, followed by the crutches and injured leg moving together. Any other method is incorrect and could increase the client's risk of falling.

Which of the following laboratory studies is most relevant to treating a client who has sustained a pelvic fracture?

A. B. C. D.

Urine myoglobin Urinalysis Type and crossmatch Serum ethanol

Rationale: Because of the rich blood supply to the pelvis, fractures to this area can result in significant blood loss. Type and crossmatch is a priority laboratory test in preparing for fluid replacement. Urinalysis and serum ethanol, although part of a trauma workup, don't alter treatment of a pelvic fracture. Urine isn't commonly analyzed for myoglobin with this injury unless the mechanism was a crush injury; even then, urinalysis isn't as high a priority as type and crossmatch.

After a traumatic back injury, a client requires skeletal traction. When caring for this client, the nurse must:

A. B. C. D.

change the client's position only if ordered by the physician. maintain traction continuously to ensure its effectiveness. support the traction weights with a chair or table to prevent accidental slippage. restrict the client's fluid and fiber intake to reduce the movement required for bedpan use.

Rationale: The nurse must maintain skeletal traction continuously to ensure its effectiveness. The client should be repositioned every 2 hours to prevent skin breakdown. Traction weights must hang freely to be effective; they should never

be supported. The nurse should increase, not restrict, the client's fluid and fiber intake (unless contraindicated by a concurrent illness) to prevent constipation associated with complete bed rest.

A client undergoes hip-pinning surgery to treat an intertrochanteric fracture of the right hip. The nurse should include which intervention in the postoperative plan of care?

A. B. C. D.

Performing passive range-of-motion (ROM) exercises on the client's legs once each shift Keeping a pillow between the client's legs at all times Turning the client from side to side every 2 hours Maintaining the client in semi-Fowler's position

Rationale: After hip pinning, the client must keep the affected leg abducted at all times; placing a pillow between the legs reminds the client not to cross the legs and to keep the leg abducted. Passive or active ROM exercises shouldn't be performed on the affected leg during the postoperative period because this could damage the operative site and cause hip dislocation. Most clients should be turned to the unaffected side, not from side to side. After hip pinning, the client must avoid acute flexion of the affected hip to prevent possible hip dislocation; therefore, semi-Fowler's position should be avoided.

The nurse is teaching a female client about preventing osteoporosis. Which of the following teaching points is correct?

A. B. C. D.

Obtaining an X-ray of the bones every 3 years is recommended to detect bone loss. To avoid fractures, the client should avoid strenuous exercise. The recommended daily allowance of calcium may be found in a wide variety of foods. Obtaining the recommended daily allowance of calcium requires taking a calcium supplement.

Rationale: Premenopausal women require 1,000 mg of calcium per day. Postmenopausal women require 1,500 mg per day. It's often, though not always, possible to get the recommended daily requirement in the foods we eat. Supplements are available but not always necessary. Osteoporosis doesn't show up on ordinary X-rays until 30% of the bone loss has occurred. Bone densitometry can detect bone loss of 3% or less. This test is sometimes recommended routinely for women over 35 who are at risk. Strenuous exercise won't cause fractures.

A client recently had a right total hip replacement. As a result of intraoperative blood loss, postoperative serum hemoglobin levels and hematocrit are low. The physician prescribes two units of packed red blood cells. During the infusion of the first unit of blood, the client develops a transfusion reaction and experiences urticaria, itching, and bronchospasm. The nurse discontinues the transfusion and notifies the physician. Which antihistamine would the physician most likely prescribe to treat this type I hypersensitivity reaction?

A. B. C. D.

tripelennamine citrate (PBZ) astemizole (Hismanal) cyclizine (Marezine) chlorpheniramine maleate (Chlor-Trimeton)

Rationale: The parenteral form of chlorpheniramine maleate is used to relieve symptoms of anaphylaxis and allergic reactions to blood or plasma. Tripelennamine citrate, astemizole, and cyclizine aren't used to treat blood transfusion reactions.

Before a client undergoes arthroscopy, the nurse reviews the assessment findings for contraindications for this procedure. Which finding is a contraindication?

A. B. C. D.

Joint pain Joint deformity Joint flexion of less than 50% Joint stiffness

Rationale: Arthroscopy is contraindicated in clients with joint flexion of less than 50% because of technical problems in inserting the instrument into the joint to see it clearly. Other contraindications for this procedure include skin and wound infections. Joint pain may be an indication, not a contraindication, for arthroscopy. Joint deformity and joint stiffness aren't contraindications for this procedure.

A client is admitted with severe pain in the knees. Which form of arthritis is characterized by urate deposits and joint pain, usually in the feet and legs, and occurs primarily in men over age 30?

A. B. C. D.

Septic arthritis Traumatic arthritis Intermittent arthritis Gouty arthritis

Rationale: Gouty arthritis, a metabolic disease, is characterized by urate deposits and pain in the joints, especially those in the feet and legs. Urate deposits don't occur in septic or traumatic arthritis. Septic arthritis results from bacterial invasion of a joint and leads to inflammation of the synovial lining. Traumatic arthritis results from blunt trauma to a joint or ligament. Intermittent arthritis is a rare, benign condition marked by regular, recurrent joint effusions, especially in the knees.

After a person experiences a closure of the epiphyses, which of the following is true?

A. B. C. D.

The bone grows in length but not thickness. The bone increases in thickness and is remodeled. Both bone length and thickness continues. No further increase in bone length occurs.

Rationale: After closure of the epiphyses, no further increase in bone length can occur. All of the other options are

inappropriate and not related to closure of the epiphyses.

A 69-year-old client asks the nurse what the difference is between osteoarthritis (OA) and rheumatoid arthritis (RA). Which response is correct?

A. B. C. D.

OA is a noninflammatory joint disease. RA is characterized by inflamed, swollen joints. OA and RA are very similar. OA affects the smaller joints and RA affects the larger, weight-bearing joints. OA affects joints on both sides of the body. RA is usually unilateral. OA is more common in women. RA is more common in men.

Rationale: OA is a degenerative arthritis, characterized by the loss of cartilage on the articular surfaces of weight-bearing joints with spur development. RA is characterized by inflammation of synovial membranes and surrounding structures. OA may occur in one hip or knee and not the other, whereas RA commonly affects the same joints bilaterally. RA is more common in women; OA affects both sexes equally.

A client has acute, painful muscle spasms. The physician prescribes chlorzoxazone (Paraflex), 500 mg P.O. t.i.d. A centrally acting skeletal muscle relaxant, chlorzoxazone commonly is used to treat:

A. B. C. D.

muscle spasm caused by cerebral palsy. chronic musculoskeletal disorder. lower extremity spasticity. severe muscle spasm.

Rationale: Chlorzoxazone is used to treat acute, painful musculoskeletal conditions or severe muscle spasm. Centrally acting skeletal muscle relaxants like chlorzoxazone are ineffective in treating spasticity associated with chronic neurologic disease, such as cerebral palsy. They can treat acute musculoskeletal disorders, not chronic ones. Chlorzoxazone and the other relaxants are used to treat spasticity of any extremity.

The nurse is assessing a client with possible osteoarthritis. The most significant risk factor for osteoarthritis is:

A. B. C. D.

congenital deformity. age. trauma. obesity.

Rationale: Age is the most significant risk factor for developing osteoarthritis. Development of primary osteoarthritis is influenced by genetic, metabolic, mechanical, and chemical factors. Secondary osteoarthritis usually has identifiable precipitating events such as trauma.

Which of the following nursing interventions is essential in caring for a client with compartment syndrome?

A. B. C. D.

Keeping the affected extremity below the level of the heart Wrapping the affected extremity with a compression dressing to help decrease the swelling Removing all external sources of pressure, such as clothing and jewelry Starting an I.V. line in the affected extremity in anticipation of venogram studies

Rationale: Nursing measures should include removing all clothing, jewelry, and external forms of pressure (such as dressings or casts) to prevent constriction and additional tissue compromise. The extremity should be maintained at heart level (further elevation may increase circulatory compromise, whereas a dependent position may increase edema). A compression wrap, which increases tissue pressure, could further damage the affected extremity. There is no indication that diagnostic studies would require I.V. access in the affected extremity.

After surgery to treat a hip fracture, a client returns from the postanesthesia care unit to the medical-surgical unit. Postoperatively, how should the nurse position the client?

A. B. C. D.

With the affected hip flexed acutely With the leg on the affected side abducted With the leg on the affected side adducted With the affected hip rotated externally

Rationale: The nurse must keep the leg on the affected side abducted at all times after hip surgery to prevent accidental dislodgment of the affected hip joint. Placing a pillow or an A-frame between the legs helps maintain abduction and reminds the client not to cross the legs. The nurse should avoid acutely flexing the client's affected hip (for example, by elevating the head of the bed excessively), adducting the leg on the affected side (such as by moving it toward the midline), or externally rotating the affected hip (such as by removing support along the outer side of the leg) because these positions may cause dislocation of the injured hip joint.

A client is treated in the emergency department for a Colles' fracture sustained during a fall. What is a Colles' fracture?

A. B. C. D.

Fracture of the distal radius Fracture of the olecranon Fracture of the humerus Fracture of the carpal scaphoid

Rationale: Colles' fracture is a fracture of the distal radius, such as from a fall on an outstretched hand. It's most common in women. Colles' fracture doesn't refer to a fracture of the olecranon, humerus, or carpal scaphoid.

A client has a herniated disk in the region of the third and fourth lumbar vertebrae. On assessment, the nurse expects to note:

A. B. C. D.

hypoactive bowel sounds. severe low back pain. sensory deficits in one arm. weakness and atrophy of the arm muscles.

Rationale: The most common finding in a client with a herniated lumbar disk is severe low back pain, which radiates to the buttocks, legs, and feet usually unilaterally. A herniated disk also may cause sensory and motor loss (such as footdrop) in the area innervated by the compressed spinal nerve root. During later stages, it may cause weakness and atrophy of leg muscles. The condition doesn't affect bowel sounds or the arms.

A client seeks care for low back pain of 2 weeks' duration. Which assessment finding suggests a herniated intervertebral disk?

A. B. C. D.

Pain radiating down the posterior thigh Back pain when the knees are flexed Atrophy of the lower leg muscles Homans' sign

Rationale: A herniated intervertebral disk may compress the spinal nerve roots, causing sciatic nerve inflammation that results in pain radiating down the leg. Slight knee flexion should relieve, not precipitate, low back pain. If nerve root compression remains untreated, weakness or paralysis of the innervated muscle group may result; lower leg atrophy may occur if muscles aren't used. Homans' sign is more typical of phlebothrombosis.

A client is brought to the emergency department after injuring the right arm in a bicycle accident. The orthopedic surgeon tells the nurse that the client has a greenstick fracture of the arm. What does this mean?

A. B. C. D.

The fracture line extends through the entire bone substance. The fracture results from an underlying bone disorder. Bone fragments are separated at the fracture line. One side of the bone is broken and the other side is bent.

Rationale: In a greenstick fracture, one side of the bone is broken and the other side is bent. A greenstick fracture also may refer to an incomplete fracture in which the fracture line extends only partially through the bone substance and doesn't disrupt bone continuity completely. (Other terms for greenstick fracture are willow fracture and hickory-stick fracture.) The fracture line extends through the entire bone substance in a complete fracture. A fracture that results from an underlying bone disorder, such as osteoporosis or a tumor, is a pathologic fracture, which typically occurs with minimal trauma. Bone fragments are separated at the fracture line in a displaced fracture.

The nurse is caring for a client who underwent a total hip replacement. What should the nurse and other caregivers do to prevent dislocation of the new prosthesis?

A. B. C. D.

Keep the affected leg in a position of adduction. Use measures other than turning to prevent pressure ulcers. Prevent internal rotation of the affected leg. Keep the hip flexed by placing pillows under the client's knee.

Rationale: External rotation and abduction of the hip will help prevent dislocation of a new hip joint. Internal rotation and adduction should be avoided. Postoperative total hip replacement clients may be turned onto the unaffected side. While the hip may be flexed slightly, it shouldn't exceed 90 degrees and maintenance of flexion isn't necessary.

A client who has just been diagnosed with mixed muscular dystrophy asks the nurse about the usual course of this disease. How should the nurse respond?

A. B. C. D.

"You should ask your physician about that." "The strength of your arms and pelvic muscles will decrease gradually, but this should cause only slight disability." "You may experience progressive deterioration in all voluntary muscles." "This form of muscular dystrophy is a relatively benign disease that progresses slowly."

Rationale: Muscular dystrophy causes progressive, symmetrical wasting of skeletal muscles, without neural or sensory defects. The mixed form of the disease typically strikes between ages 30 and 50 and progresses rapidly, causing deterioration of all voluntary muscles. Because the client asked the nurse this question directly, the nurse should answer and not simply refer the client to the physician. Limb-girdle muscular dystrophy causes a gradual decrease in arm and pelvic muscle strength, resulting in slight disability. Facioscapulohumeral muscular dystrophy is a slowly progressive, relatively benign form of muscular dystrophy; it usually arises before age 10.

A client is treated in the emergency department for acute muscle strain in the left leg caused by trying a new exercise. During discharge preparation, the nurse should provide which instruction?

A. B. C. D.

"Apply ice packs for the first 12 to 18 hours." "Apply heat packs for the first 24 to 48 hours." "Apply ice packs for the first 24 to 48 hours, then apply heat packs." "Apply heat packs for the first 24 hours, then apply ice packs for the next 48 hours."

Rationale: The nurse should instruct the client to apply ice packs to the injured area for the first 24 to 48 hours to reduce swelling and then apply heat to increase comfort, promote reabsorption of blood and fluid, and speed healing. Applying ice for only 12 to 18 hours may not keep swelling from recurring. Applying heat for the first 24 to 48 hours would worsen, not ease, swelling. Applying ice 48 hours after the injury would be less effective because swelling already has occurred by that time.

A 78-year-old client has a history of osteoarthritis. Which signs and symptoms would the nurse expect to find on physical assessment?

A. B. C. D.

Joint pain, crepitus, Heberden's nodes Hot, inflamed joints; crepitus; joint pain Tophi, enlarged joints, Bouchard's nodes Swelling, joint pain, and tenderness on palpation

Rationale: Clinical findings for osteoarthritis include joint pain, crepitus, Heberden's nodes, Bouchard's nodes, and enlarged joints. The joint pain occurs with movement and is relieved by rest. As the disease progresses, pain may also occur at rest. Heberden's nodes are bony growths that occur at the distal interphalangeal joints. Bouchard's nodes involve the proximal interphalangeal joints. Tophi are deposits of sodium urate crystals that occur in chronic gout not osteoarthritis. Hot, inflamed joints rarely occur in osteoarthritis. Swelling, joint pain, and tenderness on palpation occur with a sprain injury.

Which of the following would the nurse identify as a neurotransmitter?

A. B. C. D.

Acetylcholine Adenosine triphosphate (ATP) Cholinesterase Creatine phosphate

Rationale: Acetylcholine is a neurotransmitter contained in the axon terminal vesicles. ATP is the substance that, when broken down, provides energy for muscle contraction. Cholinesterase is an enzyme that breaks down acetylcholine and prevents continuous stimulation of skeletal muscle. Creatine phosphate is a substance found in muscle that, when broken down, releases energy.

A client who recently had a cerebrovascular accident (CVA) requires a cane to ambulate. When teaching about cane use, explain that the reason for holding a cane on the uninvolved side is to:

A. B. C. D.

prevent leaning. distribute weight away from the involved side. maintain stride length. prevent edema.

Rationale: Holding a cane on the uninvolved side distributes weight away from the involved side. Holding the cane close to the body prevents leaning. Use of a cane won't maintain stride length or prevent edema.

Which of the following is a priority nursing diagnosis for the client with an amputated extremity?

A. B. C. D.

Impaired skin integrity related to effects of the injury Anticipatory grieving related to the loss of a limb Disturbed body image related to changes in the structure of a body part Ineffective peripheral tissue perfusion related to injury and amputation

Rationale: The priority diagnosis is Ineffective peripheral tissue perfusion resulting from the loss of circulation secondary to amputation. All the nursing diagnoses listed are appropriate for a client presenting with a traumatic amputation of an extremity.

The nurse is developing a teaching plan for a client who must undergo an above-the-knee amputation of the left leg. After a leg amputation, exercise of the remaining limb:

A. B. C. D.

isn't necessary. should begin immediately postoperatively. should begin the day after surgery. begins at a rehabilitation center.

Rationale: Exercise should begin the day after surgery. Exercise is necessary to maintain the muscle tone of the remaining limb. Immediately after surgery, the client usually isn't alert enough to participate and may be in too much pain. Exercise needs to begin before discharge to a rehabilitation center.

A client with gouty arthritis is prescribed a low-purine diet. The nurse should instruct this client to avoid:

A. B. C. D.

organ meats. citrus fruits. green vegetables. fresh fish.

Rationale: Because gouty arthritis is a disorder of purine metabolism, the client should avoid foods high in purine, such as organ meats, anchovies, sardines, shellfish, chocolate, and meat extracts. Citrus fruits, green vegetables, and fresh fish should be included in a well-balanced diet.

The nurse is caring for a client who complains of lower back pain. Which instructions should the nurse give to the client to prevent back injury?

A. B. C. D.

Bend over the object you're lifting. Narrow the stance when lifting. Push or pull an object using your arms. Stand close to the object you're lifting.

Rationale: Standing close to an object you're lifting moves the body's center of gravity closer to the object, allowing the legs, rather than the back, to bear the weight. No one should bend over an object when lifting; instead, the back should be straight, and bending should be at the hips and knees. When lifting, spreading the legs apart widens the base of support and lowers the center of gravity, providing better balance. Pushing or pulling an object using the weight of the body, rather than the arms or back, prevents back strain. Using a larger number of muscle groups distributes the workload.

The nurse monitors a client receiving enoxaparin (Lovenox) 30 mg S.C. b.i.d after hip replacement surgery. Which adverse reaction is the client most likely to experience?

A. B. C. D.

Anaphylactic shock Hypersensitivity Bronchospasm Bleeding

Rationale: Bleeding is the most common adverse reaction associated with enoxaparin. The drug isn't known to induce anaphylactic shock or bronchospasm, and hypersensitivity reactions are rare.

Following a boating accident, a 30-year-old client with multiple fractures is admitted to a semiprivate room in a progressive care unit. The client, who was driving the boat, is unaware that his girlfriend's 9-year-old son was killed in the accident. The client's parents instruct the nurse to prohibit phone calls and to withhold information about the accident. During an assessment of the client, the nurse notices that the television is on and the news is starting. It would be most appropriate for the nurse to:

A. B. C. D.

turn the television off and tell the client it interferes with the assessment. allow the client to view the television and deal with any questions as they come. instruct the client to change the channel to a station that isn't televising the news. attempt to distract the client from watching the television.

Rationale: The nurse-client relationship is built on trust, so the nurse can't withhold information from her client. She may refer the client to another source for the information, but she can't prohibit the client from seeking information. It would be most appropriate to deal with the client's questions as they come. Turning the television off, changing the channel, and distracting the client are all deceitful practices, which can damage a therapeutic nurse-client relationship.

For a client with osteoporosis, the nurse should provide which dietary instruction?

A. B. C. D.

"Decrease your intake of red meat." "Decrease your intake of popcorn, nuts, and seeds." "Eat more fruits to increase your potassium intake." "Eat more dairy products to increase your calcium intake."

Rationale: Osteoporosis causes a severe, general reduction in skeletal bone mass. To offset this reduction, the nurse should advise the client to increase calcium intake by consuming more dairy products, which provide about 75% of the calcium in the average diet. None of the other options would stop osteoporosis from worsening.

The nurse is caring for an elderly female with osteoporosis. When teaching the client, the nurse should include information about which major complication?

A. B. C. D.

Bone fracture Loss of estrogen Negative calcium balance Dowager's hump

Rationale: Bone fracture is a major complication of osteoporosis that results when loss of calcium and phosphate increases the fragility of bones. Estrogen deficiencies result from menopause not osteoporosis. Calcium and vitamin D supplements may be used to support normal bone metabolism, but a negative calcium balance isn't a complication of osteoporosis. Dowager's hump results from bone fractures. It develops when repeated vertebral fractures increase spinal curvature.

After total hip replacement, a client is receiving epidural analgesia to relieve pain. Which of the following is a nursing priority for this client?

A. B. C. D.

Changing the catheter site dressing every shift Assessing capillary refill time Assessing for sensation in the legs Keeping the client flat in bed

Rationale: For epidural analgesia, a catheter is placed outside the dura mater in the epidural space. Catheter displacement, which may cause spinal injury, is signaled by loss of motion and sensation in the legs. Therefore, the nurse should assess closely for sensation and ask about numbness of the legs. The nurse should change the catheter site dressing every day or every other day. Capillary refill time has no bearing on epidural analgesia. A client with an epidural catheter may ambulate and need not be confined to bed.

The nurse is teaching the client how to use a cane. Which of the following statements is most inaccurate?

A. B. C. D.

The client should hold the cane on the involved side. The client should hold the cane close to his body. The stride length and the timing of each step should be equal. The nurse should stand behind the client to prevent falls.

Rationale: The client is instructed to hold the cane on the uninvolved side, 24" to 26" from the base of the little toe. This is done to promote a reciprocal gait pattern. The nurse should instruct the client to hold the cane close to his body to prevent leaning. The stride length and timing of each step should be equal. To prevent falls, the nurse stands behind the client as he is learning to use the cane.

Which of these findings best correlates with a diagnosis of osteoarthritis?

A. B. C. D.

Joint stiffness that decreases with activity Erythema and edema over the affected joint Anorexia and weight loss Fever and malaise

Rationale: A characteristic feature of osteoarthritis (degenerative joint disease) is joint stiffness that decreases with activity and movement. The other options are associated with rheumatoid arthritis, a more severe and destructive form of arthritis.

A client is on bed rest after sustaining injuries in a car accident. Which nursing action would help the client avoid complications of immobility?

A. B. C. D.

Decreasing fluid intake to ease dependent edema Turning the client every 2 hours and massaging bony prominences Raising the head of the bed to maximize the client's lung inflation Bathing and feeding the client to decrease energy expenditure

Rationale: To avoid pressure ulcers in an immobilized client, the nurse must assess the skin thoroughly and use such preventive measures as regular turning, massage of bony prominences, a low-air-loss mattress, and a trapeze (if the client's condition allows). The nurse should increase, not decrease, the client's fluid intake to help prevent renal calculi, which may result from immobility. To prevent atelectasis, another complication of immobility, having the client cough, deep breathe, and use an incentive spirometer would be more effective than raising the head of the bed. Instead of bathing and feeding the client, the nurse should promote independent self-care activities whenever possible to prepare the client for a return to the previous health status.

The nurse is managing the care of a client with osteoarthritis. Appropriate treatment strategies for osteoarthritis include:

A. B. C. D.

administration of narcotics for pain control. bed rest for painful exacerbations. administration of nonsteroidal anti-inflammatory drugs (NSAIDs). vigorous physical therapy for the joints.

Rationale: NSAIDs are routinely used for anti-inflammatory and analgesic effects. NSAIDs reduce inflammation, which

causes pain. Narcotics aren't used for pain control in osteoarthritis. Normal joint range of motion and exercise (not vigorous physical therapy) are encouraged to maintain mobility and reduce joint stiffness.

The nurse is developing a teaching plan for a client diagnosed with osteoarthritis. To minimize injury to the osteoarthritic client, the nurse should instruct the client to:

A. B. C. D.

install safety devices in the home. wear comfortable shoes. get help when lifting objects. wear protective devices when exercising.

Rationale: Most accidents occur in the home and safety devices are the most important element in minimizing injury. Shoes should be supportive and not too worn. The client needs to use proper body mechanics when stooping or picking up objects. Protective devices aren't usually necessary for the client to perform exercises.

A client with gangrene of the left foot is scheduled for below-knee amputation. When planning preoperative care, the nurse should assign highest priority to which nursing diagnosis?

A. B. C. D.

Imbalanced nutrition: Less than body requirements related to stress Impaired physical mobility related to effects of surgery Disturbed body image related to loss of body part Deficient knowledge related to denial

Rationale: Clients commonly fear amputation because it disturbs a familiar body image. To enhance postoperative recovery, the nurse should address such fears during the preoperative period. The client must accept body image changes before rehabilitation can occur. The other options may be appropriate but don't take precedence during the preoperative period.

A client with osteoarthritis tells the nurse she is concerned that the disease will prevent her from doing her chores. Which suggestion should the nurse offer?

A. B. C. D.

"Do all your chores in the morning, when pain and stiffness are least pronounced." "Do all your chores after performing morning exercises to loosen up." "Pace yourself and rest frequently, especially after activities." "Do all your chores in the evening, when pain and stiffness are least pronounced."

Rationale: A client with osteoarthritis must adapt to this chronic and disabling disease, which causes deterioration of the joint cartilage. The most common symptom of the disease is deep, aching joint pain, particularly in the morning and after exercise and weight-bearing activities. Because rest usually relieves the pain, the nurse should instruct the client to rest frequently, especially after activities, and to pace herself during daily activities. Option A is incorrect because the

pain and stiffness of osteoarthritis are most pronounced in the morning. Options B and D are incorrect because the client should pace herself and take frequent rests rather than doing all chores at once.

A client complains of excruciating pain and inflammation in the joint of the great left toe. Her serum urate level is 9 mg/dl. The physician diagnoses an acute attack of gouty arthritis and prescribes colchicine, 1 mg P.O. as an initial dose, followed by 0.5 mg P.O. every hour (not to exceed 4 mg in 24 hours) until the pain ceases. How long after oral administration of colchicine should pain relief occur?

A. B. C. D.

30 to 45 minutes 1 to 2 hours 4 to 12 hours 12 to 48 hours

Rationale: The pain that accompanies an acute gouty attack is relieved 12 to 48 hours after oral colchicine administration and 4 to 12 hours after I.V. therapy.

The nurse is caring for a client who recently underwent a total hip replacement. The nurse should:

A. B. C. D.

ease the client onto a low toilet seat. allow the client's legs to be crossed at the knees when out of bed. use soft chairs when the client is sitting out of bed. limit client hip flexion when sitting.

Rationale: Instruct the client to limit hip flexion to 90 degrees while sitting. Supply an elevated toilet seat so that the client can sit without having to flex his hip more than 90 degrees. Instruct the client not to cross his legs to avoid dislodging or dislocating the prosthesis. Caution the client against sitting in chairs that are too low or too soft; these chairs increase flexion, which is undesirable.

The nurse is caring for a client with a cast on the left arm. Which assessment finding is most significant for this client?

A. B. C. D.

Normal capillary refill in the great toe Presence of a normal popliteal pulse Intact skin around the cast edges Ability to move all toes

Rationale: Because a cast can irritate the skin, the nurse should inspect for this complication. Normally, the skin remains intact around the cast edges. Normal capillary refill in the left thumb and fingers is more significant than in the great toe because an arm cast can impair circulation in the affected arm. Similarly, the presence of a normal radial pulse

is more noteworthy than a popliteal pulse because a cast on the left arm could affect circulation in that limb. Movement of this client's fingers is more critical than movement of the toes because a left arm cast could compress a nerve, preventing movement of fingers in the left arm.

Which of the following structures is seldom dislocated?

A. B. C. D.

Knee Shoulder Foot Elbow

Rationale: Dislocations of the foot are rare. Dislocations of the knee, shoulder, and elbow occur more frequently than the foot.

On a visit to the family physician, a client complains of painful swelling on the lateral side of the great toe, at the metatarsophalangeal joint. After determining that the swelling is a bunion, the physician injects an intra-articular corticosteroid. The client asks the nurse what causes bunions. Which answer is correct?

A. B. C. D.

Bunions are congenital and can't be prevented. Bunions may result from wearing shoes that are too big, causing friction when the shoes slip back and forth. Some bunions are congenital; others are caused by wearing shoes that are too short or narrow. Bunions are caused by a metabolic condition called gout.

Rationale: Bunions may be congenital or may be acquired by wearing shoes that are too short or narrow, which increases pressure on the bursa at the metatarsophalangeal joint. Acquired bunions can be prevented. Wearing shoes that are too big may cause other types of foot trauma but not bunions. Gout doesn't cause bunions. Although a client with gout may have pain in the big toe, such pain doesn't result from a bunion.

The nurse is caring for a client with burns on his legs. Which nursing intervention will help to prevent contractures?

A. B. C. D.

Applying knee splints Elevating the foot of the bed Hyperextending the client's legs Performing shoulder range-of-motion (ROM) exercises

Rationale: Applying knee splints prevents leg contractures by holding the joints in a functional position. Elevating the foot of the bed doesn't prevent contractures. Hyperextending a body part for any length of time is inappropriate; it can cause contractures. Performing shoulder ROM exercises can prevent contractures in the shoulders but not in the legs.

When caring for a client experiencing an acute gout attack, the nurse anticipates administering which medication?

A. B. C. D.

allopurinol (Zyloprim) colchicine prednisone (Deltasone) propoxyphene hydrochloride (Darvon)

Rationale: The physician usually prescribes colchicine for a client experiencing an acute gout attack. This drug decreases leukocyte motility, phagocytosis, and lactic acid production, thereby reducing urate crystal deposits and relieving inflammation. Allopurinol is used to decrease uric acid production in clients with chronic gout. Although corticosteroids are prescribed to treat gout, the nurse wouldn't give them because they must be administered interarticularly to this client. Propoxyphene, a narcotic, may be used to treat osteoarthritis.

The nurse is giving instructions to a client who's going home with a cast on his leg. Which point is most critical?

A. B. C. D.

Using crutches properly Exercising joints above and below the cast, as ordered Avoiding walking on a leg cast without the physician's permission Reporting signs of impaired circulation

Rationale: Although all of these interventions are important, reporting signs of impaired circulation is the most critical. Signs of impaired circulation must be reported to the physician immediately to prevent permanent damage. The other options reflect more long-term concerns. The client should learn to use his crutches properly to avoid nerve damage. The client may exercise above and below the cast, as the physician orders. The client should be told not to walk on the cast without the physician's permission.

A client is hospitalized for open reduction of a fractured femur. During postoperative assessment, the nurse monitors for signs and symptoms of fat embolism, which include:

A. B. C. D.

pallor and coolness of the affected leg. nausea and vomiting after eating. hypothermia and bradycardia. restlessness and petechiae.

Rationale: Signs and symptoms of fat embolism include restlessness, petechiae, and an altered mental status. Pallor and coolness of the affected leg are associated with a clot in the leg, not fat embolism. Nausea and vomiting after eating may be related to gastric obstruction. Hypothermia isn't an expected result of an open reduction of a fracture. Bradycardia has no relation to fat emboli but may indicate a cardiac problem.

A client, age 50, visits the physician for a routine checkup. The history reveals that the client was diagnosed with a spinal curvature at age 45. The nurse knows that life-threatening complications can occur if the progressive spinal curvature exceeds 65 degrees. To assess the client's risk for such a complication, the nurse should evaluate the severity of the curvature in which region of the spine?

A. B. C. D.

Cervical Thoracic Lumbar Sacral

Rationale: A progressive curvature of more than 65 degrees in the thoracic region of the spine may lead to cardiopulmonary failure as well as less serious signs and symptoms, such as fatigue, back pain, decreased height, and cosmetic deformity. Although a curvature may affect any part of the spine, life-threatening complications aren't associated with curvature of the cervical, lumbar, or sacral regions.

A client was diagnosed with chronic gouty arthritis 2 years ago. He has been taking sulfinpyrazone (Anturane), 200 mg P.O. b.i.d. as maintenance therapy. How soon after administration of this drug does onset of action occur?

A. B. C. D.

30 minutes 60 minutes 90 minutes 2 hours

Rationale: Sulfinpyrazone has a rapid onset of action, within 30 minutes after oral administration. It reaches its peak concentration within 1 to 2 hours and has a duration of action of 4 to 6 hours.

Which of the following cells are involved in bone resorption?

A. B. C. D.

Chondrocytes Osteoblasts Osteoclasts Osteocytes

Rationale: Osteoclasts carry out bone resorption by removing unwanted bone while new bone is forming in other areas. Chondrocytes are responsible for forming new cartilage. Osteoblasts are bone-forming cells that secrete collagen and other substances. Osteocytes, derived from osteoblasts, are the chief cells in bone tissue.

A client is scheduled for a laminectomy to repair a herniated intervertebral disk. When developing the postoperative plan of care, the nurse should include which action?

A. B. C. D.

Keeping a pillow under the client's knees at all times Placing the client in semi-Fowler's position Maintaining bed rest for 72 hours after the laminectomy Turning the client from side to side, using the logroll technique

Rationale: To avoid twisting the spine or hips when turning a client onto the side, the nurse should use the logroll technique. (Twisting after a laminectomy could injure the spine.) After surgery, the nurse shouldn't put anything under the client's knees or place the client in semi-Fowler's position because these actions increase the risk of deep vein thrombosis. Typically, the client is allowed out of bed by the first or second day after a laminectomy.

To assess the joints, the nurse asks a client to perform various movements. As the client moves the arm away from the midline, the nurse evaluates the ability to perform:

A. B. C. D.

protraction. retraction. adduction. abduction.

Rationale: A client performs abduction when moving a body part away from the midline. Protraction refers to drawing out or lengthening of a body part. Retraction, the opposite of protraction, refers to drawing back or shortening of a body part. Adduction, the opposite of abduction, is movement of a body part toward the midline.

The nurse is caring for a client with lower back pain who is scheduled for myelography using metrizamide (a watersoluble contrast dye). After the test, the nurse should place the client in which position?

A. B. C. D.

Head of the bed elevated 45 degrees Prone Supine with feet raised Supine with the head lower than the trunk

Rationale: After a myelogram, positioning will depend on the dye injected. When a water-soluble dye such as metrizamide is injected, the head of the bed is elevated to a 45-degree angle to slow the upward dispersion of the dye. The other positions are contraindicated when a water-soluble contrast dye is used. If an air-contrast study were performed, the client should be positioned supine with the head lower than the trunk.

During a scoliosis screening in a college heath center, a student asks the public health nurse about the consequences of untreated scoliosis. The nurse would be accurate by identifying one of the direct complications as:

A.

osteoporosis of the vertebra.

B. C. D.

impingement on pulmonary function. spontaneous spinal cord injury. pituitary hyposecretion.

Rationale: As untreated scoliosis progresses, the thoracic spinal curvature can impinge on the lungs and affect pulmonary function. Osteoporosis, spinal cord injury, and pituitary hyposecretion aren't directly attributed to untreated scoliosis.

A client comes to the outpatient department with suspected carpal tunnel syndrome. When assessing the affected area, the nurse expects to find which abnormality typically associated with this syndrome?

A. B. C. D.

Positive Tinel's sign Negative Phalen's sign Positive Chvostek's sign Negative Trousseau's sign

Rationale: The nurse expects a client with carpal tunnel syndrome to exhibit a positive Tinel's sign tingling or shocklike pain in reaction to light percussion over the median nerve at the wrist. The client also may have a positive Phalen's sign, characterized by hand tingling with acute wrist flexion. The nurse checks for Chvostek's sign and Trousseau's sign in a client with suspected hypocalcemia, not carpal tunnel syndrome.

After a car accident, a client is admitted to an acute care facility with multiple traumatic injuries, including a fractured pelvis. For 24 to 48 hours after the accident, the nurse must monitor the client closely for which potential complication of a fractured pelvis?

A. B. C. D.

Compartment syndrome Fat embolism Infection Volkmann's ischemic contracture

Rationale: Fat embolism is a relatively rare but life-threatening complication of pelvis and long-bone fractures, arising 24 to 48 hours after the injury. It occurs when fat droplets released at the fracture site enter the circulation, become lodged in pulmonary capillaries, and break down into fatty acids. Because these acids are toxic to the lung parenchyma, capillary endothelium, and surfactant, the client may develop pulmonary hypertension. Signs and symptoms of fat embolism include an altered mental status, fever, tachypnea, tachycardia, hypoxemia, and petechiae. Compartment syndrome and infection may complicate any fracture and aren't specific to a pelvis fracture. Volkmann's ischemic contracture is a potential complication of a hand or forearm fracture.

The nurse is teaching a client with a long leg cast how to use crutches properly while descending a staircase. The nurse should tell the client to:

A. B. C. D.

advance both legs first. advance the unaffected leg first. advance the affected leg first. advance both crutches first.

Rationale: To walk down a flight of stairs, body weight is first transferred to the unaffected leg. Both crutches are then advanced to the stair below. Body weight is transferred to the crutches as the affected leg descends. The unaffected leg is then brought down to the next step so that both legs and crutches are all on the same step. The procedure is repeated for each step.

Elderly clients who fall are most at risk for which of the following injuries?

A. B. C. D.

Wrist fractures Humerus fractures Pelvic fractures Cervical spine fractures

Rationale: Elderly clients who fall often sustain pelvic and lower extremity fractures. These injuries are devastating because they can seriously alter an elderly client's lifestyle and reduce functional independence. Wrist fractures usually occur with falls on an outstretched hand or from a direct blow. They are commonly found in young men. Humerus fractures and cervical spine fractures aren't age-specific.

A client with arterial insufficiency undergoes below-knee amputation of the right leg. Which action should the nurse include in the postoperative plan of care?

A. B. C. D.

Elevating the stump for the first 24 hours Maintaining the client on complete bed rest Applying heat to the stump as the client desires Removing the pressure dressing after the first 8 hours

Rationale: Stump elevation for the first 24 hours after surgery helps reduce edema and pain by increasing venous return and decreasing venous pooling at the distal portion of the extremity. Bed rest isn't indicated and could predispose the client to complications of immobility. Heat application would be inappropriate because it promotes vasodilation, which may cause hemorrhage and increase pain. The initial pressure dressing usually remains in place for 48 to 72 hours after surgery.

An X-ray of the left femur shows a fracture that extends through the midshaft of the bone and multiple splintering fragments. What is this type of fracture called?

A.

Compression fracture

B. C. D.

Greenstick fracture Comminuted fracture Impacted fracture

Rationale: A comminuted fracture typically is transverse the shaft of the bone and has multiple splintered bone fragments. A closed fracture implies that the skin integrity at or near the point of fracture is intact. A greenstick fracture occurs when the bone buckles or bends and the fracture line doesn't extend through the entire bone. An impacted fracture occurs when the distal and proximal portions of the fracture are wedged into each other. A compression fracture occurs when a severe force presses the bone together on itself.

A client is diagnosed with osteoporosis. Which electrolytes are involved in the development of this disorder?

A. B. C. D.

Calcium and sodium Calcium and phosphorous Phosphorous and potassium Potassium and sodium

Rationale: In osteoporosis, bones lose calcium and phosphate salts, becoming porous, brittle, and abnormally vulnerable to fracture. Sodium and potassium aren't involved in the development of osteoporosis.

The nurse is caring for a client placed in traction to treat a fractured femur. Which nursing intervention has the highest priority?

A. B. C. D.

Assessing the extremity for neurovascular integrity Keeping the client from sliding to the foot of the bed Keeping the ropes over the center of the pulley Ensuring that the weights hang free at all times

Rationale: Although all measures are correct, assessing neurovascular integrity takes priority. The pull of the traction must be continuous to keep the client from sliding. Sufficient countertraction must be maintained at all times by keeping the ropes over the center of the pulley. The line of pull is maintained by allowing the weights to hang free.

During a senior citizen health screening, the nurse observes a 75-year-old female with a severely increased thoracic curve, or "humpback". What is this condition called?

A. B. C. D.

Lordosis Kyphosis Scoliosis Genus varum

Rationale: Kyphosis refers to an increased thoracic curvature of the spine, or "humpback." Lordosis is an increase in the lumbar curve or swayback. Scoliosis is a lateral deformity of the spine. Genus varum is a bow-legged appearance of the legs.

A client's left leg is in skeletal traction with a Thomas leg splint and Pearson attachment. Which intervention should the nurse include in this client's plan of care?

A. B. C. D.

Apply the traction straps snugly. Assess the client's level of consciousness. Remove the traction at least every 8 hours. Teach the client how to prevent problems caused by immobility.

Rationale: By teaching the client about prevention measures, the nurse can help avoid problems caused by immobility, such as hypostatic pneumonia, muscle contracture, and atrophy. The nurse applies traction straps for skin traction not skeletal traction. For a client in skeletal traction, the nurse should assess the affected limb, rather than assess the level of consciousness. Removal of skeletal traction is the physician's responsibility not the nurse's.

A client with muscle weakness and an abnormal gait is being evaluated for muscular dystrophy. Which of the following confirms muscular dystrophy?

A. B. C. D.

Electromyography Muscle biopsy Family history of muscular dystrophy Gram stain of muscle tissue

Rationale: A muscle biopsy showing fat and connective tissue deposits confirms the diagnosis of muscular dystrophy. Electromyography commonly shows short, weak bursts of electrical activity in affected muscles; however, it isn't a conclusive test for muscular dystrophy. A family history of muscular dystrophy only suggests the disorder. A Gram stain of muscle tissue is inconclusive.

Which nursing diagnosis is most appropriate for an elderly client with osteoarthritis?

A. B. C. D.

Risk for injury related to altered mobility Impaired urinary elimination related to effects of aging Ineffective breathing pattern related to immobility Imbalanced nutrition: Less than body requirements related to effects of aging

Rationale: Typically, a client with osteoarthritis has stiffness in large, weight-bearing joints, such as the hips. This joint stiffness alters functional ability and range of movement, placing the client at risk for falling and injury. Therefore, Risk

for injury is the most appropriate nursing diagnosis. The other options are incorrect because osteoporosis doesn't affect urinary elimination, breathing, or nutrition.

A client with acute osteomyelitis is to receive parenteral penicillin for 4 to 6 weeks. Before administering the first dose, the nurse asks the client about known drug allergies. An allergy to which antibiotic or antibiotic class necessitates cautious use of penicillin?

A. B. C. D.

Tetracyclines Aminoglycosides Erythromycin Cephalosporins

Rationale: A client who is allergic to cephalosporins also may be allergic to penicillin. For the same reason, penicillin must be used cautiously in clients who are allergic to cephamycins, griseofulvin, or penicillamine. Cross-sensitivity between penicillin and tetracyclines, aminoglycosides, and erthyromycins hasn't been observed.

A client is admitted with acute osteomyelitis that developed after an open fracture of the right femur. When planning this client's care, the nurse should include which measure?

A. B. C. D.

Administering large doses of oral antibiotics as prescribed Instructing the client to ambulate twice daily Withholding all oral intake Administering large doses of I.V. antibiotics as prescribed

Rationale: Treatment of acute osteomyelitis includes large doses of I.V. antibiotics (after blood cultures identify the infecting organism). Surgical drainage may be indicated, and the affected bone is immobilized. The client usually requires I.V. fluids to maintain hydration, but oral intake isn't necessarily prohibited.

A client undergoes a muscle biopsy. After the procedure, the nurse must keep the biopsy site elevated for:

A. B. C. D.

2 to 4 hours. 6 to 8 hours. 12 hours. 24 hours.

Rationale: The nurse must keep the muscle biopsy site elevated for 24 hours to reduce edema. Elevating it for a shorter period may cause edema, worsening discomfort and increasing the risk of tissue injury.

At a health fair, a woman, age 43, with a family history of osteoporosis asks the nurse how much calcium she should consume. The nurse tells her that the recommended daily calcium intake for premenopausal women is:

A. B. C. D.

250 to 500 mg. 600 to 800 mg. 1,000 to 1,200 mg. 1,500 to 2,000 mg.

Rationale: Most authorities recommend that premenopausal women consume 1,000 to 1,200 mg of calcium daily. Less than 1,000 mg may not provide adequate protection against osteoporosis; more than 1,200 mg isn't necessary and may be harmful.

A client is admitted to an acute care facility with osteomyelitis. Which organism usually causes this infection?

A. B. C. D.

Escherichia coli Klebsiella Pseudomonas Staphylococcus aureus

Rationale: S. aureus is the most common cause of osteomyelitis. Less often, E. coli, Klebsiella, or Pseudomonas is the causative organism. Proteus and Salmonella are relatively rare causes. In a few cases, osteomyelitis results from a viral or fungal infection.

The nurse should monitor the client with a pelvic fracture receiving an opium derivative, such as morphine, for what common adverse reaction?

A. B. C. D.

Respiratory depression Diarrhea High fever Pupil dilation

Rationale: One of the most common adverse reactions to opium derivatives is decreased rate and depth of respiration, which worsens as the dosage is increased. This may cause periodic, irregular breathing or precipitate asthmatic attacks in susceptible clients. Opium derivatives also can cause constipation and pupil constriction. A high fever isn't an adverse reaction associated with opium derivatives.

The physician diagnoses primary osteoporosis in a client who has lost bone mass. In this metabolic disorder, the rate of bone resorption accelerates while bone formation slows. Primary osteoporosis is most common in:

A. B. C. D.

elderly men. young children. young menstruating women. elderly postmenopausal women.

Rationale: Although the cause of primary osteoporosis is unknown, an important contributing factor may be faulty protein metabolism resulting from estrogen deficiency and a sedentary lifestyle. Typically, these conditions occur in elderly postmenopausal women.

The nurse is teaching a client with a left fractured tibia how to walk with crutches. Which instruction is appropriate?

A. B. C. D.

Use the axillae to help carry the weight. All weight should be on the hands. Keep feet 12" (30 cm) apart to provide stability and a wide base of support. Take long strides to maintain maximum mobility.

Rationale: When using crutches, all weight should be on the hands. Constant pressure on the axillae from weight bearing can damage the brachial plexus nerve and produce crutch paralysis. Feet should be 6" to 8" (15 to 20 cm) apart to provide stability and support. Short strides not long ones provide safety and maximum mobility.

Which nursing diagnosis takes highest priority for a client with a compound fracture?

A. B. C. D.

Imbalanced nutrition: Less than body requirements related to immobility Impaired physical mobility related to trauma Risk for infection related to effects of trauma Activity intolerance related to weight-bearing limitations

Rationale: A compound fracture involves an opening in the skin at the fracture site. Because the skin is the body's first line of defense against infection, any skin opening places the client at risk for infection. Imbalanced nutrition: Less than body requirements is rarely associated with fractures. Although Impaired physical mobility and Activity intolerance may be associated with any fracture, these nursing diagnoses don't take precedence because they aren't as life-threatening as infection.

A client with gout is receiving probenecid (Benemid). When caring for this client, the nurse should monitor which laboratory value?

A. B. C. D.

Red blood cell count Serum uric acid level Hemoglobin level Serum potassium level

Rationale: In gout, joint inflammation results from deposits of uric acid crystals. Probenecid relieves this inflammation by reducing the uric acid level in the blood. To assess the drug's efficacy, the nurse should monitor the client's serum uric acid level. The other options don't reflect the action or effectiveness of probenecid.

A client comes to the emergency department complaining of pain in the right leg. When obtaining the history, the nurse learns that the client was diagnosed with diabetes mellitus at age 12. The nurse knows that this disease predisposes the client to which musculoskeletal disorder?

A. B. C. D.

Degenerative joint disease Muscular dystrophy Scoliosis Paget's disease

Rationale: Diabetes mellitus predisposes the client to degenerative joint disease. It isn't a predisposing factor for muscular dystrophy, scoliosis, or Paget's disease.

A client has a Fiberglas cast on the right arm. Which action should the nurse include in the plan of care?

A. B. C. D.

Keeping the casted arm warm by covering it with a light blanket Avoiding handling the cast for 24 hours or until it is dry Evaluating pedal and posterior tibial pulses every 2 hours Assessing movement and sensation in the fingers of the right hand

Rationale: The nurse should assess a casted arm every 2 hours for finger movement and sensation to make sure the cast isn't restricting circulation. To reduce the risk of skin breakdown, the nurse should leave a casted arm uncovered, which allows air to circulate through the cast pores to the skin below. Unlike a plaster cast, a Fiberglas cast dries quickly and can be handled without damage soon after application. The nurse should assess the brachial and radial pulses distal to the cast not the pedal and posterior tibial pulses, which are found in the legs.

A client seeks medical attention for a ganglion. Which statement about this musculoskeletal disorder is true?

A. B. C. D.

A ganglion is the most common benign soft-tissue mass in the foot. A ganglion is a precursor to a primary bone tumor. Surgical excision is the treatment of choice for a ganglion. Dorsiflexion exacerbates signs and symptoms of a ganglion.

Rationale: Dorsiflexion exacerbates signs and symptoms of a ganglion. A ganglion is the most common benign softtissue mass in the hand, not foot. It isn't a known precursor to a primary bone tumor. To treat a ganglion, the physician aspirates the ganglion, then injects a corticosteroid into the joint; the physician also may prescribe nonsteroidal anti-

inflammatory agents. Surgical excision is necessary only if signs and symptoms persist and the client's range of motion is impaired.

A client has sustained a right tibial fracture and has just had a cast applied. Which instruction should the nurse provide in his cast care?

A. B. C. D.

Cover the cast with a blanket until the cast dries. Keep your right leg elevated above heart level. Use a knitting needle to scratch itches inside the cast. A foul smell from the cast is normal.

Rationale: The leg should be elevated to promote venous return and prevent edema. The cast shouldn't be covered while drying because this will cause heat buildup and prevent air circulation. No foreign object should be inserted inside the cast because of the risk of cutting the skin and causing an infection. A foul smell from a cast is never normal and may indicate an infection.

A client is prescribed diazepam (Valium) to treat severe skeletal muscle spasms. During this therapy, the nurse monitors the client closely for adverse reactions. Which adverse reaction is most likely to occur?

A. B. C. D.

Bradycardia Skin rash Hypotension Sedation

Rationale: Most adverse reactions to diazepam and other benzodiazepines involve the central nervous system; less than 1% involve other body systems. Therefore, the client is more likely to experience sedation than bradycardia, skin rash, or hypotension.

A client with possible osteoarthritis is having X-rays performed on both knees. X-rays of an osteoarthritic joint reveal:

A. B. C. D.

enlargement of the joint space or margin. fluid deposition in joint spaces. osteophyte formation. cartilage growths at weight-bearing joints.

Rationale: In osteoarthritis, osteophytes form in joint spaces. Narrowing of joint spaces or margins, cystlike bony deposits in the joints, and long-bone growths at weight-bearing areas are other X-ray findings.

Which of the following is the most appropriate nursing diagnosis for a client with a strained ankle?

A. B. C. D.

Impaired skin integrity Impaired physical mobility Risk for deficient fluid volume Disturbed body image

Rationale: Ankle strains result in pain and damage to the ligaments as well as altered physical mobility. Although the traumatic event that caused the strain may disrupt the skin, the manifestations of a strain don't include disruption of skin integrity. Risk for deficient fluid volume is an appropriate nursing diagnosis for a process that results in the loss of a large volume of fluid or blood. Disruptions in body image can occur if the client's livelihood is altered because of the strain.

Section ONE
1. A client who received Extracorpeoreal Wave Lithotripsy will expect: a. Pain at site -correct b. Hematoma at site After ESWL, most patients have transient hematuria and intermittent colic as fragments pass. Proper hydration and analgesia are essential during the immediate postoperative period. Hematoma at site is a sign of complication. 2. A client is experiencing menstrual cramps. What will you advice? a. Increase potassium in the diet a week before menstruation b. Increase protein the diet - correct. c. Eat carbohydrates liberally d. Decrease carbohydrates 3. A patient has an active PTB. What will the nurse advice? a. Close the bedroom windows at night - room with negative pressure, how the windows could be open or closed? b. Wash utensils in warm water - doesn't address the question. c. Use surgical mask when talking to friends - not effective d. Cover mouth with tissue when coughing or sneezing - correct 4. The patients were admitted in the ER after a plane crash. Who will the nurse attend to first? a. A 5-year-old boy with soot on the chest of t-shirt and singed eyebrows - stable. No evidence for neck wounds or singed chest. b. Infant with second degree burns on both thighs -correct. Unstable client. 5. What will the nurse teach a patient who will undergo CABG? a. I will start coughing and deep breathing after CABG when I'm in PACU - yes, correct action to prevent atelectasis. b. The nurse will wake up hourly to take my BP - not needed that often c. I will have the chest tube after CABG - not needed 6. In post inguinal hernia repair, what will you tell the patient? a. You will have a pink-tinged urine until 3 days post-op - no way b. You will expect swelling on the scrotal area until 2 days post-op - true. Swelling usually subsides in 48 hours post-op c. Other options . A charge nurse is making assignment for a HIV (+) patient. Who will be assigned to this patient? a. It's okay for the Nursing Aide with colds to take of this patient - no, the pt is immunosuppressed. b. Nursing Aide with psoriasis can put on gloves and give care to the patient - the same reason as above. c. Pregnant Nursing Aide can take care the patient - correct, nothing is wrong

Which of the following is high risk of colon CA? a. 50-year-old male with prostate CA and taking colace - unrelated b.60-yr-old male who has a history of surgery for Hirschsprung's disease when he was 2 years old -that was too long ago c. 48-year-old smoker - big risk factor. 9. A patient is on CPAP with apnea. Purpose of CPAP is: a. To breathe for patient when patient stops breathing - wrong. Definition is close to SIMV, but not sure. b. To keep respiratory tract open -correct. 10. In a 2-day-old infant, what is normal? a. Touch the left cheek of the infant, will turn head to the left, flexing left extremities and extending right extremities - correct b. Hold the infant upright and infant steps with one foot alternating with other foot - too early for 2 days 11. Herbal medicine- Echinacea a. St. John's Wort b. Take with caution; it interferes with Coumadin - correct statement c. Garlic decreases BP \ Echinacea shouldn't be combined with other drugs that can cause liver damage. And because this herb may stimulate the immune system, it may interfere with the effects of immunosuppressants. 12. What would you instruct a caregiver taking care of patient with G-tube at home? a. Dress G-tube with NSS -dressing should be dry b. Inject 5 cc of air before feeding to check if G-tube is in place - correct, check the placement first. c. Raise the head of the bed - also necessary, but placement first. 13. A boy has hemophilia. What is the best activity for him? a. Biking with helmet? - correct. Some of the more commonly recommended activities for children with hemophilia include swimming, bicycle riding, walking, jogging, tennis, golf, dancing, fishing, sailing, and bowling. b. Playing soccer with knee pads? - boy should avoid contact sports. c. Playing jungle in a sandbox or sand area - low activity 14. Mother of a 4-year-old is newly diagnosed with leukemia. Which of the following statements need follow up? a. I will seek a second opinion -disbelief, normal process of grieving. b. This hospital is giving us a hard time - sort of denial, but not sure... c. I will celebrate and make a party and invite all his friends when we get home - should be correct. What's the reason for the party? The word "all" is suspicious to me. The child is immunosuppressed, can catch any opportunistic disease from the invited friends. 15. A client is with bulimia nervosa and admitted in the hospital. The nurse would a. Make a contract with client that no more weight loss will occur correct upon discharge.

16. An old woman has mouth sores and anorexia. What is the best diet for her? a. A glass of cranberry juice -can be irritable b. Milkshake - correct; provides nutrients plus needed calcium for depleted bones in elderly women. 17. How can the nurse tell that a woman in labor is experiencing utero-placental insufficiency? a. Early deceleration -wrong b. Late deceleration -correct (found in the internet) http://www.nursingceu.com/NCEU/courses/placentanc/ c. Variable deceleration - wrong 18. A woman was admitted in the delivery room with prolapsed cord. What will the nurse do? a. Cover the cord with dressing moistened with NSS - wrong, there is no purpose for that. b. Place the mother in knee-chest position - correct, decreases the pressure on cord. 19. For 2 days post-op BKA with diabetes. What action will the nurse question? a. Patient teaching crutch walking to a 2-day post-op BKA -normal finding b. Social worker talking to patient about home health after discharge - normal finding c. Dietician leaving pamphlets on diet restriction at patient's bedside to read - correct, teaching should be done by RN d. Other option -wrong 20. A patient who is receiving radiation therapy has nasopharyngeal cancer. The nurse would assess? a. Dry mouth - correct, the mouth contains hundreds of different bacteria, some helpful and some harmful. Chemotherapy and radiation therapy can cause changes in the lining of the mouth and production of saliva and upset the healthy balance of bacteria. These changes may lead to mouth sores, infections, and tooth decay. b. Difficulty swallowing -this also could be affected 21. A patient with moderate Alzheimer's disease was just discharged from the hospital. The nurse's best teaching to the family would be: a. Provide limited time to maintain and restructure activities -correct, but I dont personally like the word limited b. Unlock the doors in case of fire to provide safety -sounds like correct, because it addresses safety issues, but Alzheimers patients tend to wondering around, often get confused, and in the case of fire its not enough for family members just to unlock the doors, they have to rescue the patient. 23. A patient with TB will manifest the following signs and symptoms (enumeration) a. Dry cough b. Diaphoresis c. Temperature- 37.50C d. Lethargy A, B, C - correct. Dry cough can be than turn into productive cough with bloody secretions or greenish sputum.

24. A patient with a thoracic injury is complaining of restlessness and headache. What will you assess further? a. Blood pressure -correct, because low BP is an indication of internal hemorrhaging. 25. A patient is under chemotherapy. What will you assess further? a. Hydration status b. Nutritional status 26. A nurse who is 10 weeks pregnant can take care of a patient with: a. Fifth disease who is taking motrin wrong. Pregnant women and people with impaired immune systems or certain blood disorders (such as sickle cell disease, or thalassemia are at high risk for developing complications from fifth disease. These people need close monitoring by a health professional after exposure or if they develop symptoms of infection. Medical treatment for complications sometimes requires hospitalization. b. AIDS patient with candidiasis -correct, the disease not that contagious. 27. Patient had BKA 24 years ago. How do you prevent hip contracture? a. Elevate the stump - wrong, doesnt address the problem, its a preventive measure for swelling b. Keep in prone position for a short period of time - correct, number of hours in prone position should be in doctors orders 28. A newborn baby with hypothermia is kept in a radiant warmer. What is the next step to be done? a. Apply skin probe to the abdomen - wrong, even the nurse places the infant in a fancy warmer that provides body temperature measurements, they are not 100% correct, skin probes can become detached, be defective and often provide false readings; measuring the temp is nursing priority. b. Take vital signs -correct, VS are taken frequently, especially the temp. 29. A newborn baby with acrocyanosis and harlequin sign. What will the nurse do next? a. Check APGAR score -correct. This is done to evaluate color, reflex activity, tone, heart rate, respiration - done in delivery room to determine need for resuscitation b. Weigh the baby - doesnt address the problem c. Determine if Vitamin K was given - same as above 30. A patient had colonoscopy with bowel perforation. What will the nurse assess further? a. Abdominal distention and hyperactive bowel sound - wrong, these are normal findings. ABD distention could be because of the nature of the colonoscopy - small amounts of air could be injected to facilitate the advancement of the scope into the colon. b. Abdominal pain and tenderness - correct, indication of bleeding and peritonitis. BP should also taken 31. In a patient having a hemodialysis, what will the nurse assess? a. Elevated BP -correct, means that the machine doesnt filter the blood properly, leaving toxins in the circulation. Decreased BP is normal, but not elevated. b. Leaking dialysate - wrong, its impossible. Dialysate is used in peritoneal dialysis, not in hemodialysis.

32. A patient with a skeletal traction fell down, with traction hanging on the floor. The nurse noticed moderate serous discharges from the pin site. What is the nursing diagnosis? a. Risk for infection -wrong, infection comes after injury b. Risk for injury -correct, pt fell down, probably misalignment took place, risk for infection comes after that. 33. A Muslim patient is terminally ill. The nurse's best expectation is: a. A minister will stay with the patient - correct, b. A family member of the same gender will bathe the body when the pt. dies - wrong, the body should not be washed. Islamic washing of the body is done before burial (Ghusl before burial). If no relatives are available then the Islamic Council should be contacted. Reference taken from http://www.health.qld.gov.au/multicultural/pdf/islamgde.pdf 34. A Muslim with diabetes can have the following diet: a. Vegetables -correct, these are approved in HALAL (good food) b. Hotdog -wrong, may contain pork - HARAM (bad food, or unacceptable food) c. Hamburger - wrong, the same reason as above

39. How to position a newborn after feeding a. Put on his back -wrong, could be predisposing factor for GERD b. Turn to the left side -correct, prevents burping and aspiration. 40. Growth and development of a 7-month-old a. Just beginning to turn from his abdomen to back -wrong, this happens earlier b. Transferring an object from one hand to another - wrong, this happens earlier c. Standing and holding onto a furniture -correct

41. Who will you discharge first? a. A COPD patient with oxygen, elevated PCO2, pulse oxymetry reading is 92 % correct, this is the most stable pt b. Diabetic patient with blood sugar of 360, HCO3 is elevated - wrong, 360 is too much. c. Renal patient with urine specific gravity of 1.010; BUN- 24 - wrong, BUN elevated 42. What is the cause of mononucleosis? a. Caused by cytomegalovirus in the air b. Caused by cytomegalovirus thru droplet/secretions - correct 43. OB patient- who needs the most attention? a. 18 weeks complaining of muscle cramps - wrong, not the priority b. 21 weeks complaining of headache - wrong, this is common in pregnancy. c. 34 weeks complaining of shortness of breath - correct, SOB is a priority 45. Antisocial patient needs more attention when she: a. Complains about unit rules to another patient

46. Difference between slander and libel a. Healthcare worker talking about incompetence of a nurse - correct, this is slander. slander is the spoken false defamation of a person or entity b. Healthcare worker giving negative comments to a group of nurses -wrong, just expression of anger. 47. Health teaching to a pregnant woman a. Drink 4 glasses of milk per day - wrong, too much milk b. Eat a kind of vegetable and fruit each meal - correct, fairly enough veggies and fruits in the diet. 48. How will a father of an 18-month-old assess a child with otitis media? a. How frequent is the baby drinking milk during the night? - correct, because when drinking, child's ears "pop" when yawning or swallowing, the eustachian tube is adjusting the air pressure in the middle ear. 49. What will you assess in a patient with post-epidural anesthesia? a. Hypotension 50. OB patient, 38 weeks pregnant whose cervix is 5 cm dilated, intact membranes, and bulging presenting part. 0-station, asks if she can go for a walk. The nurse's best response: a. I think it would be best for you to stay in bed -wrong, contains the words I think b. It's okay for you to walk, but if the membranes will rupture, go back to your roomcorrect statement. c. It would be best for you to lie down -wrong, lying down increases blood pressure. 51. What is the intervention of a prolapsed cord? a. Insert finger into the vagina to prevent pressure and push head 52. Patient had mastectomy; what will worry the nurse? a. Husband is more intimate, looking at the operative site - wrong answer, shows good husband adjustment to the wifes illness. b. Wake up at 2:00 am, and can't go back to sleep -correct, this should alert the nurse. A sign of depression. Further interventions needed. 57. Health assessment history of a patient with endocarditis, ask: a. Did you have any infection?-correct. Bacterial infection is the most common source of endocarditis b. Did you have a sore throat 2 weeks ago? -wrong, why should it be limited to 2 weeks? 58. A patient on chemotherapy is complaining of anorexia. What food will you give? a. grapefruit juice -wrong, very acidic, may worsen anorexia and induce vomiting. b. Orange juice -wrong, somewhat helpful, but not the best choice among given. c. Eggnog and milkshake - correct, provide maximum nutrients and calories 59. What will you ask to a pt. with sickle cell disease? a. Does anybody in the family have the disease? 60. What will you expect in a patient with allergic rhinitis? a. Presence of mucous secretions in the throat

61. In a child with asthma, what health teaching needs follow-up? a. Playing with wooden puzzle - wrong, not contraindicated b. Sleeping with stuffed toy -correct, usually stuffed toys are great sources of dust, dust mites, etc. 62. A patient with BKA, how will you assess that the patient is coping? a. Fitting the prosthesis - wrong, pt. accepts this as doctors insist. b. Inspecting the stump, looking at the mirror every morning - correct, the same as with pts with mastectomy, who looks at the incisions and JP drains. 1. An adolescent with diabetes says, "There's nothing to worry about my illness". The nurse's best response is: a. You sound not worried

66. A patient is trying to pull out his NG tube. The nurse was ordered to put restraint after the patient calms down. What would be the most appropriate statement to tell the patient? a. I am sorry that we have to use the restraint because you leave us no choice - wrong explanation of the restraints. b. It's fortunate that you did not hurt other patients, but if you do it, we will restrain you wrong, because this sounds as a threat to the pt. c. I know, this is an uncomfortable procedure, but this is done for your safety and maintaining your health. 67. What is the assessment that will concern the nurse most after the patient had bronchoscopy? a. Absence of gag reflex -wrong, not a priority, gag may return soon. b. Diminished breath sounds - correct, this indicates bronchospasms, and priority at this time. 69. What is true about BSE? a. It is done 1 week after the end of menses - correct statement. Week before or week after menses this should be performed. b. It is done after ovulation -wrong, this is hard to detect. c. Face the mirror, put hands behind your hip, lean forward - wrong, d. face the mirror, raise the corresponding hand - wrong 70. A mother has history of premature labor at 35 weeks. Now she's pregnant for 32 weeks. What will you tell the patient to prevent premature labor? a. Avoid nipple manipulation - correct, this could induce Oxytocin production, and induce premature labor. b. Other option 73. A patient is being discharged. What will you teach the patient about ferrous sulfate? Choose all that apply a. Given between meals - correct, this med should be taken on an empty stomach. 1 hour before meals or 2 hours later. b. Tell patient to avoid organ meats - wrong, organ meats is a good source of iron c. Tell patient to call MD if there's a dark stool - wrong, this is normal appearance of the stool while taking this medication.

73. What is true about borderline personality disorder? a. They have history of physical abuse 75. BKA after 24 hours (position) a. Patient on his tummy several times a day 76. A patient with closed head injury complains of polydipsia. What will be the priority to monitor? a. Capillary refill - wrong, not a priority, but needs to be assessed. b. Blood glucose -wrong, not a priority, but needs to be assessed. c. Specific gravity of the urine - correct, during CHI putitary gland could be damaged, and SG should be monitored closely 77. What is a normal development in a 7-month-old baby? a. Can sit with support -wrong, should sit without support. b. Can pass object from one hand to the other - correct, normal finding. c. Can push self in upright -wrong 78. Which one is your priority for a 2-hour-old baby? a. Baby with acrocyanosis -correct, this indicates circulatory insufficiency. Can be fixed by straightening the limb. b. Vitamin K to baby with petechiae - wrong, petechiae is normal in babies

85. A telemetry patient with a pulse of 73-52. Which one will you question about the doctor's order? a. Propranolol everyday -correct, Inderal is contraindicated in pts with irregular heartbeat and slow heart rate. b. Digoxin everyday -wrong, this med is likely to be prescribed for the patient. c. Tagamet 300 mg everyday (?) - wrong, nothing is contraindicated for tagamet (heartburn relief) d. V/S every 8 hours -wrong, this cant be questioned. 86. How many ml can you give to an infant patient in IM injection? a. 0.5 ml - correct. b. 10 ml - wrong, too much even for adult c. 15 ml - wrong, the same reason as above. 87. Where can you hear vesicular sounds? a. Trachea -wrong, this is for bronchial sounds. b. Bronchi -wrong, c. At the base of the lung -correct definition. 88. Client with active TB. What will the nurse advice? a. Cover mouth with paper tissue when coughing and sneezing - correct preventive measure. b. Use surgical mask when talking to a friend - wrong, not necessary, friend should ware the mask himself, but not a surgical one

89. What do you expect in an elderly? a. Decreased salivation - wrong, this is usually due to the side effect of medications they take. b. Decrease in anteroposterior diameter - correct, normal aging process.

94. A patient underwent total gastrectomy. How would you know that the patient understood his condition? Patient needs: a. Injection of Vitamin B12 for life 95. A patient in 3rd trimester has painful bleeding. What is the suspected diagnosis? a. Abruptio placenta - correct, this is classical case b. Placenta previa -wrong, previa-painless (good to remember P_P) c. H-mole - wrong, even dont know what is that 96. A patient underwent ECT. After the procedure, the pt slept. What will you do? a. Let the patient rest - wrong b. Wake the patient up and orient - correct, this is the part of the treatment, wake the pt up and orient. 97. Blood level of digoxin is 1.8. What will you do? a. Give the next dose -correct, 0.5-2.0 - is a safe range. b. Withhold and notify the MD - wrong, not necessary. 99. A patient is in sickle cell crisis. What is your priority? a. Oxygen -wrong, in this case its not a priority. b. Fluids -correct, always a priority in SCC. 100. A patient is experiencing itchiness during blood transfusion. Your first action is to: a. Stop transfusion

Section TWO
1. Phenolketonuria (PKU) diet a. No milkshake

2. A patient's glycosylated hemoglobin assay result is 10%. What is your priority nursing diagnosis? a. Risk for injury -wrong, b. Ineffective management of the disease - correct, it is above 7% and considered abnormal. c. Risk for aspiration -wrong, totally wrong, cant be an indication for that. d. Ineffective patient teaching -wrong 3. Potassium chloride IV is given to patient. What do you check first? a. Blood pressure b. Urinary output c. Respiration d. Pulse -correct, apical pulse should be checked first. 4. Visual examination is done to a 2-year-old child. What would you use? a. Snellen's chart b. Allen color chart c. Cover right side of the eye then the left side 5. A client is in active labor. Contraction is 30-45 seconds, interval is 3-5 minutes. What would you do first? a. Reposition client to the left side b. Check the blood pressure -correct c. Apply an external fetal monitor d. Apply an internal fetal monitor 6. An infant with Trisomy 21 was admitted 3 days ago. Which behavior by one of the parents would suggest effective coping? a. One of the parent says, "I need to find a second job to cope with the needs of our family" b. The mother states, "He's so cute, I want to hug him all the time" c. The mother cries frequently d. One of the parent states, "I will stay at home all the time because the baby have some special needs. - correct, addresses the problem. 7. For a 2-year-old, what is the appropriate toy to give? a. Push-pull toy -wrong, this toy is needed in earlier age b. A racking horse correct, appropriate for the age of a child c. Playing cards -wrong, too early. d. A picture puzzle -wrong, too early. 8. What observation in an infant needs further assessment? a. A 7-month-old with negative tonic neck reflex - wrong, normal finding. b. A 12-month-old who loves staring at the colorful lanterns - correct, this needs further assessment. At this time an infant usually explores fine motor skills, not just lying in cribs. c. A 6-month-old who needs support when sitting - wrong, this doesnt require an assessment. d. An 8-month-old with positive babinski reflex -wrong, the same as above.

9. Post surgery cleft lip/palate. What statement by the mother would suggest that she understands the instructions? a. I will let my child lie prone to promote drainage of secretions b. I will let him lie on his back to promote comfort - wrong, risk for aspiration. c. I will irrigate/clean the mouth of my baby with water before and after feeding 10. A patient has a new colostomy. What indication would indicate that colostomy is starting to function? a. Flatus in the colostomy bag -correct, flatus comes first b. Moderately formed stool from the site c. No bowel sounds -wrong, indicates that peristalsis didnt return after surgery. 11. A client has new ileal conduit in place. What observation needs further assessment? a. Dark purple color of the stoma - correct, indicates ischemia. b. Cloudy drainage c. Blood in the drainage bag d. 30 ml output 12. A rape victim continuously tells the staff what has happened to her. What will you do? a. Offer her alternative measures of how to cope with the crisis b. Refer her to rape crisis center c. Continue listening to her - correct d. Encourage her to sue the assailant - wrong, not the business of the RN 13. A patient has heart failure with low cardiac output. What is your management to correct the problem? a. Elevate legs at the level of the heart - wrong, should be above. b. Elevate legs above the level of the heart - correct, this promotes venous return and will pull some fluid from the legs into circulation. c. Raise the head of bed at least 30 degrees - wrong, of no use. d. Place client in prone position - wrong, this is unsafe. Suppresses vena cava. 14. A patient with C6 injury is sweating, uneasy and diaphoretic. What will you assess? a. Check his respiration - correct, respiratory compromise symptoms b. Check patency of the urinary catheter c. Reposition client in high-fowler's position d. Reposition client side-lying 15. A patient 8 hours post-surgery complains of pain and request scheduled medication for pain. While giving the medication, the client laughs while watching TV. What will you do? a. Tell the client to decrease the dosage because it could cause addiction b. Assess the client's pain level and give the medication at the peak c. Do not give the next dose of medication d. Give the medication as prescribed - correct, the pain is measured by clients perception, not by nurses judgment.

16. A client, 32 weeks gestation, asks why she has to take betamethasone (Celestone). Tell her it is to: a. Treat premature labor contraction b. Help lungs of the unborn to mature c. Prevent infection - correct, d. Decrease clients anxiety - wrong, doesnt help with that. 17. A client, 3 hours postpartum, is bleeding profusely. Her fundus is firm and at the level of the umbilicus. What will you assess? a. Standing orders for an Oxytocin drip b. Massage the fundus c. Check the perineum (for lacerations) d. Check for bladder distention -correct, this should be assessed first 18. A client on hypothermic blanket. What observation needs further observation? a. RR= 20 b. BP=120/80 c. Pulse= 80 d. Temperature before is 38.0, now 39.70 - correct, this requires immediate attention. 19. Appropriate use of restraints a. 16-year-old, uneasy and agitated, curses staff and other patients - wrong, no evidence for harm to the staff, patients or self. b. 65-year-old patients with Alzheimer's disease who is confused and wanders at night wrong, this client needs close attention, not restraints. c. 1-month-old post-circumcision - correct, d. Other option 20. A patient with Bell's palsy understands his condition when he states: a. That the cause of his condition is viral - wrong statement. b. That situation will resolve in a couple of weeks -wrong, without treatment it doesnt happen. c. That surgery will correct his problem - correct, decompressing of the 7th nerve will correct the problem. d. That frequently tearing of his eye is caused by blockage of the lacrimal duct - wrong, the lacrimal duct works properly, but the tears cant be spread on the eye due to the lack of blinking reflex. 21. A patient has Cushing's disease. What is the appropriate diet? a. Low caloric, low sodium - correct. b. Low protein, high sodium c. High fat, high caloric d. High caloric, high sodium 22. Who is high risk of colorectal cancer?

a. A client with rectal polyp two years ago -correct. Classical case. b. A patient with duodenal ulcer with complaints of pain c. A patient with Hirschsprung's with rectal bleeding

23. A patient developed fat emboli. What will you observe first? a. Restlessness - correct, this comes first, then - increased BP. b. Increased BP -wrong, this comes second c. Decreased urine output - wrong, not indicative d. Decreased LOC -very late symptom. 24. Contraction monitoring is used for: a. Contraction and effacement b. Dilatation c. Contraction and fetal well being -correct. d. Contraction and dilatation of the cervix 25. A patient has arterial insufficiency a. Smokes 2-4 cigarettes per day - correct b. Eat fatty foods frequently -correct as well, risk for high cholesterol levels. c. Eats banana and cantaloupe d. Has a sedentary lifestyle 26. Patient on ventilator is to be weaned off with tracheostomy with cuff in place. What is the priority nursing diagnosis? a. Risk for aspiration b. Ineffective breathing pattern c. Risk for airway obstruction -correct, mucous plug can form and also because of excessive secretions. d. Decreased cardiac output 27. 12 hours after amniocentesis, client calls the hospital and complains that she's not feeling well. What will be an appropriate response by the nurse? a. Do you have fever? - indicates sepsis, which occurs after 12-24 hours after the procedure. b. Do you have difficulty breathing? 28. A patient with bulimia nervosa needs further assessment when she: a. Express anxiety upon discharge b. Comments that her dress are to fit for her - correct, always obsessed with that. c. Exercises 1 hour everyday for her to keep fit 29. Which child will be assessed for hearing loss? a. A child who has otitis media and was treated with antibiotic - correct, antibiotics are usually ototoxic. b. A child with tonsilitis with low-grade fever c. A child with URTI d. A child with asthma

30. A patient has fractured femur. Two days after application of a long leg cast, he becomes agitated and restless. What should you do initially? a. Administer prescribed O2 and notify the physician b. Raise the side rails of bed and notify the physician c. Place the patient in low fowler's position - correct, after that - call the doc. d. Administer meds for pain 31. A patient with Hodgkin's disease best room assignment a. A patient with MRSA with draining pus on skin lesions b. A patient with chickenpox c. A patient with osteomyelitis d. A patient with simple fracture of the right leg - correct. 32. A patient is post-liver biopsy. What will you do first? a. Position to right side b. Monitor or check blood pressure -correct. c. Start an IV infusion d. Administer prescribed medications for pain 33. A patient ileostomy in place. The patient understands instruction is he states: a. "I will allow the drainage bag to be 3/4 full before replacing or draining" b. "Connect the tube to a bedside drainage bag when sleeping" - correct. 34. A 3-year-old with cystic fibrosis will begin postural drainage. What is your initial nursing action? a. Check RR c. Check BP d. Determine or assess the part of the lung that has the greatest obstruction - correct. Then report the findings to the physician. 35. You are about to begin abdominal examination. What is the proper procedure? a. Auscultate first before palpation -correct. The perfect sequence would be: observe, auscultate, palpate, percuss. b. Give prescribed pain medication c. Palpate first before inspection d. Uses bell of the stethoscope 36. A patient complains of fatigue and pallor. What should you assess? a. BUN b. Creatinine c. RBC -correct. They carry oxygen to the cells, hypoxemia results in fatigue an pallor. d. WBC 37. An infant is on phototherapy. What is an appropriate nursing action? a. Cover the genitalia during phototherapy b. Cover the infant with blanket to prevent hypothermia c. Feed the infant while on phototherapy -correct. This can help your baby's body get rid of bilirubin through the urine and stools.

38. What is the clinical manifestation of a patient with pneumothorax? a. Absence of lung sounds -correct. b. Rales c. Crackles at lung bases d. Rhonchi 39. A patient with schizophrenia paranoid type is seen grimacing and clenches fist. Suddenly the patient cries "NO" in aloud manner. What will you assess first? a. Reorient and explore client's feelings and help verbalize his emotion b. Assess for adverse side effects of prescribed medications c. Assess client if he is experiencing delusions - correct. 40. Upon assessment of a 12-year-old girl, what is the appropriate occurrence at this age? a. Developed breast b. Regular menses c. Height increases 3 inches within a period of 9 months d. Has delicate features - correct. 41. High risk for iron deficiency anemia a. 12-year-old early menses - correct. b. 65-year-old chronic alcoholic -wrong, already anemic, not at risk for c. 45-year-old post menopausal d. 35-year-old multipara 42. Hospitalized patient has a husband who is an alcoholic. What statement by the patient indicates that she will not be a co-dependent? a. Says that she will find and get all the stocks of alcohol and destroy it b. Will encourage husband to exercise more and to drink less - correct, fully understands the problem of drinking. 43. Patient is post left radical mastectomy. What statement by the client indicates the understanding of discharge teaching? a. States that she will let her blood pressure be taken on the left arm b. I will continue my regular breast exam on my right breast every month - correct. c. I can do heavy lifting in my right arm as long as I have enough rest 44. A patient has chicken pox. What symptom needs further assessment? a. Skin eruptions at the back and chest b. Slight elevation of temperature c. Painful skin ulcerations at the buccal mucosa and lips - correct. d. Lesions rupture very easily 45. A patient with mitral valve regurgitaion has surgery with mechanical valve prosthesis. Which of the following findings need further assessment? a. There is a loud clicking sound when breathing -correct. b. Temperature elevation 38 degrees centigrade - wrong, this is expected during 48 hrs.

46. Before administering epidural anesthesia, the priority nursing action is: a. Hydrate the patient b. Place on prone position c. Check blood pressure -correct. Hypotension is a major concern during EA d. Check the respiratory rate 47. A patient has a cataract on his right eye. Signs and symptoms include: a. Halos around lights -correct. b. Blurring of vision c. Unsteady gait and loose balance easily 48. Priority health teaching for a 65-year-old patient a. Eats high fiber foods -correct, risk for constipation. b. Consume more of simple sugars c. Increase calcium intake 49. A patient with Parkinson's disease is taking meds. What are the side-effects of his medications? a. Pill rolling tremors -correct, sign of extrapyramidal s/e b. Increased RR c. Decreased PR d. Decreased BP 50. Safety measures by parents to prevent poisoning a. Secure covers of medicines tightly b. Place medicines in a secured locked cabinet - correct, c. Place medicines in the highest possible cabinet holder - wrong, nothing impossible for the children 51. A patient has arterial insufficiency. Assess patient for: a. +1 pedal pulses - correct b. +3 pedal pulses -wrong, this is normal c. Edematous - wrong, this is in venous insufficiency. 52. A child has acute glomerulonephritis. What observation needs immediate attention? a. Severe headache, decreased urine output -correct, early sign of renal insufficiency b. Hematuria c. Proteinuria d. Polyuria 53. A patient is on a non-rebreather mask. Which of the following needs further observation? a. PaO2= 90 % b. O2= 4 L/min c. PCO2= 65 - correct. d. PO2=92

54. Which of the following patients should you see first? a. A patient for hemodialysis, AV shunt not working and a potassium level of 5.8 Meq/Lcorrect, needs immediate attention. b. A patient with liver disease has bleeding esophageal varices and is vomiting 55. A patient with PVD has arterial insufficiency in his right leg. The nursing intervention is: a. Elevate head of the bed and legs in dependent position - correct, b. Elevate legs above the level of the heart - wrong, this is a good implementation in venous insufficiency. c. Apply elastic hose to the affected limb - wrong, same as above d. Raise foot of the bed -wrong, same as above. 56. Patient is on TPN via central line. What should you report? a. Fever of 38.0 C 57. Which would be a high fiber diet? a. Tofu and cantaloupe -correct. b. Bread and coffee c. Baked chicken and pork wrong, this is high protein diet 58. Gout diet a. No organ meat 59. A patient was taking heroine. The patient will order: a. Narcan 60. Oxytocin side effect
Side effects with oxytocin are not common. Serious side effects include:an allergic reaction (shortness of breath; closing of the throat; hives; swelling of the lips, face, or tongue; rash; or fainting); difficulty urinating; chest pain or irregular heart beat; difficulty breathing; confusion; sudden weight gain or excessive swelling; severe headache; rash; excessive vaginal bleeding; or seizures.

1. Nurse Betty is assigned to the following clients. The client that the nurse would see first after endorsement? a. A 34 year-old post operative appendectomy client of five hours who is complaining of pain. b. A 44 year-old myocardial infarction (MI) client who is complaining of nausea. c. A 26 year-old client admitted for dehydration whose intravenous (IV) has infiltrated. d. A 63 year-old post operatives abdominal hysterectomy client of three days whose incisional dressing is saturated with serosanguinous fluid.
1. Answer: (B) A 44 year-old myocardial infarction (MI) client who is complaining of nausea. Rationale: Nausea is a symptom of impending myocardial infarction (MI) and should be assessed immediately so that treatment can be instituted and further damage to the heart is avoided.

2. Nurse Gail places a client in a four-point restraint following orders from the physician. The client care plan should include: a. Assess temperature frequently. b. Provide diversional activities. c. Check circulation every 15-30 minutes. d. Socialize with other patients once a shift.
2. Answer: (C) Check circulation every 15-30 minutes. Rationale: Restraints encircle the limbs, which place the client at risk for circulation being restricted to the distal areas of the extremities. Checking the clients circulation every 15-30 minutes will allow the nurse to adjust the restraints before injury from decreased blood flow occurs.

3. A male client who has severe burns is receiving H2 receptor antagonist therapy. The nurse Incharge knows the purpose of this therapy is to: a. Prevent stress ulcer b. Block prostaglandin synthesis c. Facilitate protein synthesis. d. Enhance gas exchange
3. Answer: (A) Prevent stress ulcer Rationale: Curlings ulcer occurs as a generalized stress response in burn patients. This results in a decreased production of mucus and increased secretion of gastric acid. The best treatment for this prophylactic use of antacids and H2 receptor blockers.

4. The doctor orders hourly urine output measurement for a postoperative male client. The nurse Trish records the following amounts of output for 2 consecutive hours: 8 a.m.: 50 ml; 9 a.m.: 60 ml. Based on these amounts, which action should the nurse take? a. Increase the I.V. fluid infusion rate b. Irrigate the indwelling urinary catheter c. Notify the physician d. Continue to monitor and record hourly urine output

4. Answer: (D) Continue to monitor and record hourly urine output Rationale: Normal urine output for an adult is approximately 1 ml/minute (60 ml/hour). Therefore, this client's output is normal. Beyond continued evaluation, no nursing action is warranted.

5. Tony, a basketball player twist his right ankle while playing on the court and seeks care for ankle pain and swelling. After the nurse applies ice to the ankle for 30 minutes, which statement by Tony suggests that ice application has been effective? a. My ankle looks less swollen now. b. My ankle feels warm. c. My ankle appears redder now. d. I need something stronger for pain relief
5. Answer: (A) My ankle looks less swollen now Rationale: Ice application decreases pain and swelling. Continued or increased pain, redness, and increased warmth are signs of inflammation that shouldn't occur after ice application

6.The physician prescribes a loop diuretic for a client. When administering this drug, the nurse anticipates that the client may develop which electrolyte imbalance? a. Hypernatremia b. Hyperkalemia c. Hypokalemia d. Hypervolemia
Answer: (C) Hypokalemia Rationale: A loop diuretic removes water and, along with it, sodium and potassium. This may result in hypokalemia, hypovolemia, and hyponatremia.

7.She finds out that some managers have benevolent-authoritative style of management. Which of the following behaviors will she exhibit most likely? a. Have condescending trust and confidence in their subordinates. b. Gives economic and ego awards. c. Communicates downward to staffs. d. Allows decision making among subordinates.
Answer:(A) Have condescending trust and confidence in their subordinates Rationale: Benevolent-authoritative managers pretentiously show their trust and confidence to their followers.

8. Nurse Amy is aware that the following is true about functional nursing a. Provides continuous, coordinated and comprehensive nursing services. b. One-to-one nurse patient ratio. c. Emphasize the use of group collaboration. d. Concentrates on tasks and activities.
(A) Provides continuous, coordinated and comprehensive nursing services. Rationale: Functional nursing is focused on tasks and activities and not on the care of the patients.

9.Which type of medication order might read "Vitamin K 10 mg I.M. daily 3 days?" a. Single order b. Standard written order c. Standing order d. Stat order
(B) Standard written order Rationale: This is a standard written order. Prescribers write a single order for medications given only once. A stat order is written for medications given immediately for an urgent client problem. A standing order, also known as a protocol, establishes guidelines for treating a particular disease or set of symptoms in special care areas such as the coronary care unit. Facilities also may institute medication protocols that specifically designate drugs that a nurse may not give.

10.A female client with a fecal impaction frequently exhibits which clinical manifestation? a. Increased appetite b. Loss of urge to defecate c. Hard, brown, formed stools d. Liquid or semi-liquid stools
(D) Liquid or semi-liquid stools Rationale: Passage of liquid or semi-liquid stools results from seepage of unformed bowel contents around the impacted stool in the rectum. Clients with fecal impaction don't pass hard, brown, formed stools because the feces can't move past the impaction. These clients typically report the urge to defecate (although they can't pass stool) and a decreased appetite.

11.Nurse Linda prepares to perform an otoscopic examination on a female client. For proper visualization, the nurse should position the client's ear by: a. Pulling the lobule down and back b. Pulling the helix up and forward c. Pulling the helix up and back d. Pulling the lobule down and forward
(C) Pulling the helix up and back Rationale: To perform an otoscopic examination on an adult, the nurse grasps the helix of the ear and pulls it up and back to straighten the ear canal. For a child, the nurse grasps the helix and pulls it down to straighten the ear canal. Pulling the lobule in any direction wouldn't straighten the ear canal for visualization.

12. Nurse Michelle is assessing a 24 year old client with a diagnosis of hydatidiform mole. She is aware that one of the following is unassociated with this condition? a. Excessive fetal activity. b. Larger than normal uterus for gestational age. c. Vaginal bleeding d. Elevated levels of human chorionic gonadotropin. (A) Excessive fetal activity. Rationale: The most common signs and symptoms of hydatidiform mole includes elevated levels of human chorionic gonadotropin, vaginal bleeding, larger than normal uterus for gestational age, failure to detect fetal heart activity even with sensitive instruments, excessive nausea and vomiting, and early development of pregnancy-induced hypertension. Fetal activity would not be noted.

13. A pregnant client is receiving magnesium sulfate for severe pregnancy induced hypertension (PIH). The clinical findings that would warrant use of the antidote , calcium gluconate is: a. Urinary output 90 cc in 2 hours. b. Absent patellar reflexes. c. Rapid respiratory rate above 40/min. d. Rapid rise in blood pressure. (B) Absent patellar reflexes Rationale: Absence of patellar reflexes is an indicator of hypermagnesemia, which requires administration of calcium gluconate.

14. During vaginal examination of Janah who is in labor, the presenting part is at station plus two. Nurse, correctly interprets it as:

a. Presenting part is 2 cm above the plane of the ischial spines. b. Biparietal diameter is at the level of the ischial spines. c. Presenting part in 2 cm below the plane of the ischial spines. d. Biparietal diameter is 2 cm above the ischial spines. 14. Answer: (C) Presenting part in 2 cm below the plane of the ischial spines. Rationale: Fetus at station plus two indicates that the presenting part is 2 cm below the plane of the ischial spines.

15. A pregnant client is receiving oxytocin (Pitocin) for induction of labor. A condition that warrant the nurse in-charge to discontinue I.V. infusion of Pitocin is: a. Contractions every 1 minutes lasting 70-80 seconds. b. Maternal temperature 101.2 c. Early decelerations in the fetal heart rate. d. Fetal heart rate baseline 140-160 bpm. 15. Answer: (A) Contractions every 1 minutes lasting 70-80 seconds. Rationale: Contractions every 1 minutes lasting 70-80 seconds, is indicative of hyperstimulation of the uterus, which could result in injury to the mother and the fetus if Pitocin is not discontinued.

16. Calcium gluconate is being administered to a client with pregnancy induced hypertension (PIH). A nursing action that must be initiated as the plan of care throughout injection of the drug is: a. Ventilator assistance b. CVP readings c. EKG tracings d. Continuous CPR 16. Answer: (C) EKG tracings Rationale: A potential side effect of calcium gluconate administration is cardiac arrest. Continuous monitoring of cardiac activity (EKG) throught administration of calcium gluconate is an essential part of care.

17. A trial for vaginal delivery after an earlier caesareans, would likely to be given to a gravida, who had:

a. First low transverse cesarean was for active herpes type 2 infections; vaginal culture at 39 weeks pregnancy was positive. b. First and second caesareans were for cephalopelvic disproportion. c. First caesarean through a classic incision as a result of severe fetal distress. d. First low transverse caesarean was for breech position. Fetus in this pregnancy is in a vertex presentation. Answer: (D) First low transverse caesarean was for breech position. Fetus in this pregnancy is in a vertex presentation. Rationale: This type of client has no obstetrical indication for a caesarean section as she did with her first caesarean delivery. 18.Nurse Ryan is aware that the best initial approach when trying to take a crying toddlers temperature is: a. Talk to the mother first and then to the toddler. b. Bring extra help so it can be done quickly. c. Encourage the mother to hold the child. d. Ignore the crying and screaming. 18. Answer: (A) Talk to the mother first and then to the toddler. Rationale: When dealing with a crying toddler, the best approach is to talk to the mother and ignore the toddler first. This approach helps the toddler get used to the nurse before she attempts any procedures. It also gives the toddler an opportunity to see that the mother trusts the nurse.

19.Baby Tina a 3 month old infant just had a cleft lip and palate repair. What should the nurse do to prevent trauma to operative site? a. Avoid touching the suture line, even when cleaning. b. Place the baby in prone position. c. Give the baby a pacifier. d. Place the infants arms in soft elbow restraints. 19. Answer: (D) Place the infants arms in soft elbow restraints. Rationale: Soft restraints from the upper arm to the wrist prevent the infant from touching her lip but allow him to hold a favorite item such as a blanket. Because they could damage the operative site, such as objects as pacifiers, suction catheters, and small spoons shouldnt be placed in a babys mouth after cleft repair. A baby in a prone position may rub her face on the sheets and traumatize the operative site. The suture line should be cleaned gently to prevent infection, which could interfere with healing and damage the cosmetic appearance of the repair.

20. Which action should nurse Marian include in the care plan for a 2 month old with heart failure? a. Feed the infant when he cries.

b. Allow the infant to rest before feeding. c. Bathe the infant and administer medications before feeding. d. Weigh and bathe the infant before feeding. 20. Answer: (B) Allow the infant to rest before feeding. Rationale: Because feeding requires so much energy, an infant with heart failure should rest before feeding. 21.Nurse Hazel is teaching a mother who plans to discontinue breast feeding after 5 months. The nurse should advise her to include which foods in her infants diet? a. Skim milk and baby food. b. Whole milk and baby food. c. Iron-rich formula only. d. Iron-rich formula and baby food. 21. Answer: (C) Iron-rich formula only. Rationale: The infants at age 5 months should receive iron-rich formula and that they shouldnt receive solid food, even baby food until age 6 months.

22. Nurse Patricia finds a female client who is post-myocardial infarction (MI) slumped on the side rails of the bed and unresponsive to shaking or shouting. Which is the nurse next action? a. Call for help and note the time. b. Clear the airway c. Give two sharp thumps to the precordium, and check the pulse. d. Administer two quick blows. 22. Answer: (A) Call for help and note the time. Rationale: Having established, by stimulating the client, that the client is unconscious rather than sleep, the nurse should immediately call for help. This may be done by dialing the operator from the clients phone and giving the hospital code for cardiac arrest and the clients room number to the operator, of if the phone is not available, by pulling the emergency call button. Noting the time is important baseline information for cardiac arrest procedure.

23. Nurse Monett is caring for a client recovering from gastro-intestinal bleeding. The nurse should: a. Plan care so the client can receive 8 hours of uninterrupted sleep each night. b. Monitor vital signs every 2 hours. c. Make sure that the client takes food and medications at prescribed intervals. d. Provide milk every 2 to 3 hours. 23. Answer: (C) Make sure that the client takes food and medications at prescribed intervals.

Rationale: Food and drug therapy will prevent the accumulation of hydrochloric acid, or will neutralize and buffer the acid that does accumulate.

24. A male client was on warfarin (Coumadin) before admission, and has been receiving heparin I.V. for 2 days. The partial thromboplastin time (PTT) is 68 seconds. What should Nurse Carla do? a. Stop the I.V. infusion of heparin and notify the physician. b. Continue treatment as ordered. c. Expect the warfarin to increase the PTT. d. Increase the dosage, because the level is lower than normal. 24. Answer: (B) Continue treatment as ordered. Rationale: The effects of heparin are monitored by the PTT is normally 30 to 45 seconds; the therapeutic level is 1.5 to 2 times the normal level.

25. A client undergone ileostomy, when should the drainage appliance be applied to the stoma? a. 24 hours later, when edema has subsided. b. In the operating room. c. After the ileostomy begin to function. d. When the client is able to begin self-care procedures. Answer: (B) In the operating room. Rationale: The stoma drainage bag is applied in the operating room. Drainage from the ileostomy contains secretions that are rich in digestive enzymes and highly irritating to the skin. Protection of the skin from the effects of these enzymes is begun at once. Skin exposed to these enzymes even for a short time becomes reddened, painful, and excoriated.

26. A client undergone spinal anesthetic, it will be important that the nurse immediately position the client in: a. On the side, to prevent obstruction of airway by tongue.

b. Flat on back. c. On the back, with knees flexed 15 degrees. d. Flat on the stomach, with the head turned to the side. 26. Answer: (B) Flat on back. Rationale: To avoid the complication of a painful spinal headache that can last for several days, the client is kept in flat in a supine position for approximately 4 to 12 hours postoperatively. Headaches are believed to be causes by the seepage of cerebral spinal fluid from the puncture site. By keeping the client flat, cerebral spinal fluid pressures are equalized, which avoids trauma to the neurons.

27.While monitoring a male client several hours after a motor vehicle accident, which assessment data suggest increasing intracranial pressure? a. Blood pressure is decreased from 160/90 to 110/70. b. Pulse is increased from 87 to 95, with an occasional skipped beat. c. The client is oriented when aroused from sleep, and goes back to sleep immediately. d. The client refuses dinner because of anorexia. 27. Answer: (C) The client is oriented when aroused from sleep, and goes back to sleep immediately. Rationale: This finding suggest that the level of consciousness is decreasing.

28.Mrs. Cruz, 80 years old is diagnosed with pneumonia. Which of the following symptoms may appear first? a. Altered mental status and dehydration b. Fever and chills c. Hemoptysis and Dyspnea d. Pleuritic chest pain and cough 28. Answer: (A) Altered mental status and dehydration Rationale: Fever, chills, hemortysis, dyspnea, cough, and pleuritic chest pain are the common symptoms of pneumonia, but elderly clients may first appear with only an altered lentil status and dehydration due to a blunted immune response.

29. A male client has active tuberculosis (TB). Which of the following symptoms will be exhibit? a. Chest and lower back pain

b. Chills, fever, night sweats, and hemoptysis c. Fever of more than 104F (40C) and nausea d. Headache and photophobia 29. Answer: (B) Chills, fever, night sweats, and hemoptysis Rationale: Typical signs and symptoms are chills, fever, night sweats, and hemoptysis. Chest pain may be present from coughing, but isnt usual. Clients with TB typically have low-grade fevers, not higher than 102F (38.9C). Nausea, headache, and photophobia arent usual TB symptoms.

30. Mark, a 7-year-old client is brought to the emergency department. Hes tachypneic and afebrile and has a respiratory rate of 36 breaths/minute and has a nonproductive cough. He recently had a cold. Form this history; the client may have which of the following conditions? a. Acute asthma b. Bronchial pneumonia c. Chronic obstructive pulmonary disease (COPD) d. Emphysema 30. Answer:(A) Acute asthma Rationale: Based on the clients history and symptoms, acute asthma is the most likely diagnosis. Hes unlikely to have bronchial pneumonia without a productive cough and fever and hes too young to have developed (COPD) and emphysema.

31. Marichu was given morphine sulfate for pain. She is sleeping and her respiratory rate is 4 breaths/minute. If action isnt taken quickly, she might have which of the following reactions? a. Asthma attack b. Respiratory arrest c. Seizure d. Wake up on his own 31. Answer: (B) Respiratory arrest Rationale: Narcotics can cause respiratory arrest if given in large quantities. Its unlikely the client will have asthma attack or a seizure or wake up on his own.

32. A 77-year-old male client is admitted for elective knee surgery. Physical examination reveals shallow respirations but no sign of respiratory distress. Which of the following is a normal physiologic change related to aging? a. Increased elastic recoil of the lungs b. Increased number of functional capillaries in the alveoli

c. Decreased residual volume d. Decreased vital capacity 32. Answer: (D) Decreased vital capacity Rationale: Reduction in vital capacity is a normal physiologic changes include decreased elastic recoil of the lungs, fewer functional capillaries in the alveoli, and an increased in residual volume.

33. Tracy is receiving combination chemotherapy for treatment of metastatic carcinoma. Nurse Ruby should monitor the client for the systemic side effect of: a. Ascites b. Nystagmus c. Leukopenia d. Polycythemia 33. Answer: (C) Leukopenia Rationale: Leukopenia, a reduction in WBCs, is a systemic effect of chemotherapy as a result of myelosuppression.

34. Norma, with recent colostomy expresses concern about the inability to control the passage of gas. Nurse Oliver should suggest that the client plan to: a. Eliminate foods high in cellulose. b. Decrease fluid intake at meal times. c. Avoid foods that in the past caused flatus. d. Adhere to a bland diet prior to social events. 34. Answer: (C) Avoid foods that in the past caused flatus. Rationale: Foods that bothered a person preoperatively will continue to do so after a colostomy.

35. Nurse Ron begins to teach a male client how to perform colostomy irrigations. The nurse would evaluate that the instructions were understood when the client states, I should: a. Lie on my left side while instilling the irrigating solution. b. Keep the irrigating container less than 18 inches above the stoma. c. Instill a minimum of 1200 ml of irrigating solution to stimulate evacuation of the bowel.

d. Insert the irrigating catheter deeper into the stoma if cramping occurs during the procedure. 35. Answer: (B) Keep the irrigating container less than 18 inches above the stoma. Rationale: This height permits the solution to flow slowly with little force so that excessive peristalsis is not immediately precipitated.

36. Patrick is in the oliguric phase of acute tubular necrosis and is experiencing fluid and electrolyte imbalances. The client is somewhat confused and complains of nausea and muscle weakness. As part of the prescribed therapy to correct this electrolyte imbalance, the nurse would expect to: a. Administer Kayexalate b. Restrict foods high in protein c. Increase oral intake of cheese and milk. d. Administer large amounts of normal saline via I.V. 36. Answer: (A) Administer Kayexalate Rationale: Kayexalate,a potassium exchange resin, permits sodium to be exchanged for potassium in the intestine, reducing the serum potassium level.

37. Mario has burn injury. After Forty48 hours, the physician orders for Mario 2 liters of IV fluid to be administered q12 h. The drop factor of the tubing is 10 gtt/ml. The nurse should set the flow to provide: a. 18 gtt/min b. 28 gtt/min c. 32 gtt/min d. 36 gtt/min Answer:(B) 28 gtt/min Rationale: This is the correct flow rate; multiply the amount to be infused (2000 ml) by the drop factor (10) and divide the result by the amount of time in minutes (12 hours x 60 minutes)

38.Terence suffered form burn injury. Using the rule of nines, which has the largest percent of burns? a. Face and neck b. Right upper arm and penis c. Right thigh and penis d. Upper trunk Answer: (D) Upper trunk

Rationale: The percentage designated for each burned part of the body using the rule of nines: Head and neck 9%; Right upper extremity 9%; Left upper extremity 9%; Anterior trunk 18%; Posterior trunk 18%; Right lower extremity 18%; Left lower extremity 18%; Perineum 1%.

39. Herbert, a 45 year old construction engineer is brought to the hospital unconscious after falling from a 2-story building. When assessing the client, the nurse would be most concerned if the assessment revealed: a. Reactive pupils b. A depressed fontanel c. Bleeding from ears d. An elevated temperature 39. Answer: (C) Bleeding from ears Rationale: The nurse needs to perform a thorough assessment that could indicate alterations in cerebral function, increased intracranial pressures, fractures and bleeding. Bleeding from the ears occurs only with basal skull fractures that can easily contribute to increased intracranial pressure and brain herniation.

40. Nurse Sherry is teaching male client regarding his permanent artificial pacemaker. Which information given by the nurse shows her knowledge deficit about the artificial cardiac pacemaker? a. take the pulse rate once a day, in the morning upon awakening b. May be allowed to use electrical appliances c. Have regular follow up care d. May engage in contact sports 40. Answer: (D) may engage in contact sports Rationale: The client should be advised by the nurse to avoid contact sports. This will prevent trauma to the area of the pacemaker generator.

41.The nurse is ware that the most relevant knowledge about oxygen administration to a male client with COPD is a. Oxygen at 1-2L/min is given to maintain the hypoxic stimulus for breathing. b. Hypoxia stimulates the central chemoreceptors in the medulla that makes the client breath. c. Oxygen is administered best using a non-rebreathing mask d. Blood gases are monitored using a pulse oximeter. 41. Answer: (A) Oxygen at 1-2L/min is given to maintain the hypoxic stimulus for breathing. Rationale: COPD causes a chronic CO2 retention that renders the medulla insensitive to the CO2 stimulation for breathing. The hypoxic state of the client then becomes the stimulus for breathing. Giving the client oxygen in low concentrations will maintain the clients hypoxic drive.

42.Tonny has undergoes a left thoracotomy and a partial pneumonectomy. Chest tubes are inserted, and one-bottle water-seal drainage is instituted in the operating room. In the postanesthesia care unit Tonny is placed in Fowler's position on either his right side or on his back. The nurse is aware that this position: a. Reduce incisional pain. b. Facilitate ventilation of the left lung. c. Equalize pressure in the pleural space. d. Increase venous return 42. Answer: (B) Facilitate ventilation of the left lung. Rationale: Since only a partial pneumonectomy is done, there is a need to promote expansion of this remaining Left lung by positioning the client on the opposite unoperated side.

43. What is Nurse John likely to note in a male client being admitted for alcohol withdrawal? a. Perceptual disorders. b. Impending coma. c. Recent alcohol intake. d. Depression with mutism. 43. Answer: (A) Perceptual disorders. Rationale: Frightening visual hallucinations are especially common in clients experiencing alcohol withdrawal 44. Aira has taken amitriptyline HCL (Elavil) for 3 days, but now complains that it doesnt help and refuses to take it. What should the nurse say or do? a. Withhold the drug. b. Record the clients response. c. Encourage the client to tell the doctor. d. Suggest that it takes awhile before seeing the results. 44. Answer: (D) Suggest that it takes awhile before seeing the results. Rationale: The client needs a specific response; that it takes 2 to 3 weeks (a delayed effect) until the therapeutic blood level is reached.

45. Dervid, an adolescent has a history of truancy from school, running away from home and barrowing other peoples things without their permission. The adolescent denies stealing, rationalizing instead that as long as no one was using the items, it was all right to borrow them. It is important for the nurse to understand the psychodynamically, this behavior may be largely attributed to a developmental defect related to the: a. Id b. Ego c. Superego d. Oedipal complex 45. Answer: (C) Superego Rationale: This behavior shows a weak sense of moral consciousness. According to Freudian theory, personality disorders stem from a weak superego.

46. In preparing a female client for electroconvulsive therapy (ECT), Nurse Michelle knows that succinylcoline (Anectine) will be administered for which therapeutic effect? a. Short-acting anesthesia b. Decreased oral and respiratory secretions. c. Skeletal muscle paralysis. d. Analgesia. 46. Answer: (C) Skeletal muscle paralysis. Rationale: Anectine is a depolarizing muscle relaxant causing paralysis. It is used to reduce the intensity of muscle contractions during the convulsive stage, thereby reducing the risk of bone fractures or dislocation.

47. Nurse Gina is aware that the dietary implications for a client in manic phase of bipolar disorder is: a. Serve the client a bowl of soup, buttered French bread, and apple slices. b. Increase calories, decrease fat, and decrease protein. c. Give the client pieces of cut-up steak, carrots, and an apple. d. Increase calories, carbohydrates, and protein. Answer: (D) Increase calories, carbohydrates, and protein. Rationale: This client increased protein for tissue building and increased calories to replace what is burned up (usually via carbohydrates).

48.What parental behavior toward a child during an admission procedure should cause Nurse Ron to suspect child abuse? a. Flat affect b. Expressing guilt c. Acting overly solicitous toward the child. d. Ignoring the child. Answer: (C) Acting overly solicitous toward the child. Rationale: This behavior is an example of reaction formation, a coping mechanism.

49.Nurse Lynnette notices that a female client with obsessive-compulsive disorder washes her hands for long periods each day. How should the nurse respond to this compulsive behavior? a. By designating times during which the client can focus on the behavior. b. By urging the client to reduce the frequency of the behavior as rapidly as possible. c. By calling attention to or attempting to prevent the behavior. d. By discouraging the client from verbalizing anxieties. Answer: (A) By designating times during which the client can focus on the behavior. Rationale: The nurse should designate times during which the client can focus on the compulsive behavior or obsessive thoughts. The nurse should urge the client to reduce the frequency of the compulsive behavior gradually, not rapidly. She shouldn't call attention to or try to prevent the behavior. Trying to prevent the behavior may cause pain and terror in the client. The nurse should encourage the client to verbalize anxieties to help distract attention from the compulsive behavior.

50.After seeking help at an outpatient mental health clinic, Ruby who was raped while walking her dog is diagnosed with posttraumatic stress disorder (PTSD). Three months later, Ruby returns to the clinic, complaining of fear, loss of control, and helpless feelings. Which nursing intervention is most appropriate for Ruby? a. Recommending a high-protein, low-fat diet. b. Giving sleep medication, as prescribed, to restore a normal sleepwake cycle. c. Allowing the client time to heal. d. Exploring the meaning of the traumatic event with the client. Answer: (D) Exploring the meaning of the traumatic event with the client. Rationale: The client with PTSD needs encouragement to examine and understand the meaning of the traumatic event and consequent losses. Otherwise, symptoms may worsen and the client may become depressed or engage in self-destructive behavior such as substance abuse. The client must explore the meaning of the event and won't heal without this, no matter how much time passes. Behavioral techniques, such as relaxation therapy, may help decrease the client's anxiety and induce sleep. The physician may prescribe antianxiety agents or antidepressants cautiously to avoid dependence; sleep medication is rarely appropriate. A special diet isn't indicated unless the client also has an eating disorder or a nutritional problem.

Medical Surgical Nursing Pre-test 2


Thread Started on Aug 18, 2010, 1:20pm

1. When teaching a client about propranolol hydrochloride, the nurse should base the information on the knowledge that propranolol hydrochloride a. Blocks beta-adrenergic stimulation and thus causes decreased heart rate, myocardial contractility, and conduction. b. Increases norepinephrine secretion and thus decreases blood pressure and heart rate. c. Is a potent arterial and venous vasodilator that reduces peripheral vascular resistance and lowers blood pressure. d. Is an angiotensin-converting enzyme (ACE) inhibitor that reduces blood pressure by blocking the conversion of angiotensin I to angiotensin II. Ans: A propranolol is -adrenergic blocking agent. Actions of propranolol hydrochloride include reducing heart rate, decreasing myocardial contractility, and slowing conduction. 2. The nurse understands that a priority nursing diagnosis for the client with hypertension would be a. Pain. b. Deficient Fluid Volume. c. Impaired skin integrity. d. Ineffective health maintenance. Ans: D managing hypertension is a priority for the client with hypertension. Clients with hypertension frequently do not experience other signs and symptoms such as pain, deficient fluid volume, or impaired skin integrity. It is the asymptomatic nature of hypertension that makes it so difficult to treat, because clients may not recognize they are hypertensive or may not perceive the need for aggressive management of the disease. 3. The most important long-term goal for a client with hypertension would be to a. Learn how to avoid stress. b. Explore a job change or early retirement. c. Make a commitment to long-term therapy. d. Control high blood pressure. Ans: C compliance is the most critical element of hypertension therapy. In most cases, hypertensive clients require lifelong treatment and their hypertension cannot be managed successfully without during therapy. Stress management and weight management are important components of hypertension therapy, but the priority goal is related to compliance. 4. The client with hypertension is prone to long-term complications of the disease. Which of the following is a longterm complication of hypertension? a. Renal insufficiency and failure. b. Valvular heart disease. c. Endocarditis d. Peptic ulcer disease. Ans: A renal disease, including renal insufficiency and failure is a complication of hypertension. effective treatment of hypertension assists in preventing this compliance valvular heart disease, endocarditis, and peptic ulcer disease are not complications of hypertension. 5. Hypertension is known as the silent killer. This phrase is associated with the fact that hypertension often goes undetected until symptoms of other system failures occur. This may occur in the form of a. Cerebrovascular accidents (CVAs) b. Liver disease. c. Myocardial infarction. d. Pulmonary disease. Ans: A hypertension is referred to as the silent killer for adults, because until the adult has significant damage to others systems, the hypertension may go undetected. CVAs can be related to long-term hypertension. Liver or pulmonary disease is not generally associated with hypertension. Myocardial infraction is generally related to coronary artery disease.

6. During the past few months, a 56-year old woman has felt brief twinges of chest pain while working in her garden and has had frequent episodes of indigestion. She comes to the hospital after experiencing severe anterior chest pain while raking leaves. Her evaluation confirms a diagnosis of stable angina pectoris. After stabilization and treatment, the client is discharged from the hospital. At her follow-up appointment, she is discouraged because she is experiencing pain with increasing frequency. She states that she visits an invalid friend twice a week and now cannot walk up the second flight of steps to the friends apartment without pain. Which of the following measures that the nurse could suggest would most likely help the client deal with this problem? a. Visit her friend b. Rest for at least an hour before climbing the stairs c. Take a nitroglycerin tablet before climbing the stairs. d. Lie down once she reaches the friends apartment. Ans: C nitroglycerin may be used prophylactically before stressful physical activities such as stair-climbing to help the client remain pain free. Visiting her friend early in the day would have no impact on decreasing pain episodes. Resting before or after an activity is not as likely to help prevent an activity-related pain episode. 7. The client who experiences angina pectoris has been told to follow a low-cholesterol diet. Which of the following meals should the nurse tell the client would be best on her low cholesterol diet? a. Hamburger, salad, and milkshake. b. Baked liver, green beans, and coffee. c. Spaghetti with tomato sauce, salad, and coffee d. Fried chicken, green beans, and skim milk Ans: C pasta, tomato sauce, salad, and coffee would be the best selection for the client following a low-cholesterol diet. Hamburgers, milkshakes, liver, and fried foods tend to be high in cholesterol. 8. Which of the following symptoms should the nurse teach the client with unstable angina to report immediately to her physician? a. A change in the pattern of her pain b. Pain during sexual activity c. Pain during an argument with her husband d. Pain during or after an activity such as lawn mowing Ans: A the client should report a change in the pattern of chest pain. It may help increasing severity of coronary artery disease. Pain occurring during stress or sexuality activity would not be unexpected, and the client may be instructed to take nitroglycerin to prevent this pain. Pain during or after an activity such as lawn mowing also would not be unexpected; the client may be instructed to take nitroglycerin to prevent this pain or may be restricted from doing such activities. 9. The physician refers the client with unstable angina for a cardiac catheterization. The nurse explains to the client that this procedure is being used in this specific case to: a. Open and dilate blocked coronary arteries b. Assess the extent of arterial blockage c. Bypass obstructed vessels d. Assess the functional adequacy of the valves and heart muscle Ans: B cardiac catheterization is done in clients with angina primarily to assess the extent and severity of the coronary artery blockage. A decision about medical management, angioplasty, or coronary artery bypass surgery will be based on the catheterization results. Coronary bypass surgery would be used to bypass obstructed vessels. Although cardiac catheterization can be used to assess the functional adequacy of the valves and heart muscle, in this case the client has unstable angina and therefore would need the procedure to assess the extent of arterial blockage.

10. The client is scheduled for a percutaneous transluminal coronary angioplasty (PTCA) to treat angina. Priority goals for the client immediately after PTCA would include: a. Minimizing dyspnea b. Maintaining adequate blood pressure control c. Decreasing myocardial contractility d. Preventing fluid volume deficit Ans: D because the contrast medium used in PTCA acts as an osmotic diuretic, the client may experience diuresis with resultant fluid volume deficit after the procedure. Additionally, potassium levels must be closely monitored because the client may develop hypokalemia due to the diuresis. Dyspnea would not be anticipated after this procedure. Maintaining adequate blood pressure control should not be a problem after the procedure. Increased myocardial contractility would be a goal, not decreased contractility. 11. Which of the following is not generally considered to be a risk factor for the development of atheroclerosis? a. Family history of early heart attack b. Late onset of puberty c. Total blood cholesterol level greater than 220 mg/dL d. Elevated fasting blood sugar concentration Ans: B late onset of puberty is not generally considered to be a risk factor of the development of atherosclerosis. Risk factors for atherosclerosis include cigarette smoking, hypertension, high blood cholesterol level, male gender, family history of atherosclerosis, diabetes mellitus, obesity, and physical inactivity. 12. Many more men than women younger than 50 years of age have coronary artery disease as a result of atherosclerosis. The leading cause of death in women is: a. Acquired immunodeficiency syndrome b. Breast cancer c. Coronary artery disease d. Chronic obstructive pulmonary disease Ans: C coronary artery disease is the leading cause of dearth in women as well as men. Although it is generally agreed that estrogen helps protect women from atherosclerotic changes before menopause, women are still at risk for coronary artery disease. Much attention has been focused on the lack of research studies dealing with cardiac disease in women and minorities, and work is under way to gain a better understanding of cardiac disease in these populations. 13. A client angina asks the nurse, What information does an ECG provide? The nurse would respond that an electrocardiogram (ECG) primarily gives information about the: a. Electrical conduction of the myocardium b. Oxygenation and perfusion of the heart c. Contractile status of the ventricles d. Physical integrity of the heart muscle Ans: A an ECG directly reflects the transmission of electrical cardiac impulses through the heart. This information makes it possible to evaluate indirectly the functional status of the heart muscle and the contractile response of the ventricles. However, these elements are not measured directly. The ECG does not give information about the oxygenation and perfusion of the heart. 14. As an initial step in treating a client with angina, the physician prescribes nitroglycerin tablets, 0.3 mg given sublingually. This drugs principal effects are produced by: a. Antispasmodic effects on the pericardium b. Causing an increased myocardial oxygen demand c. Vasodilation of peripheral vasculature d. Improved conductivity in the myocardium Ans: C nitroglycerin produces peripheral vasodilation, which reduces myocardial oxygen consumption and demand. Vasodilation in coronary arteries and collateral vessels may also increase blood flow to the ischemic areas of the heart. Nitroglycerin decreases myocardial oxygen demand. Nitroglycerin does not have an effect on pericardial spasticity or conductivity in the myocardium.

15. The nurse teaches the client with angina about the common expected side effects of nitroglycerin, including: a. Headache b. High blood pressure c. Shortness of breath d. Stomach cramps Ans: A because of its widespread vasodilating effects, nitroglycerin often produces such as side effects as headache, hypotension, and dizziness. The client should sit or lie down to avoid fainting. Nitroglycerin does not cause shortness of breath or stomach cramps. 16. Sublingual nitroglycerin tablets begin to work within 1 to 2 minutes. How should the nurse instruct the client to use the drug when chest pain occurs? a. Take one tablet every 2 to 5 minutes until the pain stops b. Take one tablet and rest for 10 minutes. Call the physician if pain persists after 10 minutes c. Take one tablet, then an additional tablet every 5 minutes for a total of three tablets. Call the physician if pain persists after these tablets d. Take one tablet. If pain still persists 5 minutes later, call the physician Ans: C the correct protocol for nitroglycerin use involves immediate administration, with subsequent doses taken at 5-minute intervals as needed, for a total dose of three tablets. Sublingual nitroglycerin appears in the bloodstream within 2 to 3 minutes and is metabolized within about 10 minutes. 17. A client with angina has been taking nifedipine. The client should be taught to: a. Monitor blood pressure monthly b. Perform daily weights c. Inspect gums daily d. Limit intake of green leafy vegetables Ans: C the client taking nifedipine should inspect the gums daily to monitor for gingival hyperplasia. This is an uncommon side effect but one that requires monitoring and intervention if it occurs. The client taking nifedipine might be taught to monitor blood pressure, but more than monthly. These clients would not generally need to perform daily weights or limit intake of green leafy vegetables. The Client With A Permanent Pacemaker 18. A 74-year-old woman is admitted to the telemetry unit for placement of a permanent pacemaker would be to: a. Maintain skin integrity b. Maintain cardiac conduction stability c. Decrease cardiac output d. Increase activity level Ans: B maintaining cardiac conduction stability to prevent dysrythmias is a priority immediately after artificial pacemaker implantation. The client should have continuous electrocardiographic (ECG) monitoring until proper pacemaker functioning is verified. 19. The client who had a permanent pacemaker implanted 2 days earlier is being discharged from the hospital. Outcome criteria include that the client: a. Selects a low-cholesterol diet to control coronary artery disease b. States a need for bed rest for 1 week after discharge c. Verbalizes safety precautions needed to prevent pacemaker malfunction d. Explain sign and symptoms of myocardial infraction Ans: C education is a major component of the discharge plan for a client with an artificial pacemaker. The client with a permanent pacemaker needs to be able to state specific information about safety precautions necessary to maintain proper pacemaker function. The Client Requiring Cardiopulmonary Resuscitation

20. A rescuer is called to a neighbors home after a 56-year-old man collapses. After quickly assessing the victim, the rescuer determines that the victim is unresponsive. To determine unresponsiveness, the rescuer can: a. Call the victims name and gently shake the victim b. Perform the chin-tilt to open the victims airway c. Feel for any air movement from the victims nose or mouth d. Watch the victims chest for respirations Ans: A calling the victims name and gently shaking the victim is used to establish unresponsiveness. The headtilt, chin-lift maneuver is used to open the victims airway. Feeling for any air movement from the victims nose or mouth indicates whether the victim is breathing on his own. The rescuer can watch the victims chest for respirations to see if the victim is breathing. 21. Proper hand placement for chest compressions during cardiopulmonary resuscitation (CPR) is essential to reduce the risk of which of the following complications? a. Gastrointestinal bleeding b. Myocardial infraction c. Emesis d. Rib fracture Ans: D proper hand placement during chest compressions is essential to reduce the risk of rib fractures, which may lead to pneumothorax and other internal injuries. Gastrointestinal bleeding and myocardial infarction are generally not considered complications of CPR. Although the victim may vomit during CPR, this is not associated with poor hand placement, but rather with distention of the stomach. 22. The American Heart Association guidelines urge greater availability of automated external defibrillators (AEDs) and people trained to use them. AEDs are used in cardiac arrest situations for: a. Early defibrillation in cases of atrial fibrillation b. Cardioversion in cases of atrial fibrillation c. Pacemaker placement d. Early defibrillation in cases of ventricular fibrillation Ans: D AEDs are used for early defibrillation in cases of ventricular fibrillation. The American Heart Association places major emphasis on early defibrillation for ventricular fibrillation and use of the AED as a toll to increase sudden cardiac arrest survival rates. 23. A client who has been given CPR is transported by ambulance to the hospitals emergency department, where the admitting nurse quickly assesses the clients condition. Of the following observations, the one most often recommended for determining the effectiveness of CPR is noting whether the: a. Pulse rate is normal b. Pupils are reacting to light c. Mucous membranes are pink d. Systolic blood pressure is at least 80 mmHg Ans: B Pupillary reaction is the best indication of whether oxygenated blood has been reaching the clients brain. Pupils that remain widely dilated and do not react to light probably indicate that serious brain damage has occurred. The pulse rate may be normal, mucous membranes may still be pink, and systolic blood pressure may be 80 mmHg or higher, and serious brain damage may still have occurred. 24. The client receives epinephrine during resuscitation in the emergency department. This drug is administered primarily because of its ability to: a. Dilate bronchioles b. Constrict arterioles c. Free glycogen from the liver d. Enhance myocardial contractility Ans: D. Epinephrine is administered during resuscitation efforts primarily for its ability to improve cardiac activity. Epinephrine has great affinity for adrenergic receptors in cardiac tissue and acts to strengthen and speed the heart rate as well as to increase impulses conduction from atria to ventricles. Epinephrine dilates bronchioles and constricts arterioles, but these are not the primary reasons for administering it during resuscitation. Epinephrine is not associated with freeing glycogen from the liver.

25. The rescuer understands that the compression-to-ventilation ratio for one-rescuer adult CPR is: a. 5:1 b. 15:1 c. 5:2 d. 15:2 Ans: D With one-rescuer CPR, the compression to ventilation ratio is 15:2. 26. During CPR, the xiphoid process at the lower end of the sternum should not be compressed when performing cardiac compressions. Which of the following organs would be most likely at risk for laceration by forceful compressions over the xiphoid process? a. Lung b. Liver c. Stomach d. Diaphragm Ans: B Because of its location near the xiphoid process, the liver is the organ most easily damaged from pressure exerted over the xiphoid process during CPR. The pressure on the victims chest wall should be sufficient to compress the heart but not so great as to damage internal organs. Injury may result, however, even when CPR is performed properly. 27. When performing external chest compressions on an adult during CPR, the rescuer should depress the sternum. a. 0.5 to 1 inch b. 1 to 1.5 inches c. 1.5 to 2 inches d. 2 to 2.5 inches Ans: C an adults sternum must be depressed 1.5 to 2 inches with each compression to ensure adequate heart compression. 28. The American Heart Association guidelines for Basic Cardiac Life Support recommend that the rescuer after first establishing unresponsiveness, should: a. Perform CPR for 2 minutes on the adult victim then place a call for emergency assistance b. Place a call for emergency assistance immediately c. Begin rescue breathing for the victim d. Begin CPR on the adult victim and wait until help comes on the scene Ans: B the American Heart Association guidelines for Basic Cardiac Life Support now recommends that the rescuer call for emergency assistance immediately after establishing unresponsiveness in the adult victim. A call for emergency assistance takes places precedence over initiating CPR in the adult victim, in an effort to get emergency personnel and an AED to the scene. Early defibrillation and prompt bystander CPR have increased sudden cardiac arrest survival rates. 29. If the victims chest wall fails to rise with each inflammation when rescue breathing is administered during CPR, the most likely reason is that the: a. Airway is not opened properly b. Victim is beyond resuscitation c. Inflations are being given at too rapid a rate d. Rescuer is using inadequate force for cardiac compression Ans: A if the airway is not opened properly, it is impossible to inflate the lungs during CPR. A common signs of airway obstruction is failure of the victims chest wall to rise with each inflation. The victim should not be considered beyond resuscitation; rather the airway should be opened properly. Inflations may be being given too rapidly. However, this is not the usual cause of not being able to adequately ventilate the victim. If the rescuer is using inadequate force for cardiac compression, it should not interfere with how ventilations are delivered.

30. During rescue breathing in CPR, the victim with exhale by: a. Normal relaxation of the chest b. Gentle pressure of the rescuers hand on the upper chest c. The presence of cardiac compressions d. Turning the head to the side Ans: A the exhalation phase of ventilation is a passive activity that occurs during CPR as part of the normal relaxation of the victims chest. No action by the rescuer is necessary. Medical Surgical Nursing Pre-test 1
Thread Started on Aug 18, 2010, 1:19pm

1. If parents or legal guardians aren't available to give consent for treatment of a life-threatening situation in a minor child, which of the following statements is most accurate? a.)Consent may be obtained from a neighbor or close friend of the family. b.)Consent may not be needed in a life-threatening situation. c.)Consent must be in the form of a signed document; therefore, parents or guardians must be contacted. d.)Consent may be given by the family physician. B. RATIONALE: In emergencies, including danger to life or possibility of permanent injury, consent may be implied, according to the law. Parents have full responsibility for the minor child and are required to give informed consent whenever possible. Verbal consent may be obtained. 2. You're admitting a 15-month-old boy who has bilateral otitis media and bacterial meningitis. Which room arrangements would be best for this client? a.)In isolation off a side hallway b.)A private room near the nurses' station c.)A room with another child who also has meningitis d.)A room with two toddlers who have croup B. RATIONALE: With meningitis, the child should be isolated for the first day but be close to where he can be observed frequently. In isolation off a side hallway is too far away for frequent observation. Putting the client in a room with another child who has meningitis or with two toddlers who have croup present an infectious hazard to the other children. 3. Which of the following points should a team leader consider when delegating work to team members in order to conserve time? a.)Assign unfinished work to other team members. b.)Explain to each team member what needs to be done. c.)Relinquish responsibility for the outcome of the work. d.)Assign each team member the responsibility to obtain dietary trays. B. RATIONALE: When all team members know what needs to be done, they can work together on the most efficient plan for accomplishing necessary tasks. Delegation can be flexible, ranging from telling a staff member exactly what needs to be done and how to do it to allowing team members some freedom to decide how best to carry out the tasks. Assigning unfinished work to other team members and assigning each team member the responsibility to obtain dietary trays don't allow for input from team members. It's the team leader's job to maintain responsibility for the outcome of a task.

4. The nurse is caring for a client admitted to the emergency department after a motor vehicle accident. Under the law, the nurse must obtain informed consent before treatment unless: a.)the client is mentally ill. b.)the client refuses to give informed consent. c.)the client is in an emergency situation. d.)the client asks the nurse to give substituted consent. C. RATIONALE: The law doesn't require informed consent in an emergency situation when the client is unable to give consent and no next of kin is present. A mentally competent client may refuse or revoke consent at any time. Even though a client who is declared mentally incompetent can't give informed consent, mental illness doesn't by itself indicate that the client is incompetent to give informed consent. Although the nurse may act as a client advocate, the nurse can never give substituted consent. 5. The nurse is assigned to care for an elderly client who is confused and repeatedly attempts to climb out of bed. The nurse asks the client to lie quietly and leaves her unsupervised to take a quick break. While the nurse is away, the client falls out of bed. She sustains no injuries from the fall. Initially, the nurse should treat this occurrence as: a.)a quality improvement issue. b.)an ethical dilemma. c.)an informed consent problem. d.)a risk-management incident. D. RATIONALE: The nurse should treat this episode as a risk-management incident; her immediate responsibility is to fill out an incident report and notify the risk manager. Quality improvement and ethics aren't the nurse's initial concerns. The facility may choose to look at these types of problems and make changes to deliver a higher standard of care institutionally. Informed consent isn't a relevant issue in this incident. 6. The nurse receives an assignment to provide care to 10 clients. Two of them have had kidney transplantation surgery within the last 36 hours. The nurse feels overwhelmed with the number of clients. In addition, the nurse has never cared for a client who has undergone recent transplantation surgery. What's the appropriate action for the nurse to take? a.)Speak to the manager and document in writing all concerns related to the assignment. b.)Refuse the assignment. c.)Ignore the assignment and leave the unit. d.)Trade assignments with another nurse. A. RATIONALE: When a nurse feels incapable of performing an assignment safely, the appropriate action is to speak to the manager or nurse in charge. The nurse should also document the concerns in writing and ask that the assignment be changed. In the event that the manager chooses to leave the assignment as given, the nurse should accept the assignment. The nurse should never abandon the assigned clients by leaving the workplace or asking another nurse to care for them. The nurse may, however, refuse to perform a task outside the scope of practice. 7. The nurse works with a colleague who consistently fails to use standard precautions or wear gloves when caring for clients. The nurse calls the colleague's attention to these oversights. The colleague tells the nurse that standard precautions and gloves aren't necessary unless the client is known to have tested positive for the human immunodeficiency virus. What's the most appropriate action for the nurse to take? a.)Ignore it because it isn't directly the nurse's problem b.)Document the problem in writing for the manager. c.)Talk to other staff members to ascertain their practices. d.)Instruct the clients to remind this colleague to wear gloves. B. RATIONALE: The nurse has spoken to her colleague under the appropriate circumstances and the behavior hasn't changed. Therefore, the appropriate action is to bring the problem to the manager's attention. It's unproductive to talk with other staff members about the situation because they don't have the authority to bring the colleague's practice into compliance. The nurse should never point out to a client that another staff member's practice isn't meeting standards.

8. An adult client is diagnosed with acquired immunodeficiency syndrome. The nurse who is caring for the client is also his friend. The nurse tells the client's parents about the diagnosis; after all, they know their son is the nurse's friend. Several weeks later, the nurse receives a letter from the client's attorney stating that the nurse has committed an intentional tort. Which intentional tort has this nurse committed? a.)Fraud b.)Defamation of character c.)Assault and battery d.)Breach of confidentiality D. RATIONALE: A nurse shouldn't disclose confidential information about a client to a third party who has no legal right to know; doing so is a breach of confidentiality. Defamation of character is injuring someone's reputation through false and malicious statements. Assault and battery occurs when the nurse forces a client to submit to treatment against the client's will. A nurse commits fraud when she misleads a client to conceal a mistake she made during treatment. 9. A nurse accidentally administers 40 mg of propranolol (Inderal) to a client instead of 10 mg. Although the client exhibits no adverse reactions to the larger dose, the nurse should: a.)call the facility's attorney. b.)inform the client's family. c.)complete an incident report. d.)do nothing because the client's condition is stable. C. RATIONALE: The nurse should file an incident report. Incident reports highlight areas of potential liability. It's then the risk manager's responsibility to notify the facility's attorney if the incident is believed to be serious. The risk manager, in consultation with the physician and facility administrator, will decide who should inform the family of the error. The quality assurance coordinator may choose to use such incidents when trying to improve the quality of care received by clients in a particular facility. Taking no action isn't an acceptable option. 10. The nurse is assigned to care for a postoperative client who has diabetes mellitus. During the assessment interview, the client reports that he's impotent and says that he's concerned about its effect on his marriage. In planning this client's care, the most appropriate intervention would be to: a.)encourage the client to ask questions about personal sexuality. b.)provide time for privacy. c.)provide support for the spouse or significant other. d.)suggest referral to a sex counselor or other appropriate professional. D. RATIONALE: The nurse should refer this client to a sex counselor or other professional. Making appropriate referrals is a valid part of planning the client's care. The nurse doesn't normally provide sex counseling. 11. The nurse is assigned to care for eight clients. Two nonprofessionals are assigned to work with the nurse. Which statement is valid in this situation? a.)The nurse may assign the two nonprofessionals to work independently with a client assignment. b.)The nurse is responsible to supervise assistive personnel. c.)Nonprofessionals aren't responsible for their own actions. d.)Nonprofessionals don't require training before they work with clients. B. RATIONALE: Assistive personnel may not be assigned to care for clients without the supervision of a professional nurse. It's essential that assistive personnel understand that they're responsible for their own actions. Assistive personnel must be adequately trained to perform all tasks they're assigned to perform.

12. Each state has guidelines that regulate the different levels of nursing & licensed practical or vocational nurse, registered nurse, or advanced practice nurse. Legal guidelines outlining the scope of practice for nurses are known as: a.)consent to treatment. b.)client's bill of rights. c.)nurse practice acts. d.)licensure requirements. C. RATIONALE: Each state has a nurse practice act that defines the scope of nursing practice within the state. Consent to treatment refers to informed consent for a treatment or procedure. The client's bill of rights defines the rights of clients. Licensure requirements are constructed by the state board of nursing to set standards for receiving a nursing license. 13. A client is dissatisfied with his hospitalization. He decides to leave against medical advice and refuses to sign the paperwork. The nurse's next course of action is to: a.)detain him until he signs the paperwork. b.)detain him until his physician arrives. c.)call security for assistance. d.)let him leave. D. RATIONALE: The nurse is obligated to let him leave. Detaining him in any form is a violation of the patient's bill of rights. 14. A nurse needs assistance transferring an elderly, confused client to bed. The nurse leaves the client to find someone to assist her with the transfer. While the nurse is gone, the client falls and hurts herself. The nurse is at fault because she hasn't: a.)properly educated this client about safety measures. b.)restrained the client. c.)documented that she left the client. d.)arranged for continual care of the client. D. RATIONALE: By leaving the client, the nurse is at fault for abandonment. The better course of action is to turn on the call bell or elicit help on the way to the client's room. Educating the client about safety measures doesn't alleviate the nurse from responsibility for ensuring the client's safety. The nurse can't restrain the client without a physician's order and restraints won't ensure the client's safety. Documenting that she left the client doesn't excuse the nurse from her responsibility for ensuring the client's safety. 15. When prioritizing a client's care plan based on Maslow's hierarchy of needs, the nurse's first priority would be: a.)allowing the family to see a newly admitted client. b.)ambulating the client in the hallway. c.)administering pain medication. d.)placing wrist restraints on the client. C. RATIONALE: In Maslow's hierarchy of needs, pain relief is on the first layer. Activity is on the second layer. Safety is on the third layer. Love and belonging are on the fourth layer.

16. When developing a therapeutic relationship with a client, the nurse should begin preparing the client for termination of the relationship: a.)at discharge. b.)during the first meeting. c.)at the midpoint of the relationship. d.)when the client demonstrates the ability to function independently. B. RATIONALE: When initiating a therapeutic relationship with a client, preparation for termination of the relationship should begin during the first meeting. For example, the nurse should introduce herself to the client and tell him exactly when she'll be involved in his care. This sets the boundaries of the relationship. In the middle and at discharge of care, the relationship may be too involved to end abruptly without warning. The client's ability to function independently isn't the deciding factor in preparing the client for the termination of the therapeutic relationship. 17. To be effective, a clinical nurse-manager in a managed care environment must: a.)expect all staff to accept change. b.)go along with a proposed change. c.)be a catalyst for change. d.)document staff nurses' reactions to change. C. RATIONALE: The clinical nurse-manager is responsible for making things happen, not just letting things happen. She must be more than a role model who goes along with change & she must also encourage change and support staff during change. Documentation of the nurses' reactions to change can be threatening and serves no purpose in helping change to occur. 18. In community-based nursing, primary responsibility for decisions related to health care belongs to the: a.)nurse. b.)client. c.)health care team. d.)physician. B. RATIONALE: The client is primarily responsible for health care decisions in community-based nursing. The nurse assists with monitoring of health treatment and teaching and intervenes only as needed after assessing the client's ability to follow a regimen. The health care team collaborates on decisions related to treatment. The physician dictates medical orders related to treatment and medication. 19. A client became seriously ill after a nurse gave him the wrong medication. After his recovery, he files a lawsuit. Who is most likely to be held liable? a.)No one because it was an accident b.)The hospital c.)The nurse d.)The nurse and the hospital D. RATIONALE: Nurses are always responsible for their actions. The hospital is liable for negligent conduct of its employees within the scope of employment. Consequently, both the nurse and the hospital are liable. Although the mistake wasn't intentional, standard procedure wasn't followed.

20. The nurse is providing care for a client who underwent mitral valve replacement. The best example of a measurable client outcome goal is to: a.)change his own dressing. b.)walk in the hallway. c.)walk from his room to the end of the hall and back before discharge. d.)eat a special diet. C. RATIONALE: Walking from his room to the end of the hall and back before discharge is a specific, measurable, attainable, timed goal. It's also a client-oriented outcome goal. Having the client change his own dressing is incomplete and not as significant. Just walking in the hall isn't measurable. The need for a special diet isn't evident in this case. 21. A client with end-stage liver cancer tells the nurse he doesn't want extraordinary measures used to prolong his life. He asks what he must do to make these wishes known and legally binding. How should the nurse respond to the client? a.)Tell him that it's a legal question beyond the scope of nursing practice. b.)Give him a copy of the client's bill of rights. c.)Provide information on active euthanasia. d.)Discuss documenting his wishes in an advance directive. D. RATIONALE: Advance directives give a competent client control over his situation and a legal forum in which to express his wishes about his care. Discussion of advance directives isn't outside the scope of nursing practice. The client's bill of rights involves multiple client rights and doesn't provide detailed information about advance directives. Active euthanasia is illegal. 22. While admitting a client with pneumonia, the nurse notes multiple bruises in various stages of healing. The client has Alzheimer's disease and a history of multiple fractures. Legally, the most important action for the nurse to take is to: a.)document findings thoroughly. b.)question the client about the bruising. c.)inform appropriate local authorities. d.)tell the client's physician. C. RATIONALE: This client may be experiencing elder abuse based on her history and symptoms. Authorities to be notified may include local social service or law enforcement agencies. The nurse should also document findings and include illustrations to support the assessment. The client with Alzheimer's disease may not be able to accurately inform the nurse about what happened. Reporting findings to the physician may not be sufficient for fulfilling the nurse's legal responsibility. 23. The nurse is providing care for a client with multiple myeloma, a disorder characterized by episodes of remissions and exacerbations. Which resource can best help the client adapt to the disease? a.)The client's family b.)Pastoral care c.)Support group .)Hospice care C. RATIONALE: Support groups consist of clients with the same diagnoses who share experiences of the disease with each other. Sharing experiences helps the client understand disease-related problems and gives him a forum in which he can vent his feelings, which are usually similar to those of the group. The client's family and clergy, although supportive, can't share similar disease experiences. Hospice care is usually implemented late in the disease, at the end of life.

24. A client with brain cancer is deteriorating and the prognosis is poor. The client meets brain-death criteria. Which nursing intervention is most appropriate at this time? a.)Approach the client's family about organ donation. b.)Make the decision to withdraw life support. c.)Sedate the client. d.)Talk to the staff about their feelings. A. RATIONALE: The most appropriate nursing intervention is to discuss organ donation with the family. The decision to withdraw life isn't within a nurse's scope of practice. Because the client is brain-dead, he doesn't need sedation. Although talking to the staff is a viable strategy for staff decompression, it isn't the first action to take. 25. A client is scheduled to have a descending colostomy. He's very anxious and has many questions concerning the surgical procedure, care of a stoma, and lifestyle changes. It would be most appropriate for the nurse to make a referral to which member of the health care team? a.)Social worker b.)Registered dietitian c.)Occupational therapist d.)Enterostomal nurse therapist D. RATIONALE: An enterostomal nurse therapist is a registered nurse who has received advanced education in an accredited program to care for clients with stomas. The enterostomal nurse therapist can assist with selection of an appropriate stoma site, teach about stoma care, and provide emotional support. Social workers provide counseling and emotional support, but they can't provide preoperative and postoperative teaching. A registered dietitian can review any dietary changes and help the client with meal planning. The occupational therapist can assist a client with regaining independence with activities of daily living. 26. A 92-year-old client with prostate cancer and multiple metastases is in respiratory distress and is admitted to a medical unit from a skilled nursing facility. His advance directive states that he doesn't want to be placed on a ventilator or receive cardiopulmonary resuscitation. Based on the client's advance directive, which intervention should the nursing care plan include? a.)Check on the client once per shift. b.)Provide mouth and skin care only if the family requests it. c.)Turn the client only if he's uncomfortable. d.)Provide emotional support and pain relief. D. RATIONALE: When advance directives state that a client doesn't want life-prolonging interventions, nursing care focuses on providing emotional and spiritual support and comfort measures. The client still needs to be checked regularly. The client and family shouldn't feel as if they've been abandoned. Providing mouth and skin care makes the client more comfortable. Turning the client provides comfort and prevents potentially painful complications such as pressure ulcers. 27. The registered nurse has an unlicensed assistant working with her for the shift. When delegating tasks, the registered nurse understands that the unlicensed assistant: a.)interprets clinical data. b.)collects clinical data. c.)is trained in the nursing process. d.)can function independently. B. RATIONALE: Unlicensed personnel make observations, collect clinical data, and report findings to the nurse. The registered nurse has learned critical thinking skills and is able to interpret the clinical findings. Unlicensed assistants are trained to perform skills & they don't learn the nursing process. Unlicensed assistants don't function independently & they're assigned tasks by a registered nurse who retains overall responsibility for the client. Other nursing responsibilities when delegating tasks to unlicensed assistants include knowing the institutions policies regarding delegation, knowing the assistant's training, knowing the client's needs, receiving frequent updates from the assistant, asking specific questions, and making frequent rounds of clients.

28. A nurse on a medical-surgical floor is making assignments for an 8-hour shift. Which of the following considerations has the highest priority? a.)Complexity of care required b.)Age of the clients c.)Skills of the assigned personnel d.)The number of clients C. RATIONALE: The nurse is legally responsible for assigning personnel according to skill level. All of the other factors are important but don't take priority. 29. The nurse is caring for a homeless client with active tuberculosis. The client is almost ready for discharge; however, the nurse is concerned about the client's ability to follow the medical regimen. Which intervention will best ensure that the client complies with treatment? a.)Referring the client to a social worker for discharge planning b.)Providing individualized client education c.)Having the client attend a formal education session d.)Attempting to contact a member of the client's family to provide assistance A. RATIONALE: Referring the client to a health care professional with knowledge of community resources is the best intervention to ensure compliance in a homeless client. Educating the client about his condition may help, but basic needs for shelter, food, and clothing must be met first. Providing formal education and attempting to contact family members are inappropriate when seeking to help a homeless client. 30. The nurse is following a critical pathway to help a client who underwent hip replacement surgery meet specific objectives. What's a critical pathway? a.)A nursing care plan that helps the nurse to decide which intervention to perform first b.)A multidisciplinary care plan that helps the nurse to use a variety of critical interventions c.)A standardized care plan that lists basic interventions for the nurse to use with every client d.)A clinical management tool that organizes the major interventions for a multidisciplinary health care team D. RATIONALE: Critical pathways are management tools developed for particular types of cases or conditions. They set forth expectations for interventions, outcomes, and client progression. Elements of the nursing care plan are commonly folded into the critical pathway. The descriptions of standardized and multidisciplinary plans of care don't adequately describe the critical pathway. Because the critical pathway is standardized and multidisciplinary, the nurse may need to develop a separate care plan to document nursing diagnoses for an individual client.

1. A 60-year-old male client comes into the emergency department with complaints of crushing substernal chest pain that radiates to his shoulder and left arm. The admitting diagnosis is acute myocardial infraction (MI). Immediate admission orders include oxygen by nasal cannula at 4 L/minute, blood work, a chest radiograph, a 12lead electrocardiogram (ECG), and 2 mg of morphine sulfate given intravenously. The nurse should first: a. Administer the morphine b. Obtain a 12-lead ECG c. Obtain the blood work d. Order the chest radiograph Ans: A although obtaining the ECG, chest radiograph, and blood work are all important, the nurses priority action should be to relieve the crushing chest pain. Therefore, administering morphine sulfate is priority action. 2. When administering a thrombolytic drug to the client experiencing an MI, the nurse explains to him that the purpose of the drug is to: a. Help keep him well hydrated b. Dissolve clots that he may have c. Prevent kidney failure d. Treat potential cardiac dysrhythmias Ans: B thrombolytic drugs are administered within the first 6 hours after of myocardial infarction to lyse clots and reduce the extent of myocardial damage. 3. If the client who has admitted for MI develops cardiogenic shock, which characteristic signs should the nurse expect to observe? a. Oliguria b. Bradycardia c. Elevated blood pressure d. Fever Ans: A oliguria occurs during cardiogenic shock because there is reduced blood flow to the kidneys. Typically signs of cardiogenic shock include low blood pressure, rapid and weak pulse, decrease urine output, and signs of diminished blood flow to the brain, such as confusion and restlessness. Cardiogenic shock is a serious complication of MI, with a mortality rate approaching 90%. Fever is not a typical sign of cardiogenic shock. 4. The physician orders continuous intravenous nitroglycerin infusion for the client with MI. essential nursing action include which of the following? a. Obtaining an infusion pump for the medication b. Monitoring blood pressure every 4 hours c. Monitoring urine output hourly d. Obtaining serum potassium levels daily Ans: A intravenous nitroglycerin infusion requires an infusion pump for precise control of the medication. Blood pressure monitoring would be done with a continuous system, and more frequently than every 4 hours. Hourly urine outputs are not always required. Obtaining serum potassium levels is not associated with nitroglycerin infusion.

5. When teaching the client with MI, the nurse explains that the pain associated with MI is caused by: a. Left ventricular overload b. Impending circulatory collapse c. Extracellular electrolyte imbalances d. Insufficient oxygen reaching the heart muscle Ans: D an MI interferes with or blocks circulation to the heart muscle. Decreased blood supply to the heart muscle causes ischemia, or poor myocardial oxygenation. Diminished oxygenation or lack of oxygen to the cardiac muscle results in ischemic pain or angina. 6. Aspirin is administered to the client experiencing an MI because of its: a. Antipyretic action b. Antithrombotic action c. Antiplatelet action d. Analgesic action Ans: B aspirin does have antipyretic, antiplatelet, and analgesic actions, but the primary reason aspirin is administered to the client experiencing an MI is its antithrombotic action. In clinical trials, the antithrombotic action of aspirin has been thought to account for improved outcomes in clients with MI. 7. While caring for a client who has sustained an MI, the nurse notes eight PVCs in 1 minute on the cardiac monitor. The client is receiving an intravenous infusion of 5% dextrose in water and oxygen at 2 L/minute. The nurses first course of action should be to: a. Increase the intravenous infusion rate b. Notify the physician promptly c. Increase the oxygen concentration d. Administer a prescribed analgesic Ans: B PVCs are often a precursor of life-threatening dysrhythmias, including ventricular tachycardia and ventricular fibrillation. An occasional PVC is not considered dangerous, but if PVCs occur at a rate greater than five or six per minute in the post-MI client, the physician should be notified immediately. More than six PVCs per minute is considered serious and usually calls for decreasing ventricular irritability by administering medications such as lidocaine hydrochloride. Increasing the intravenous infusion rate would not decrease the number of PVCs. Increasing the oxygen concentration should not be the nurses first course of action; rather, the nurse should notify the physician promptly. Administering a prescribed analgesic would not decrease ventricular irritability. 8. Which of the following is an expected outcome for a client on the second day of hospitalization after an MI? The client: a. Has minimal chest pain b. Can identify risk factors for MI c. Agrees to participate in a cardiac rehabilitation program d. Can perform personal self-care activities without pain Ans: D by day 2 of hospitalization after an MI, clients are expected to be able to perform personal care without chest pain. Day 2 of hospitalization may be too soon for clients to be able to identify risk factors for MI or to be able to agree to participate in a cardiac rehabilitation program.

9. When teaching a client about the expected outcomes after intravenous administration of furosemide, the nurse would include which outcome? a. Increased blood pressure b. Increased urine output c. Decreased pain d. Decreased PVCs Ans: B furosemide is a loop diuretic acts to increase urine output. Furosemide does not increase blood pressure, decrease pain, or decrease dysrhythmias. 10. After an MI, the hospitalized client is taught to move the legs about while resting in bed. This type of exercise is recommended primarily to help: a. Prepare the client for ambulation b. Promote urinary and intestinal elimination c. Prevent thrombophlebitis and blood clot formation d. Decrease the likelihood of decubitus ulcer formation Ans: C although this type of exercise may decrease the likelihood of heel decubitus ulcer form formation, it is taught to the MI client to prevent thrombophlebitis and blood clot formation. Movement of the lower extremities provides muscular action and aids venous return. As a result, the activity helps prevent stasis of blood, which predisposes the client to thrombophlebitis and blood clot formation. This type of exercise is not associated with promoting urinary and intestinal elimination. 11. Which of the following reflects the principle on which a clients diet will most likely be based during the acute phase of MI? a. Liquids as desired b. Small, easily digested meals c. Three regular meals per day d. Nothing by mouth Ans: B recommended dietary principles in the acute phase of MI include avoiding large meals because small, easily digested foods are better tolerated. Fluids are given according to the clients needs, and sodium restrictions may be prescribed, especially for clients with manifestations of heart failure. Cholesterol restrictions may be ordered as well. Clients are not prescribed diets of liquids only or restricted to nothing by mouth unless their condition is very unstable. 12. Of the following controllable risk factors for coronary artery disease (CAD) appears most closely linked to the development of the disease? a. Age b. Medication usage c. High cholesterol levels d. Gender Ans: C high cholesterol levels are considered a controllable risk factor for CAD and appear most clearly linked to the development of the disease. High cholesterol levels can be modified through diet, exercise, and medication. Age and gender are uncontrollable risk factors for CAD. Medication usage is not considered a risk factor for CAD.

13. Which of the following is an uncontrollable risk factor that has been linked to the development of CAD? a. Exercise b. Obesity c. Stress d. Heredity Ans: D heredity has been linked to CAD and is an uncontrollable risk factor. Exercise, obesity, and stress are controllable risk factor for CAD. 14. If a client displays risk factors for CAD such as smoking cigarettes, eating a diet high in saturated fat, or leading a sedentary lifestyle, technique of behavior modification may be used to help the client change behavior. The nurse can best reinforce new adaptive behaviors by: a. Explaining how the old behavior leads to poor health b. Withholding praise until the new behavior is well established c. Rewarding the client whenever the acceptable behavior is performed d. Instilling mild fear into the client to extinguish the behavior Ans: C a basic principle of behavior modification is that behavior that is learned and continued is behavior that has been rewarded. Other reinforcement techniques have not been found to be as effective as reward. 15. Alteplase recombinant. Or tissue plasminogen activator (t-PA), a thrombolytic enzyme, is administered during the first 6 hours after onset of MI to: a. Control chest pain b. Reduce coronary artery vasospasm c. Control the dysrhythmias associated with MI d. Revascularize the blocked coronary artery Ans: D the thrombolytic agent t-PA, administered intravenously, lyses the clot blocking the coronary artery. The drug is most effective when administered within the first 6 hours after onset. 16. After the administration of t-PA, the nurse understands that a nursing assessment priority is to: a. Observe the client for chest pain b. Monitor for fever c. Monitor the 12-lead ECG every 4 hours d. Monitor breath sounds Ans: A although monitoring the 12-lead ECG and monitoring breath sounds are important, observing the client for chest pain is the nursing assessment priority, because closure of the previously obstructed coronary artery may recur. Clients who receive t-PA frequently receive heparin to prevent closure of the artery after t-PA. Careful assessment for signs of bleeding and monitoring of partial thromboplastin time are essential to detect complications. Administration of t-PA should not cause fever.

17. When monitoring a client who is receiving t-PA, the nurse understands it is important to monitor vital signs and have resuscitation equipment available because reperfusion of the cardiac tissue can result in which of the following? a. Cardiac dysrhythmias b. Hypertension c. Seizure d. Hypothermia Ans: A cardiac dysrhythmias are commonly observed with administration of t-PA. Cardiac dysrhythmias associated with reperfusion of the cardiac tissue. Hypotension is commonly observed with administration of t-PA. Seizures and hypothermia are not generally associated with reperfusion of the cardiac tissue. 18. Contraindication to the administration of t-PA include which of the following? a. Age greater than 60 years b. History of cerebral hemorrhage c. History of heart failure d. Cigarette smoking Ans: B a past history of cerebral hemorrhage is a contraindication to administration of t-PA because the risk of hemorrhage may be further increased. Age greater than 60 years, history of heart failure, and cigarette smoking are not contraindications. 19. A client has driven himself into the emergency room. He is 50 years old, has a history of hypertension, and informs the nurse that his father died from a heart attack at 60 years of age. The client is presently complaining of indigestion. The nurse connects him to an ECG monitor and begins administering oxygen at 2 L/minute per nasal cannula. The nurses next action would be to: a. Call for the doctor b. Start an intravenous line c. Obtain a portable chest radiograph d. Draw blood for laboratory studies Ans: B advanced cardiac life support recommends that at least one or two intravenous lines be inserted in one or both of the antecubital spaces. Calling the physician, obtaining a portable chest radiograph, and drawing blood for the laboratory are important but secondary to starting the intravenous line. 20. Crackles heard on lung auscultation indicate which of the following? a. Cyanosis b. Bronchospasm c. Airway narrowing d. Fluid-filled alveoli Ans: D crackles are auscultated over fluid-filled alveoli. Crackles heard on lung auscultation do not have to be associated with cyanosis. Bronchospasm and airway narrowing generally are associated with wheezing sounds.

21. A 68-year-old female client on day 2 after hip surgery has no cardiac history but starts to complain of chest heaviness. The first nursing action should be to: a. Inquire about the onset, duration, severity, and precipitating factors of the heaviness b. Administer oxygen via nasal cannula c. Offer pain medication for the chest heaviness d. Inform the physician of the chest heaviness Ans: A further assessments is needed in this situation. It is premature to initiate other actions until further data have been gathered. Inquiring about the onset, duration, location, severity, and precipitating factors of the chest heaviness will provide pertinent information to convey to the physician. 22. The nurse receives emergency laboratory results for a client with chest pain and immediately informs the physician. An increased myoglobin level suggests which of the following? a. Cancer b. Hypertension c. Liver disease d. Myocardial damage Ans: D detection of myoglobin is one diagnostic tool to determine whether myocardial damage has occurred. Myoglobin is generally detected about 1 hour after a heart attack is experienced and peaks within 4 to 6 hours after physician. 23. An older, sedentary adult may not respond to emotional or physical stress as well as a younger individual because of: a. Left ventricular atrophy b. Irregular heart beats c. Peripheral vascular occlusion d. Pacemaker placement Ans: A in older adults who are less active and do not exercise the heart muscle, atrophy can result. Disuse or deconditioning can lead to abnormal changes in the myocardium of the older adult. As a result, under sudden able to respond to the increased demands on the myocardial muscle. Decreased cardiac output, cardiac hypertrophy, and heart failure are examples of the chronic conditions that may develop in response to inactivity, rather than in response to the aging process. Irregular heartbeats are generally not associated with an older sedentary adults lifestyle. Peripheral vascular occlusion of pacemaker placement should not affect response to stress. The Client With Heart Failure 24. A 69-year-old woman has a history of heart failure. She is admitted to the emergency department with heart failure complicated by pulmonary edema. On admission of this client, which of the following should be assessed first? a. Blood pressure b. Skin breakdown c. Serum potassium d. Urine output Ans: A it is a priority to assess the blood pressure first, because people with pulmonary edema typically experience severe hypertension that requires early intervention.

25. In which of the following should the nurse place a client with suspected heart failure? a. Semi-sitting (Low Fowlers position) b. Lying on the right side (Sims position) c. Sitting almost upright (High Fowlers position) d. Lying on the back with the head lowered (Trendelenburg position) Ans: C sitting almost upright in bed with the feet and legs resting on the mattress decreases venous return to the heart, thus reducing myocardial workload. Also, the sitting position allows maximum space for lung expansion. Low Fowlers position would be used if the client could not tolerate high Fowlers position for some reason. Lying on the right side would not be a good position for the client in heart failure. The client in heart failure would not tolerate the Trendelenburg position. 26. Which of the following would be a priority nursing diagnosis for the client with heart failure and pulmonary edema? a. Risk for infection related to line placements b. Impaired skin integrity related to pressure c. Activity intolerance related to imbalance between oxygen supply and demand d. Constipation related to immobility Ans: C activity intolerance is a primary problem for clients with heart failure and pulmonary edema. The decreased cardiac output associated with heart failure leads to reduced oxygen and fatigue. Clients frequently complain of dyspnea and fatigue. The client could be at risk for infection related to line placements or impaired skin integrity related to pressure. However, these are not the priority nursing diagnoses for the client with heart failure and pulmonary edema, nor is constipation related to immobility. 27. The major goal of therapy for a client with heart failure and pulmonary edema would be to: a. Increase cardiac output b. Improve respiratory edema c. Decrease peripheral edema d. Enhance comfort Ans: A increasing cardiac output is the main goal of therapy for the client with heart failure or pulmonary edema. Pulmonary edema is an acute medical emergency requiring immediate intervention. Respiratory status and comfort will be improved when cardiac output increases to an acceptable level. Peripheral edema is not typically associated with pulmonary edema. 28. Digoxin is administered intravenously to a client with heart failure, primarily because the drug acts to: a. Dilate coronary arteries b. Increase myocardial contractility c. Decrease cardiac dysrhythmias d. Decrease electrical conductivity in the heart Ans: B digoxin is cardiac glycoside with positive inotropic activity. This inotropic activity causes increased strength of myocardial contractions and thereby increases output of blood from the left ventricle. Digoxin does not dilate coronary arteries. Although digoxin can be used to treat dysrhythmias and does decrease the electrical conductivity of the myocardium, this is not the primary reason for its use in clients with heart failure and pulmonary edema.

29. Captopril, an antigiotensin-converting enzyme (ACE) inhibitor, may be administered to a client with heart failure because it acts as a: a. Vasopressor b. Volume expander c. Vasodilator d. Potassium-sparing diuretic Ans: C- ACE inhibitors have become the vasodilators of choice in the client with mild to severe congestive heart failure. Vasodilator drugs are the only class of drugs clearly shown to improve survival in overt heart failure. 30. Furosemide is administered intravenously to a client with heart failure. How soon after administration should the nurse begin to see evidence of the drugs desired effect? a. 5 to 10 minutes b. 30 to 60 minutes c. 2 to 4 hours d. 6 to 8 hours Ans: A after intravenous injection of furosemide, diuresis normally begins in about 5 minutes and reaches its peak within about 30 minutes. Medication effects last 2 to 4 hours. When furosemide is given intramuscularly or orally, drug action begins more slowly and lasts longer than when it is given intravenously. 31. The nurse teaches a client with heart failure to take oral Furosemide in the morning. The primary reason for this is to help: a. Prevent electrolyte imbalances b. Retard rapid drug absorption c. Excrete excessive fluids accumulated during the night d. Prevent sleep disturbances during the night Ans: D when diuretics are given early in the day, the client will void frequently during the daytime hours and will not need to void frequently during the night. Therefore, the clients sleep will not be disturbed. Taking furosemide in the morning has no effect on preventing electrolyte imbalances or retarding rapid drug absorption. The client should not accumulate excessive fluids throughout the night. 32. Clients with heart failure are prone to atrial fibrillation. During physical assessment, the nurse would suspect atrial fibrillation when palpation of the radial pulse reveals: a. Two regular beats followed by one irregular b. An irregular pulse rhythm c. Pulse rate below 60 bpm d. A weak, thready pulse Ans: B characteristics of atrial fibrillation include pulse rate greater than 100 bpm, totally irregular rhythm, and no definite P waves on the ECG. During assessment, the nurse is likely to note the irregular rate and should report it to the physician. A weak, thready pulse is characteristic of a client in shock.

33. When teaching the client about complications of atrial fibrillation, the nurse understands that the complications can be caused by: a. Stasis of blood in the atria b. Increased cardiac output c. Decreased pulse rate d. Elevated blood pressure Ans: A atrial fibrillation occurs when the sinoatrial node no longer functions as the hearts pacemaker and impulses are initiated at sites within the atria. Because conduction through the atria is disturbed, atrial contractions are reduced and stasis of blood in the atria occurs, predisposing to emboli. Some estimates predict that 30% of clients with atrial fibrillation develop emboli. Atrial fibrillation is not associated with increased cardiac output, elevated blood pressure, or decreased pulse rate; rather, it is associated with an increased pulse rate. 34. The nurse should teach the client that signs of digitalis toxicity include which of the following? a. Skin rash over the chest and back b. Increased appetite c. Visual disturbances such as seeing yellow spots d. Elevated blood pressure Ans: C colored vision and seeing yellow spots are symptoms of digitalis toxicity. Abdominal pain, anorexia, nausea, and vomiting are other common symptoms of digitalis toxicity. Additional signs of toxicity include dysrhythmias, such as atrial fibrillation or bradycardia. Skin rash, increased appetite, and elevated blood pressure are not associated with digitalis toxicity. 35. The nurse should be especially alert for signs and symptoms of digitalis toxicity if serum levels indicate that the client has a: a. Low sodium level b. High glucose level c. High calcium level d. Low potassium level Ans: D a low serum potassium level (hypokalemia) predisposes the client to digitalis toxicity. Because potassium inhibits cardiac excitability, a low serum potassium level would mean that the client would be prone to increased cardiac excitability. 36. Which of the following foods should the nurse teach a client with heart failure to avoid or limit when following a 2-g sodium diet? a. Apples b. Tomato juice c. Whole wheat bread d. Beef tenderloin Ans: B canned foods and juices, such as tomato juice, are typically high in sodium and should be avoided in a sodium-restricted diet, canned foods and juices in which sodium has been removed or limited are available. The client should be taught to read labels carefully. Apples and whole wheat breads are not high in sodium. Beef tenderloin would have less sodium than canned foods or tomato juice.

37. To help maintain a normal blood serum level of potassium, the client receiving a loop diuretic should be encouraged to eat such foods as bananas, orange juice, and, a. Spinach b. Skimmed milk c. Baked chicken d. Brown rice Ans: A foods rich in potassium include bananas, orange juice, and green leafy vegetables such as spinach. Honeydew melon, cantaloupe, and watermelons are also rich in potassium. Other good sources of potassium are grapefruit juice, nectarines, potatoes, dried prunes, raisins, and figs. Skimmed milk, baked chicken, and brown rice are not considered high in potassium. 38. The nurse finds the apical impulses below the fifth intercostals space. The nurse suspects a. Left atrial enlargement b. Left ventricular enlargement c. Right atrial enlargement d. Right ventricular enlargement Ans: B - a normal apical impulse is found over the apex of the heart and is typically located and auscultated in the left fifth intercostals space in the midclavicular line. An apical impulse located or auscultated below the fifth intercostals space or lateral to the midclavicular line may indicate left ventricular enlargement. 39. The nurse is admitting a 69-year old man to the clinical unit. The client has a history of left ventricular enlargement. During the assessment the nurse notes +3 pitting edema of the ankles bilaterally. The client does not have chest pain. The nurse observes that the client does have dyspnea at rest. The nurse infers that the client may have a. Arteriosclerosis b. Congestive heart failure c. Chronic bronchitis d. Acute myocardial infarction Ans: B peripheral edema is a symptom of congestive heart failure. Congestive heart failure results when the heart chronically pumps against increased resistance or is unable to contract forcefully to pump the blood out into the systemic circulation. As a result, the ventricles become overfilled and there is an accumulation of volume within the closed system. The clients symptoms do not indicate arteriosclerosis, chronic bronchitis, or acute MI. 40. The nurses discharge teaching plan for the client with congestive heart failure would stress the significance of which of the following? a. Maintaining a high-fiber diet b. Walking 2 miles every day c. Obtaining daily weights at the same time each day d. Remaining sedentary for most of the day Ans: C Congestive heart failure is a complex and chronic condition. Education should focus on health promotion and preventive care in the home environment. Signs and symptoms can be monitored by the client. Instructing the client to obtain daily weights at the same time each day is very important. The client should be told to call the physician if there has been a weight gain of 2 pounds or more. This may indicate fluid overload, and treatment can be prescribed early and on an outpatient basis, rather than waiting until the symptoms become life threatening.

Following a high-fiber diet id beneficial, but it is not relevant to the teaching needs of the client with congestive heart failure. Prescribing an exercise program for the client, such as walking 2 miles everyday, would not be appropriate at discharge. The clients exercise program would need to be planned in consultation with the physician and based on his history and the physical condition of the client. The client may require exercise tolerance testing before an exercise plan is laid out. Although the nurse does not pre-lifestyle should not be recommended. 41. A 70-year-old woman is scheduled to undergo mitral valve replacement for severe mitral stenosis and mitral regurgitation. Although the diagnosis was made during childhood, she did not have symptoms until 4 years ago. Recently, she noticed increased symptoms, despite daily doses of digoxin and furosemide. During the initial interview with the client, the nurse would most likely learn that the clients childhood health history included: a. Chicken pox b. Poliomyelitis c. Rheumatic fever d. Meningitis Ans: C Most clients with mitral stenosis have a history of rheumatic fever or bacterial endocarditis. Chicken pox, poliomyelitis, and meningitis are not associated with mitral stenosis. 42. A client experiences some initial signs of excitation after having an intravenous infusion of lidocaine hydrochloride started. The nurse would assess that the client is demonstrating a typical adverse reaction to lidocaine hydrochloride when the client complains of: a. Palpitations b. Tinnitus c. Urinary frequency d. Lethargy Ans: B Common adverse effects of lidocaine hydrochloride include dizziness, tinnitus, blurred vision, tremors, numbness and tingling of extremities, excessive perspiration, hypotension, convulsions, and finally coma. Cardiac effects include slowed conduction and cardiac arrest. Palpitations, urinary frequency, and lethargy are not considered typical adverse reactions to lidocaine hydrochloride. 43. A woman with severe mitral stenosis and mitral regurgitation has a pulmonary artery catheter inserted. The physician orders pulmonary capillary wedge pressures. The purpose of this is to help assess the: a. Degree of coronary artery stenosis b. Peripheral arterial pressure c. Pressure from fluid within the left ventricle d. Oxygen and carbon dioxide concentrations in the blood Ans: C the pulmonary artery pressures are used to assess the hearts ability to receive and pump blood. The pulmonary capillary wedge pressure reflects the left ventricular end-diastolic pressure and guides the physician in determining fluid management for the client. The degree of coronary artery stenosis is assessed during a cardiac catheterization. The peripheral arterial pressure is assessed with an arterial line. The oxygen and carbon dioxide concentrations in the arterial blood can be measured by an arterial blood gas determination.

44. Which of the following signs and symptoms would most likely be found in a client with mitral regurgitation? a. Exertional dyspnea b. Confusion c. Elevated creatine phosphokinase concentration d. Chest pain Ans: A weight gain due to fluid retention and worsening heart failure cause exertional dyspnea in clients with mitral regurgitation. The rise in left atrial pressure that accompanies mitral valve disease is transmitted backward to the pulmonary veins, capillaries, and arterioles and eventually to he right ventricle. Signs and symptoms of pulmonary and systemic venous congestion follow. Confusion, elevated creatine phosphokinase concentration, and chest pain are not typically associated with mitral regurgitation. 45. The nurse expects that a client with mitral stenosis would demonstrate symptoms associated with congestion in the: a. Aorta b. Right atrium c. Superior vena cava d. Pulmonary circulation Ans: D when mitral stenosis is present, the left atrium has difficulty emptying its contents into the left ventricle. Hence, because there is no valve to prevent backward flow into the pulmonary vein, the pulmonary circulation is under pressure. functioning of the aorta, right atrium, and superior vena cava is not immediately influenced by mitral stenosis. 46. Because a client has mitral stenosis and is a prospective valve recipient, the nurse preoperatively assesses the clients past compliance with medical regimens. Lack of compliance with which of the following regimens would pose the greatest health hazard to this client? a. Medication therapy b. Diet modification c. Activity restrictions d. Dental care Ans: A preoperatively, anticoagulants may be prescribed for the client with advanced valvular heart disease to prevent emboli. Postoperatively, all clients with mechanical valves and some clients with bioprostheses are maintained indefinitely on anticoagulant therapy. Adhering strictly to a dosage schedule and observing specific precautions are necessary to prevent hemorrhage or thromboembolism. Some clients are maintained on lifelong antibiotic prophylaxis to prevent recurrence of rheumatic fever. Episodic prophylaxis is required to prevent infective endocarditis after dental procedures or upper respiratory, gastrointestinal, or genitourinary tract surgery. Diet modification, activity restrictions, and dental care are important; however, they do not have as much significance postoperatively as medication therapy does.

47. In preparing the client and the family for a postoperative stay in the intensive care unit after open heart surgery, the nurse should explain that: a. The client will remain in the intensive care unit for 5 days b. The client will sleep most of the time while in the intensive care unit c. Noise and activity within the intensive care unit are minimal d. The client will receive medication to relieve pain Ans: D management of postoperative pain is priority for the client after surgery, including valve replacement surgery, according to the Agency for Health Care Policy and Research. The client and family should be informed that pain will be assessed by the nurse and medications will be given to relieve the pain. The client will stay in the intensive care unit as long as monitoring and intensive care are needed. Sensory deprivation and overload, high noise levels, and disrupted sleep and rest patterns are some environmental factors that affect recovery from valve replacement surgery. 48. A client who has undergone a mitral valve replacement experiences persistent bleeding from the surgical incision during the early postoperative period. Which of the following pharmaceutical agents should the nurse be prepared to administer to this client? a. Vitamin C b. Protamine sulfate c. Quinidine sulfate d. Warfarin sodium (Coumadin) Ans: B protamine sulfate is used to help combat persistent bleeding in a client who has had open heart surgery. Vitamin C and quinidine sulfate do not influence blood clotting. Warfarin sodium is an anticoagulant, as is heparin, and these two agents would tend to cause the client to bleed even more. 49. The most effective measure the nurse can use to prevent wound infection when changing a clients dressing after coronary artery bypass surgery is to: a. Observe careful handwashing procedures b. Cleanse the incisional area with an antiseptic c. Use prepackaged sterile dressings to cover the incision d. Place soiled dressings in a waterproof bag before disposing of them Ans: A many factors help prevent wound infections, including washing hands carefully, using the sterile prepackaged supplies and equipment, cleansing the incisional area well, and disposing of soiled dressings properly. However, most authorities say that the single most effective measure in preventing wound infections is to wash the hands carefully before and after changing dressings. Careful handwashing is also important in helping reduce other infections often acquired in hospitals, such as urinary tract and respiratory system infections.

50. For a client who excretes excessive amounts of calcium during the postoperative period after open surgery, which of the following measures should the nurse institute to help prevent complications associated with excessive calcium excretion? a. Ensure a liberal fluid intake b. Provide an alkaline-ash diet c. Prevent constipation d. Enrich the clients diet with dairy products Ans: A in an immobilized client, calcium leaves the bone and concentrates in the extracellular fluid. When a large amount of calcium passes through the kidneys, calcium can precipitate and form calculi. Nursing interventions that help prevent calculi include ensuring a liberal fluid intake (unless contraindicated). A diet rich in acid should be provided to keep the urine acidic, which increases the solubility of calcium. Preventing constipation is not associated with excessive calcium excretion. Limiting foods rich in calcium, such as dairy products, will help in preventing renal calculi. 51. The nurse teaches the client who is receiving warfarin sodium that: a. Partial thromboplastin time values determine the dosage of warfarin sodium b. Protamine sulfate is used to reverse the effects of warfarin sodium c. The international normalized ration (INR) is used to assess effectiveness d. Warfarin sodium will facilitate clotting of the blood Ans: C - the INR is the value used to assess effectiveness of the warfarin sodium therapy. INR is the prothrombin time ratio that would be obtained if the thromboplastin reagent from the World Health Organization was used for the plasma test. It is now the recommended method to monitor effectiveness of warfarin sodium. Generally, the INR for clients administered warfarin sodium should range from 2 to 3. In the past, prothrombin time was used to assess effectiveness of warfarin sodium and was maintained at 1.5 to 2.5 times the control value. Partial thromboplastin time is used to assess the effectiveness of heparin therapy. Fresh frozen plasma or vitamin K is used to reverse warfarin sodiums anticoagulant effect, whereas protamine sulfate reverses the effects of heparin. Warfarin sodium will help to prevent blood clots. 52. Good dental care is an important measure in reducing risk of endocarditis. A teaching plan to promote good dental care in a client with mitral stenosis should include demonstration of the proper use of: a. A manual toothbrush b. An electric toothbrush c. An irrigation device d. Dental floss Ans: A daily dental care and frequent checkups by a dentist who is informed about the clients condition are required to maintain good oral health. Use of an electric toothbrush, an irrigation device, or dental floss may cause gums to bleed and allow bacteria to enter mucous membranes and the bloodstream, increasing the risk of endocarditis.

53. Before a clients disease discharge after mitral valve replacement surgery, the nurse should evaluate the clients understanding of postsurgery activity restrictions. Which of the following should the client not engage in until after the 1-month-old postdischarge appointment with the surgeon? a. Showering b. Lifting anything heavier than 10 pounds c. A program of gradually progressive walking d. Light housework Ans: B most cardiac surgical clients have median sternotomy incisions, which take about 3 months to heal. Measures that promote healing include avoiding heavy lifting, performing muscle reconditioning exercises, and using caution when driving. Showering or bathing is allowed as long as the incision is well approximated with no open areas or drainage. Activities should be gradually resumed on discharge. 54. Three days after mitral valve surgery, a 45-year-old woman comments that she hears a clicking noise coming from her chest and her rather large chest incision. The nurses response should reflect the understanding that the client may be experiencing which of the following? a. Anxiety related to altered body image b. Anxiety related to altered health status c. Altered tissue perfusion d. Lack of knowledge regarding the postoperative course Ans: A verbalized concerns from the client may stem from her anxiety over the changes her body has gone through after open heart surgery. Although the client may experience anxiety related to her altered health status or may have a lack of knowledge regarding her postoperative course, she is pointing out the changes in her body image. The client is not concerned about altered tissue perfusion. The Client With Hypertension 55. An industrial health nurse at a large printing plant finds a male employees blood pressure to be elevated on two occasions 1 month apart and refers him to his provide physician. The employee is about 25 pounds overweight and has smoked a pack of cigarettes daily for more than 20 years. The clients physician prescribes atenolol for the hypertension. The nurse should instruct the client to: a. Avoid sudden discontinuation of the drug b. Monitor the blood pressure annually c. Follow a 2-g sodium diet d. Discontinue the medication if severe headaches develop Ans: A atenolol is -adrenergic antagonists indicated for management of hypertension. Sudden discontinuation of this drug is dangerous because it may exacerbate symptoms. The medication should not be discontinued without a doctors order. Blood pressure needs to be monitored more frequently than annually in a client who is newly diagnosed and treated for hypertension. Clients are not usually placed on a 2-g sodium diet for hypertension.

56. The nurse teaches her client, who has recently been diagnosed with hypertension, about his dietary restrictions: a low-calorie, low-fat, low-sodium diet. Which of the following menu selections would best meet the clients? a. Mixed green salad with blue cheese dressing, crackers, and cold cuts b. Ham sandwich on rye bread and an orange c. Baked chicken, an apple, and a slice of white bread d. Hot dogs, baked beans, and celery and carrot sticks Ans: C processed and cured meat products, such as cold cuts, ham, and hot dogs, are all high in both and fat and sodium and should be avoided on a low-calorie, low-fat, low-salt diet. Dietary restrictions of all types are complex and difficult to implement\ with clients who are basically asymptomatic. 57. A clients job involves working in a warm, dry room, frequently bending and crouching to check the underside of a high-speed press, and wearing eye guards. Given this information, the nurse should assess the client for which of the following? a. Muscle aches b. Thirst c. Lethargy d. Postural hypotension Ans: D possible dizziness from postural hypotension when rising a crouched or bent position increases the clients risk of being injured by the equipment. The nurse should assess the clients blood pressure in all three positions (lying, sitting, and standing) at all routine visits. The client may experience muscle aches, or thirst from working in a warm, dry room, but these are not as potentially dangerous as postural hypotension. The client should not be experiencing lethargy. 58. An exercise program is prescribed for the client with hypertension. Which intervention would be most likely to assist the client in maintaining an exercise program? a. Giving the client a written exercise program. b. Explaining the exercise program to the clients spouse. c. Reassuring the client that he or she can do the exercise program. d. Tailoring a program to the clients needs and abilities. Ans: D tailoring or individualizing a program to the clients lifestyle has been shown to be an effective strategy for changing health behaviors. Providing a written program, explaining the program to the clients spouse, and reassuring the client that he or she can do the program may be helpful but are not as likely to promote adherence as individualizing the program. 59. The client realizes the importance of quitting smoking, and the nurse develops a plan to help the client achieve this goal. Which of the following nursing interventions should be the initial step in this plan? a. Review the negative effects of smoking on the body. b. Discuss the effects of passive smoking on environmental pollution. c. Established the clients smoking pattern. d. Explain how smoking worsens high blood pressure. Ans: C - a plan to reduce or stop smoking begins with establishing the clients personal daily smoking pattern and activities associated with smoking. It is important that the client understands the associated health and environmental risk, but this knowledge has not been shown to help clients change their smoking behavior.

60. Essential Hypertension would be diagnosed in a 40-year-old man whose blood pressure readings were consistently at or above which of the following? a. 120/90 mmHg b. 130/85 mmHg c. 140/90 mmHg d. 160/80 mmHg Ans: C Heart Center of the Philippines standards define hypertension as a consistent systolic blood pressure level greater than 140 mmHg and a consistent diastolic blood pressure level greater than 90 mmHg.

Community Health Nursing Pre-test 1


Thread Started on Aug 18, 2010, 12:53pm

1. This STD is characterized by greenish yellow, frothy musty odorous vaginal discharge accompanied with vaginal itchiness and painful urination? a. Trichomoniasis b. Chancroid c. Moniliasis d. Chlamydia Ans. A. Trichomoniasis 2. DOTS is a comprehensive strategy to detect and cure TB. The primary element of DOTS is a. Health workers counsel and observe their patients swallow each anti-TB medication and monitor progress until cures b. Regular drug supply c. Political will in terms of manpower and funding d. Sputum microscopy services Ans. A. The main strategy of the NTP is Directly Observed Treatment, short course (DOTS). This is a comprehensive strategy to control TB which primary health services around the world are using in the detection and cure TB 3. What is the mode of transmission of Pulmonary Tuberculosis? a. Fecal-oral c. Airborne, droplet b. Direct contact d. Blood borne Ans. C. (Martinez, D. Intensive Final Coaching Primary Health Care, Legazpi City: A1 Review Center, 06, question 36) 4. Standard examination in detecting PTb: a. X-ray c. Ultrasound b. Sputum Exam d. Tuberculin Test Ans. B. 5. In Catergory II of the TB treatment regimen all are prescribed BUT one: a. Relapse case b. New pulmonary smear (+) cases c. Failure cases d. Other (smear +) Ans. B. Category I: Prescribed for a) new pulmonary smear (+) cases, b) new seriously ill pulmonary smear(-) cases with extensive parenchymal involvement and c.) new severely ill extrapulmonary TB cases; Category II: Prescribed for a.) failure cases, b) relapse cases; c) RAD (smear +), and OTHER (smear+); Category III Prescribed for a.) new smear (-) but with minimal pulmonary TB on radiography as confirmed by medical officer and b) new extrapulmonary TB (not serious). ( 6. Malaria can be prevented by the following EXCEPT: a. Avoiding outdoor night time activities b. Use of mosquito repellants c. Planting of herbal plants which can be mosquito repellants d. Spending leisure time in the forest Ans. D. Preventive activities against malaria include all but not limited to the following: 1. Avoid going outdoor between 9pm to 3 am , the peak biting hours of female anopheles mosquitoes 2. Prophylaxis: taking chloroquine every week starting one to two weeks before traveling 3. Using mosquito net or curtain treatment by soaking the mosquito net in insecticidal solution and allow to dry before using 4. House spraying of insecticide inside the house 5. Wearing clothes that cover arms and legs at night

7. During this stage the H fever is said to cause severe abdominal pain, vomiting and frequent bleeding from GI tract: a. Invasive Stage c. Convalescent Stage b. Toxic stage d. Febrile Stage Ans. B. Toxic Stage A. Invasive stage or febrile stage starts abruptly as high fever, abdominal pain and headache; later flushing which maybe accompanied by vomiting and epistaxis; B. Toxic or hemorrhagic stage is the time where severe abdominal pain, vomiting and frequent bleeding from the GI is observed; C. Convalescent or recovery stage most of the vital signs are stable.

8. What is the etiologic agent of Diptheria? a. Klebs-Loffler bacillus b. Bordet Gengou Bacillus c. Vibrio El Tor d. Filterable virus Ans. A. Klebs-Loffler bacillus

9. An example of this route of transmission is the dust particles with infectious agent and residue of evaporated droplets that remain suspended in the air: a. Vectorborne transmission c. Contact transmission b. Airborne transmission d. Droplet transmission Ans. Ans. B. Airborne transmission A. Vectorborne transmission examples are flies, ticks, mosquitoes and flies; B. Airborne transmission includes droplet nuclei, dust particles with infectious agent and organisms shed into the environment from the skin, hair or perineal area; C. Contact transmission includes direct contact, indirect contact and droplet contact; D. theres no such thing as droplet transmission only droplet contact.

10. This chain of infection allows the microorganisms to move from reservoir to host: a. Portal of Exit c. Portal of entry b. infectious agent d. susceptible host Ans. A. Portal of exit A. Some examples of movement from reservoir to host are excretions, secretions and skin droplets; B. The common site for the portal of entry is mucous membranes, GI tract and non-intact skin; C. Examples of infectious agents are bacteria and virus; D. susceptible host are the immunosuppressed patient, fatigued, elderly and other hospitalized patient.

11. Inflammatory processes provided by the WBC (leukocytes), whose main purpose is to limit the effect of harmful bacteria or injury by destroying or neutralizing the organism, and by limiting its spread throughout the body: a. First line of defense b. Second line of defense c. Third line of defense d. Fourth line of defense Ans. B. Second line of defense A. First line of defense include the skin and mucus membrane; B. Second line of defense involves leukocytes; C. Third line of defense include the antibodies

12. The immune response is considered to be: a. First line of defense b. Second line of defense c. Third line of defense d. Fourth line of defense Ans. C. Third line of defense A. First line of defense include the skin and mucus membrane; B. Second line of defense involves leukocytes; C. Third line of defense includes the antibodies.

13. Nurse Angelika believed that the complete destruction of all microorganisms, including the spores is: a. Disinfection c. Cleaning b. Sterilization d. all of these Ans. B. Sterilization 14. When asked about the risk of transmitting agents through large particles (5 micrometer or more) Crizelda refer this type of precaution to her clients as: a. Airborne precaution b. Contact precaution c. Standard precaution d. Droplet precaution Ans. D. Droplet precaution 15. It is an upper respiratory condition characterized by escalating fever (peaks 3-5 days), conjunctivitis and coryza: a. Measles b. German Measles c. chicken pox d. Varicella Ans. Measles 16. A day biting female mosquito that breeds in household or standing clean water: a. Female anopheles b. Male anopheles c. Aedes aegypti d. None of these Ans. C. Aedes aegypti 17. The following are the major causes of intestinal parasitic infections in the Philippines, EXCEPT: a. Trichuris trichiura c. Hookworm b. Ascaris lumbricoides d. none of these Ans. D. All of them are the three major causes of intestinal parasitic infections 18. What is the period of communicability of the epidemic parotitis? a. Begins before the glands are swollen and presumed to last as long as localized glandular swelling remains b. Begins after the glands are swollen especially when the localized glandular swelling remains c. Begins before the glands are swollen and after the glands are swollen even when the glandular swelling is not observed. d. except c Ans. A. Begins before the glands are swollen and presumed to last as long as localized glandular swelling remains 19. Which his TRUE regarding Filariasis? a. Microfilarie rate increases with age and then levels off during the acute stage b. Men have higher micronlariae rate than women c. During the chronic stage the disease develops 2-5 years from the onset of attack d. Hydrocoele is evident in men during the acute stage A. It is not during the acute stage but during the asymptomatic stage: B True that men have higher micronlariae rate than women; C. The disease develops from 10-15 years from the onset of attack; D. It is during the chronic stage when hydrocoele will be noted.

20. Which is FALSE about Tuberculosis? a. The most hazardous period for development of clinical disease is the first 6-12 months after infection b. The risk of developing the disease is highest in children under 3 years old. c. The degree of communicability depends on the virulence of the bacilli, adequacy of ventilation with no direct exposure of bacilli to the sun or UV light d. Susceptibility of the disease is markedly increased in those with HIV infection and other forms of immunosuppression Ans. C. All are true except C because the degree of communicability always involves exposure of the bacilli to the sun or UV light. 21. Which of the following tetanus immunization is given to a mother to prevent from a neonatal tetanus and provide 10 years protection for the mother? a. TT1 b.TT2 c. TT3 d. TT4 Ans. C. Schedule of TT immunization: VACCINATION SCHEDULE PROTECTION DURATION OF PROTECTION TT1 As early as possible during pregnancy 80% TT2 4 weeks after TT1 80% Infants protected from neonatal tetanus. Provides 3 year protection to the mother TT3 6 months after TT2 90% Infants protected from neonatal tetanus. Provides 5 year protection to the mother TT4 1 year after TT3 99% Infants protected from neonatal tetanus. Provides 10 year protection to the mother TT5 1 year after TT4 99% Infants protected from neonatal tetanus. Provides lifetime protection to the mother.

22. Which of the following vaccines is not recommended by IM injection? a. Measles vaccine c. Hepa B vaccine b. Tetanus toxoid d. DPT Ans. A. Measles vaccine 23. The following are vaccines which are less sensitive to heat: a. oral polio, DPT b. Measles, Hep B c. BCG, TT d. BCG, oral polio Ans. C. BCG and TT Most sensitive to heat: oral polio and measles; least sensitive to heat: DPT, Hep B, BCG, TT 24. OPV should be stored in a freezer with a temperature of: a. 2-8 degrees C b. -15 to -25 degrees C c. -2 to -8 degrees C d. none of these Ans. B. -15 to -25 degrees C 25. Hepa B is given at birth with 6 weeks interval from the 1st dose to 2nd dose, 8 weeks interval from the 2nd to 3rd dose. It is given intramuscularly in the: a. outer part of upper arm b. gluteus maximus c. vastus lateralis d. deltoid region Ans. C. vastus lateralis

26. It is the principle practiced in order to assure that all vaccines are utilized before its due date: a. First expiry and first out b. First used and first contained c. First expiry and last out d. First used and last contained Ans. A. First expiry and First out

27. When reconstituting the freeze dried BCG vaccine the nurse must always keep the diluents in the room temperature by sustaining with the BCG vaccine ampules in the clinic table: a. TRUE b. FALSE c. TRUE in Hep B and DPT d. except A Ans. A. FALSE The BCG vaccine must always be kept in cold temperature and sustaining it in the refrigerator or vaccine carrier. 28. What is the classification of disease when the level of management is urgent referral in hospital? a. mild b. moderate c. severe d. most severe Ans. C. Severe. 29. What is the color presentation to children whose level of management is home care. a. yellow b. red c. green d. pink Ans. C. Green. ( 30. The following BUT one is part of the danger signs that needs to be checked in assessing the child using the integrated case management process? a. vomits everything b. convulsions c. unable to drink/breastfeed d. difficulty breathing Ans. D. Difficulty of breathing is part of the main symptoms that needs to be assessed not the danger signs. The missing danger sign is abnormality sleepy or difficult to awaken.

Community Health Nursing Comprehesive


Thread Started on Aug 18, 2010, 1:01pm

1. The nurse learned that the health of individuals and communities are, to a large extent, affected by a combination of many factors. She understands that these are the determinants of health listed by the World Health Organization, EXCEPT: a. Greater support from families, friends and communities are linked to better health. b. Access and use of services that prevent and treat disease influence health. c. Research, development and implementation of innovative public health solutions is also part d. Customs and tradition, and the beliefs of the family and community is also part Ans. C. Is not part of the determinants of health rather it is one of the functions of public health A. Social support system is part of the determinants of health; B. Health services is also part of the determinants of health; C. Is not part of the determinants of health rather it is one of the functions of public health; D. Culture is part of the determinants of health. ( 2. The classic definition of public health is the science and the art of preventing disease, prolonging life, promoting health and efficiency through organizing community effort for the sanitation of the environment, control of CD, education of personal hygiene, organization of medical and nursing services for the early diagnostic and treatment of disease and the development of the social machinery to ensure everyone a standard of living adequate for the maintenance of health, so organize to enable every citizen to realize his birthright of health and longevity. This definition was coined form: a. WHO Expert Committee on Nursing c. Ruth Freeman b. Dr. C.E. Winslow d. Public Health Nurse Lilian Wald Ans. B. Dr. C.E. Winslow A. WHO defined public health as the art of applying science in the context of politics so as to reduce inequities in health while ensuring the best health for the greatest number; B. Public health defined by Dr. Winslow exactly as mentioned above; C. Freeman defined community health nursing not public health as a service rendered by a professional nurse with communities, groups, familiesfor the promotion of health, prevention of illness, care of the sick at home and rehabilitation; D. Lilian Wald defined public health nursing not public health as service to all the people. (Cuevas, Frances Prescilla. Public Health Nursing in the Philippines. 10th ed. Philippines: Publication Committee of NLPGN,Inc., 2007 p. 4-7) 3. The Health Care Delivery System involves two major players; the first is financed through a tax-based budgeting system and the other is largely market-oriented. They are known to be as? a. National and Local Government Agencies b. For-profit and Non-profit health providers c. Public sector and LGU d. Department of Health and Non-profit health providers Ans. D. Department of Health and Non-profit health providers (Public and Private Sector) A. The National and Local Government Agencies are part of the public sector; B. For-profit and Non-profit health providers are both part of the private sector; C. The public sector and LGU are the same that took part in the public sector. D. The department of health is a public sector while the Non-profit health provider is part of the private sector. The 2 major players in the Health Care Delivery System are the Public and Private sectors. (Cuevas, Frances Prescilla. Public Health Nursing in the Philippines. 10th ed. Philippines: Publication Committee of NLPGN,Inc., 2007 p. 19-20)

4. What is the vision of the Department of Health? a. FOURmula ONE for health b. Health Sector Reform Agenda c. Health for all in the Philippines

d. Guarantee equitable, sustainable and quality health for all Filipinos, especially the poor and shall lead the quest for excellence in health Ans. C. Health for all in the Philippines A. FOURmula One for health is the framework for implementation of Health Sector Reform Agenda (HSRA); B. Health Sector Reform Agenda is the goal of the DOH; C. Health for all in the Philippines is the vision of the Department of Health; D. This is the Mission of the DOH. (Cuevas, Frances Prescilla. Public Health Nursing in the Philippines. 10th ed. Philippines: Publication Committee of NLPGN,Inc., 2007 p. 25-26) 5. The framework for the implementation of health sector reform agenda of the department of health is: a. FOURmula ONE for health b. Health Sector Reform Agenda c. Health for all in the Philippines d. Guarantee equitable, sustainable and quality health for all Filipinos, especially the poor and shall lead the quest for excellence in health Ans. A. FOURmula ONE for health A. FOURmula One for health is the framework for implementation of Health Sector Reform Agenda (HSRA); B. Health Sector Reform Agenda is the goal of the DOH; C. Health for all in the Philippines is the vision of the Department of Health; D. Guarantee . is the Mission of the DOH. (Cuevas, Frances Prescilla. Public Health Nursing in the Philippines. 10th ed. Philippines: Publication Committee of NLPGN,Inc., 2007 p. 25-26) 6. The overriding goals of the Department of Health: a. FOURmula ONE for health b. Health Sector Reform Agenda c. Health for all in the Philippines d. Guarantee equitable, sustainable and quality health for all Filipinos, especially the poor and shall lead the quest for excellence in health Ans. B. Health Sector Reform Agenda A. FOURmula One for health is the framework for implementation of Health Sector Reform Agenda (HSRA); B. Health Sector Reform Agenda is the goal of the DOH; C. Health for all in the Philippines is the vision of the Department of Health; D. Guarantee . is the Mission of the DOH. (Cuevas, Frances Prescilla. Public Health Nursing in the Philippines. 10th ed. Philippines: Publication Committee of NLPGN,Inc., 2007 p. 25-26) 7. The goal of this element in FOURmula ONE is to ensure the quality and affordability of health goods and services. a. Health financing c. Health regulation b. Health service delivery d. Good governance Ans. C. Health regulation A. Health financings goal of this health reform area is to foster greater, better and sustained investments in health; B. Health service deliverys goal is to improve and ensure the accessibility and availability of the basic and essential health care in both public and private facilities and services; C. Health regulation ensure the quality and affordability of health goods and services; D. Good governance help enhance the health system performance at the national and local levels. Cuevas, Frances Prescilla. Public Health Nursing in the Philippines. 10th ed. Philippines: Publication Committee of NLPGN,Inc., 2007 p. 26) 8. The first international conference of Primary Health Care was held in Alma Ata, USSR on September 6-12, 1978 by WHO. The goal was adopted in the Philipppines on October 19, 1979. What is its underlying theme? a. Health in the Hands of the People by 2020 b. Health for All by the year 2000 c. Health for all in the Philippines d. Health Sector Reform Agenda Ans. A. Health in the Hands of the People by 2020 A. Health in the Hands of the People by 2020 is the underlying theme in the Philippines; B. Health for All by the year 2000 is the goal during the conference; C. Health for all in the Philippines is the vision of the Department of Health; D. Health Sector Reform Agenda is the goal of the DOH

9. The primary components of the PHC include: a. Immunization and Control Communicable Diseases b. Health Education c. Environmental Sanitation d. All of the above Ans. D. All of the above The elements or components of PHC include: Environmental Sanitation; Control of Communicable diseases; Immunizations; Health Education; Maternal and Child Health and Family Planning; Adequate Food and Proper Nutrition; Provision of Medical Care and Emergency Treatment; Treatment of Locally Endemic Diseases and Provision of Essential Drugs. 10. Mark is a trained community worker in the community. He is identified at what level of primary health care worker? a. BHWs b. Volunteer c. Intermediate level health care d. Trained Hilot Ans. A. There are two levels of primary health care workers in the community. They are: 1. Village or Barangay Health Workers who are the trained community health workers, health auxiliary volunteers, TBSs and trained healers and 2. Intermediate Level Health Workers such as the general medical practitioner, PHN, midwife, rural sanitary inspector and midwife. 11. 10. There are 4 cornerstones/pillars in PHC. Of the following which is NOT included? a. Government Funds b. Intra and intersectoral linkages c. Use of appropriate technology d. Active community participation Ans. A. Government Funds A. Government Funds is not part of the pillars of PHC; B. Intra and Intersecoral linkages is part of the 4 pillars; C. The use of appropriate technology is also part; D. Active community participation is also part of the PHC cornerstone. The missing pillar is Support Mechanism. 12. The intention of this level of prevention is to halt the disease or injury process and assist the person in obtaining an optimal status according to his present capabilities: a. Secondary Level b. Tertiary Level c. Primary Level d. Quaternary Level Ans. B. Tertiary Level A. Secondary level: Early diagnosis, detection, screening, case finding, and prompt treatment. Examples are Sputum exam, cataract screening, ORESOL , etc.; B. Tertiary level: Rehabilitation. Services provided after a disease or disability has occurred and the recovery process has begun. Ex. Community based rehabilitation program; C. Primary level: Health promotion and specific protection, seeks to prevent a disease or condition before the person gets sick. To encourage optimal health and to increase the persons resistance to illness examples are prenatal care; D. There is no such thing as quaternary level. 13. The PHN understands that this function is the most inherent function. Her practice as a nurse is based on the science and art of caring, in whatever setting she maybe or role she may presume. a. Collaborating and coordinating function b. Supervisory function c. Health promotion and Education function d. Nursing care function Ans. D. Nursing Care function A. Collaborating and coordinating function bring the PHN its activities systematically into proper relation or

harmony with each other; B. Supervisory function supervises the midwives and other auxiliary health workers in the catchment area. C. Health Promotion and Education function tells that her activities go beyond health teachings and health information campaign; D. Nursing care function is the inherent function, as such PHNs are expected to provide nursing care. 14. The nursing process is systematic, scientific, and dynamic, on going interpersonal process in which the nurse and the clients are viewed as system with each affecting the other and both being affected by the factors within the behavior. The following are part of the Planning Nursing Action, EXCEPT: a. Develop evaluation parameters b. Identify needed alterations c. Prioritize needs d. Construct action and Operational plan Ans. B. Identify needed alterations A. Developing evaluation parameter is part of planning nursing action; B. Identifying needed alterations is part of Evaluation of care and services rendered; C. Prioritize needs is in planning nursing action; D. Constructing action and Operational plan is also part of the planning nursing action phase. 15. Maita a student nurse when asked by her clinical instructor the basic principles of health education responded INCORRECTLY when she mentioned that health education is: a. a creative process b. achieved by doing c. recognized as a basic function of the people d. helping people attain their health through their efforts Ans. C. Recognize as a basic function of the people A. a creative process is part of the basic principles of health education as well as B. achieved by doing; C. is wrong because it should be the primary function of the public health workers not the people; D. is correct also because it emphasizes the own effort of the people. 16. The primary focus of community health nursing is health promotion. What is its ultimate goal? a. worth and dignity of man b. making the community health nurses as generalist c. raise the level of health of the citizenry d. provide quality nursing services to individuals, families and communities utilizing as basis the standards set community health nursing practice. Ans. C. raise the level of health of the citizenry A. worth and dignity of man is the philosophy of CHN; B. making the community health nurses as generalist in the nursing profession; C. raising the level of health of citizenry is the ultimate goal; D. Provide quality nursing. Is one of the objectives. 17. These are the following principles of Community health nursing, EXCEPT: a. In CHN, the family is the unit service b. The CHNurse is responsible for her own professional growth. c. The CHNurse utilizes the already-existing active organized groups in the community. d. Opportunities for continuing staff education programs for nurses are the CHNurses responsibility. Ans. D. Opportunities for continuing staff education programs for nurses are the CHNurses responsibility A,B and C are part of the principles of Community Health Nursing. 18. The primary group focus of community health nursing practice especially in terms of maintaining the peoples optimum level of functioning is: a. Health Promotion b. Rehabilitation c. Reduction of lifestyle diseases d. Prevention of communicable disease Ans. A. Health Promotion

Although letters b, c and d are also functions of community health nursing, health promotion is the primary group focus of community health nursing practice especially in terms of maintaining the peoples OLOF. 19. These are the steps in conducting home visit. Arrange in order of priority: I. Put the bag in a convenient place then proceed to perform the bag technique II. Record all important data, observation and care rendered III. State the purpose of the visit IV. Perform the nursing care needed and give health teachings V. Greet the patient and introduce yourself a. III,V,I,II, and V b. V,III,I,IV and II c. III,V,IV,I and II d. V,I,III,IV and II Ans. B. V,III,I,IV and II. Steps in conducting home visits: 1. Greet the patient and introduce self; 2. State the purpose of the visit; 3. Observe the patient and determine the health needs; 4. Put the bag in a convenient place then proceed to perform the bag technique; 5 Perform the nursing care needed and give health teachings; 6. Record all important data, observation and care rendered and 7. Make appointment for a return visit. 20. In the Philippine Health Care Delivery System, the secondary level of health service is provided by the: a. Regional Medical Center c. Provincial hospitals b. Puericulture Centers d. Rural Health Units Ans. C. Provincial hospitals The RHCDS provides three levels of health services: a. Primary Level: BHS, RHU, Community Hospitals, Lying in centers, puericulture centers, clinics of private practitioner b. Secondary Level: Emergency hospitals, District Hospitals, Provincials and City Hospitals c. Tertiary Level: Regional Hospitals, National Medical Centers and Training Hospitals 21. The devolution of health services to the local government unit was mandated by: a. R.A. 7160 b. E.O. 851 c. R.A. 6713 d. R.A. 8749 Ans. A. RA 7160 is the local autonomy code, which transferred local health services under the authority of the local governments. EO 851 is the law of the Reorganization of DOH. RA 6713 is the code of conduct and ethical standards of public officials and employees. AR 8749 is the Clean Air Act, Approved in 2000 and took effect in 2001. 22. Nurse Cara learned that bag technique is a tool by which the nurse, during her visit enabled her to perform a nursing procedure with ease and deftness, to save time and effort in rendering nursing care to clients. Which of the following equipments are carried separately? a. Spirit of ammonia, Zephiran solution and 70% alcoohol b. Sphygmomanometer, Thermometer and Stethoscope c. Sphygmomanometer only d. Stethoscope and sphygmomanometer Ans. D. Stethoscope and sphygmomanometer A. Spirit of ammonia et als. Are carried in the bag; B. Thermometer is carried in the bag; C. Lacking stethoscope; D. Both are carried separately 23. The basic qualification of a CH Nurse is: a. Masters of Public Health b. Bachelor of Science in Nursing b. Licensed Practical Nurse d. b only Ans. D. b only 24. A process of analyzing and determining the community health status is community diagnosis. These are the following demographic variables needed in the diagnosis, EXCEPT:

a. age and sex composition c. pattern of migration b. educational level d. total population Ans. B. Educational level A, C and D are all demographic variables; B. educational level is part of socio-economic and cultural variable.

25. The nurse gathered data that will be used in community diagnosis. She assessed the physical/geographical characteristics. In what factor does this belong? a. Socioeconomic and cultural variable b. Environmental Indicator c. Demographic data d. both B and D Ans. B. Environmental Indicator

26. What level of water source where a system with a source is observed, there is reservoir and piped distribution network up to the household. a. Level I b. Level II c. Level III d. Level IV Ans. C. Level III A. Level I- a protected well or developed spring with outlet but without distribution; B. Level II a system composed of source, a reservoir and piped distribution network and communal faucet located not more than 2 meter from the nearest household; C. there is piped distribution network up to the household; D. no Level IV 27. Water system and deep well construction require the approval of: a. Mayors permit b. Secretary of health c. Rural sanitary inspector d. City health engineer Ans. B. The certification of the potability of an existing water source is also issued by the secretary of the DOH. 28. Which of the following is level 1 approved type of toilet facility? a. Water sealed b. Flushed type c. Pit latrines d. Water carriage type Ans. C. Level 1: non water carriage such as pit latrines, reed odorless earth closet; Level II water carriage type such as water sealed and flushed type septic tank; Level III water carriage type connected to septic tanks and sewerage system. 29. Vital Statistics refers to the systematic study of vital events such as births, illnesses, marriages, divorce, separation and deaths. It is the measure of the risk of dying from the cause related pregnancy, child birth and puerperium. a. Infant Mortality Rate b. Maternal Mortality Rate c. Maternal Morbidity rate d. Specific Death Rate Ans. B. Maternal Mortality Rate 30. In vital statistics, this measures the frequency of occurrence of the phenomenon during a given period of time. a. Prevalence Rate b. Incidence Rate c. Specific Rate d. except c Ans. B. Incidence Rate A. Prevalence Rate measures the proportion of the population which exhibits particular disease at a particular time;

B. is the correct answer; C. Specific Rate the relationship is for a specific population class or group. It limits the occurrence of the event to the portion of the population definitely exposed to it; D. only Incidence Rate or letter B 31. This is the phase of COPAR where nurse do an ocular survey of short listed community. a. Pre-entry c. Organization Building b. Entry d. Sustenance and Strengthening Ans. A. Pre-entry

32. What part of the pre-entry phase where the nurse pay courtesy call to the community leaders, sensitize community leaders and conduct baseline survey? a. Community preparation selection c. Coordination b. Final preparation d. Community preparation survey Ans. B. Final Preparation 33. When the Core group is selected the phase of COPAR is said to be: a. Phase I c. Phase II b. Phase III d. Phase IV Ans. C. Phase II 34. These are ways of partnership in COPAR, EXCEPT: a. Networking c. Collaboration b. Competition d. Cooperation Ans. B. Competition 35. The fundamental unit of any society, composed of father, mother and children related by blood or marriage is the family. When the authority inside the family is 50-50 basis or there is sharing of decision. This is said to be: a. Matriarchal c. Egalitarian b. Patriarchal d. Equalitarian Ans. C. Egalitarian A. Matriarchal mother is the dominant figure of authority; B. Patriarchal father is the dominant figure of authority; C. Egalitarian 50-50 basis, sharing of decision making; D. There is no such thing as Equalitarian 36. When the health status of the family is measured by nutritional status weight, height, mid-upper arm circumference. It is called: a. Nutritional assessment c. Dietary History b. Arthropometric assessment d. Eating/feeding habits, practices Ans. B. Arthropometric assessment A. Nutritional assessment especially for vulnerable or at risk members; B. Arthropometric assessment is the correct answer; C. Dietary history specifying the quality and quantity of food/nutrient intake per day; 37. It is the term used for intermittent occurrence of disease in a few and unrelated cases within a given locality. a. Endemic occurrence c. Epidemic occurrence b. Sporadic occurrence d. Pandemic occurrence Ans. B. Sporadic occurrence 38. Mortality and morbidity are categorized as community health nursing problems on: a. health resources c. health-related b. health status d. health indicators Ans. B. Vital Statistics is the collection of data of significant events that occur over a period of time within a population. Morbidity and mortality are among of the significant data needed to determine the health status of the community. 39. What is the leading cause of mortality in the Philippines as of 2003 statistics? a. Malignant Neoplasm c. Heart Diseases

b. Vascular System Diseases d. Tuberculosis, all forms Ans. C. Heart Disease Mortality Statistics (2003) 1. Heart Diseases 2. Vascular System Diseases 3. Malignant Neoplasm 4. Accidents 5. Pneumonia 6. Tuberculosis, all forms 40. The nurse is well versed in environmental health and sanitation. She understands that the components and factors in the prevention of illness lie in the following, EXCEPT: a. environment b. disease agent c. food safety practices d. man Ans. C. Food safety practices is not part 41. What would you include in your health teaching in iodine deficiency? a. Eat sea foods and vegetables b. Eat beans c. Eat meats and vegetables d. Eat sweet potatoes Ans. A. Good source of iodine include sea foods and iodized salt( 42. A traditional plant used to lower the uric acid. a. Pansit pansitan b. Lagundi c. Bayabas d. Sambong Ans. A Pansit pansitan , also known as ulasimang bato (Peperonia pellucida) is a herbal medicine which is known to lower uric acid and beneficial in persons with rheumatism and gout. Its leaves 1 cups leaves to two glassful of water are boiled and the decoction is taken three times a day. Fresh leaves can also be eaten as a salad three times a day. 43. A traditional plant use to treat diarrhea: a. Lagundi b. Bayabas c. Pansit pansitan d. Sambong Ans. B. Bayabas or guava (Psidium guajava) is a fruit bearing tree. Its leaves are boiled for 15 minutes at low fire and decoction. It can be used for the following medicinal purposes: washing, diarrhea, mouth gargle and relief of tooth ache. 44. An alternative for mefenamic acid for toothache is: a. Tsaang gubat b. Lagundi c. Sambong d. Bawang Ans. D. Bawang or ajos is pounded and applied to the aching tooth. To lower cholesterol and for hypertension, 2 pcs. Bawang may be fried, roasted, soaked in vinegar or boiled and taken three times a day. 45. Used to relieve muscle and joint pain: a. Lagundi b. Akapulko c. Yerba Buena d. Sambong

Ans. C Yerba Buena is indicative for the following ailments: Headache, stomachache, cough and colds, arthritis causing joint pains, swollen gums, toothache, etc. 46. In the principle of from cleanest to the dirtiest care. Which of the following is arranged correctly? a. measles case, pregnant woman, newborn b. post-partum, newborn, measles case c. tuberculosis, hypertension, measles d. post-partum, measles case, newborn Ans. B. Post-partum, newborn, measles care 47. To disseminate information, the BEST type of nurse-family contact is: a. home visit b. clinic visit c. community assembly d. except b&c Ans. C. community assembly 48. The type of public health nursing that focuses on the promotion of health and wellness of pupils, teaching and non teaching personnel of the school. a. Occupational Health Nursing b. School Nursing c. Mental Health Nursing d. All of the above Ans. B. School Nursing 49. This type of vaccines are the most sensitive to heat. They are in the form of live attenuated and freeze dried: a. DPT, Hep B b. BCG, Tetanus Toxoid c. Measles, Oral Polio d. BCG, Measles Ans. C. Oral Polio Vaccine and Measles were the most sensitive to heat with storage temperature of -15 to -25 degree C at the freezer. 50. In DPT the Diptheria and Tetanus are in the form of weakened toxin, what is the form of Pertussis? a. Freeze dried bacteria b. Live attenuated c. Weakened toxoid d. Killed bacteria Ans. D. Killed bacteria. Oral Polio is live attenuated. Measles and BCG are freeze dried bacteria. Diptheria and Tetanus are weakened toxin. 51. It helps the nurse to explain the probable cause of health conditions that occur in the community: a. Vital Statistics b. Demography c. Epidemiology d. Multiple causation theory Ans. C. Epidemiology. Vital Statistics are indices of health and illness status of the community; Demography describes the characteristics of the population in terms of size, composition and distribution in space. Multiple causation theory explains disease as caused not by single condition but by several conditions. 52. It is the intrinsic property of microorganism to survive and multiply in the environment to produce disease. a. Agent c. Causative Agent b. Host d. Environment Ans. A. Agent A. is the correct answer; B. Host influences exposure, susceptibility or response to agents; C. Causative agent is the infectious agent or its toxic component that is transmitted from the source of infection; D. Environment is the sum total of all external condition

53. The following are the components of the environment, EXCEPT: a. Physical c. Spiritual b. Biological d. Socio-economic Ans. C. Spiritual A. Physical environment is composed of the inanimate surroundings such as the geophysical conditions of the climate; B. Biological environment makes up the living things around us such as plants and animal life; C. There is no such thing as spiritual environment; D. Socio-economic environment which may be in the form of level of economic development of the community, presence of social disruptions and the like. 54. Marco was bitten by a dog. You interviewed Marco and his father to take the history before seeing a doctor. You told Marcos father not to kill the dog because the dog will be: a. Given a vaccine b. Confined c. Observed for 10 days d. Be examined Ans. C. When a person is bitten by a dog, the animal should be observed for 10 to 14 days. 1. The person bitten is immunized with rabies immunoglobulin and tetanus toxoid right after the incident to provide immediate protection; 2. If the dog dies before the the observation period is completed, the bitten person should be immunized with rabies vaccine; and 3. If the dog shows signs of rabies, the person should be immunized with rabies vaccineand the dog should be killed immediately and its brain be examined for the presence of Negri bodies, the diagnostic sign of the disease. 55. In patient with dengue fever, which of the following will you give to the patient as part of your intervention? a. Water and salt solution b. Oresol c. Saline solution d. IV fluids Ans. B. H-Fever is characterized by internal bleeding that could result in hypovolemia so rapid fluid replacement is the most important part of the treatment. ORESOL at 75ml/kg in 4-6 hours. Up to 2-3 liters can be given in adult clients. 56. What is the test that confirms the diagnosis of Dengue Fever? a. Torniquet Test b. Capillary Fragility Test c. Platelet Count d. Rumpel Lead Test Ans. C. 57. In doing a tourniquet test, how would petechiae be considered positive? a. 10 b, 20 c. 30 d. 5 Ans. B. The tourniquet test for H-Fever is also known as Rumpel Leads Test. The procedure in taking the test is: 1. Take BP 2. Inflate the BP cuff midway between the systolic and diastolic pressure and leave for 5 minutes 3. Release cuff 4. Count then umber of petechiae below the cuff at the atecubital fossa that would fit inside a 2.5cm square of 1 inch square. The test is positive if thepatient develops 20 or more petechiae with in the area

58. The communicable disease that causes infertility in both male and female? a. Gonorrhea c. Herpes b. Syphilis d. Chlamydia Ans. A

59. Which of the following is the most common and highly contagious STD? a. Gonorrhea c. Herpes b. Syphilis d. Chlamydia Ans. D 60. Which STD causes oral thrush? a. Trichomoniasis c. Chlamydia b. Candidiasis d. Gonorrhea Ans. B

Terms 1. A health care provider who has not been immunized for hepatitis B is exposed to the hepatitis B virus (HBV) through a needle stick from an infected patient. The infection control nurse informs the individual that treatment for the exposure should include a. baseline hepatitis B antibody testing now and in 2 months. b. active immunization with hepatitis B vaccine. c. hepatitis B immune globulin (HBIG) injection. d. both the hepatitis B vaccine and HBIG injection. 2. A patient contracts hepatitis from contaminated food. During the acute (icteric) phase of the patient's illness, the nurse would expect serologic testing to reveal a. hepatitis B surface antigen (HBsAg). b. anti-hepatitis B core immunoglobulin M (anti-HBc IgM). c. anti-hepatitis A virus immunoglobulin G (anti-HAV IgG). d. anti-hepatitis A virus immunoglobulin M (anti-HAV IgM). 3. During evaluation of a patient at an outpatient clinic, the nurse determines that administration of hepatitis B vaccine has been effective when a specimen of the patient's blood reveals a. HBsAg. b. anti-HBs. c. anti-HBc IgM. d. anti-HBc IgG 4. A patient in the outpatient clinic has positive serologic testing for anti-HCV. Which action by the nurse is appropriate? a. Schedule the patient for HCV genotype testing. b. Teach the patient that the HCV will resolve in 2 to 4 months.

Definitions

D Rationale: The recommended treatment for exposure to hepatitis B in unvaccinated individuals is to receive both HBIG and the hepatitis B vaccine, which would provide temporary passive immunity and promote active immunity. Antibody testing may also be done, but this would not provide protection from the exposure.

D Rationale: Hepatitis A is transmitted through the oral-fecal route, and antibody to HAV IgM appears during the acute phase of hepatitis A. The patient would not have antigen or antibodies for hepatitis B. Anti-HAV IgG would indicate past infection and lifelong immunity.

B Rationale: The presence of surface antibody to HBV (anti-HBs) is a marker of a positive response to the vaccine. The other laboratory values indicate current infection with HBV.

A Rationale: Genotyping of HCV has an important role in managing treatment and is done before drug therapy with -interferon or other medications is started. HCV has a high percentage of conversion to the chronic state so the nurse should not teach the patient that the HCV will resolve in 2 to 4

c. Administer immune globulin and the HCV months. Immune globulin or vaccine is not vaccine. available for HCV. d. Instruct the patient on self-administration of -interferon. 5. A homeless patient with severe anorexia, fatigue, jaundice, and hepatomegaly is diagnosed with viral hepatitis and has just been admitted to the hospital. In planning care for the patient, the nurse assigns the highest priority to the patient outcome of a. maintaining adequate nutrition. b. establishing a stable home environment. c. increasing activity level. d. identifying the source of exposure to hepatitis.

A Rationale: The highest priority outcome is to maintain nutrition because adequate nutrition is needed for hepatocyte regeneration. Finding a home for the patient and identifying the source of the infection would be appropriate activities, but they do not have as high a priority as having adequate nutrition. Although the patient's activity level will be gradually increased, rest is indicated during the acute phase of hepatitis.

6. A patient with acute hepatitis B asks the nurse if treatment is available for the condition. The nurse explains to the patient that A a. because no medication is available to treat Rationale: There are no drug therapies to treat acute viral hepatitis, adequate nutrition and acute hepatitis, although -interferon and rest are the most important treatments. nucleoside analogs (i.e., lamivudine) may be used b. lamivudine (Epivir) can decrease viral load to treat chronic hepatitis B. Immune globulin may and liver damage in patients with acute be given within 24 hours after exposure to prevent hepatitis B, but it must be taken for at least 1 hepatitis B, but it is not used to decrease symptoms year. for patients with acute hepatitis. c. patients with acute hepatitis B can be given HBIG to help reduce the symptoms. d. various antiviral drugs are available to treat acute hepatitis B, but serious side effects limit their use. 7. Combination therapy of -interferon and ribavirin (Rebetol) is being used to treat hepatitis C in a patient with human immunodeficiency virus (HIV). The nurse will plan to monitor a. blood glucose. b. lymphocyte count.

B Rationale: Therapy with ribavirin and -interferon may decrease lymphocyte counts. The other laboratory values should not be changed by the drug therapy.

c. potassium level. d. serum creatinine. 8. When taking a health history for a new patient, which information given by the B patient would indicate that screening for Rationale: Any patient with a history of IV drug hepatitis C is appropriate? use should be tested for hepatitis C. Blood a. The patient had a blood transfusion after transfusions given after 1992, when an antibody surgery in 1998. test for hepatitis C became available, do not pose a b. The patient reports a one-time use of IV risk for hepatitis C. Hepatitis C is not spread by the drugs 20 years ago. oral-fecal route and therefore is not caused by c. The patient eats frequent meals in fast-food contaminated food or by traveling in restaurants. underdeveloped countries. d. The patient recently traveled to an undeveloped country.

9. A patient is admitted with an abrupt onset of jaundice, nausea and vomiting, hepatomegaly, and abnormal liver function B studies. Serologic testing is negative for viral Rationale: The patient's symptoms, lack of causes of hepatitis. Which question by the antibodies for hepatitis, and the abrupt onset of nurse is most appropriate? symptoms suggest toxic hepatitis, which can be a. "Have you been around anyone with caused by commonly used OTC drugs such as jaundice?" acetaminophen (Tylenol). Exposure to a jaundiced b. "Do you use any prescription or over-the- individual and a history of IV drug use are risk counter (OTC) drugs?" factors for viral hepatitis. Corticosteroid use does c. "Are you taking corticosteroids for any not cause the symptoms listed. reason?" d. "Is there any history of IV drug use?"

C 10. When teaching a patient recovering from hepatitis B about management of the illness, Rationale: After the acute (icteric) phase, there is a the nurse determines that additional teaching convalescent phase lasting several months. The is needed when the patient says other patient statements are correct and indicate a. "I should not drink alcohol for at least the that teaching has been effective. next year." b. "My family members should be tested for hepatitis B." c. "When the jaundice is gone, I have recovered from my illness and the infection is cured." d. "Until my tests for the virus are negative, I should use a condom for sexual intercourse."

11. A patient with cirrhosis has 4+ pitting edema of the feet and legs and massive ascites. The data indicate that it is most important for the nurse to monitor the patient's a. temperature. b. albumin level. c. hemoglobin. d. activity level.

B Rationale: The low oncotic pressure caused by hypoalbuminemia is a major pathophysiologic factor in the development of ascites and edema. The other parameters should also be monitored, but they are not contributing factors to the patient's current symptoms.

B 12. A 32-year-old patient has early alcoholic Rationale: The disease progression can be stopped cirrhosis diagnosed by a liver biopsy. When or reversed by alcohol abstinence. The other planning patient teaching, the priority interventions may be used when cirrhosis becomes information for the nurse to include is the more severe to decrease symptoms or need for complications, but the priority for this patient is to a. vitamin B supplements. stop the progression of the disease. b. abstinence from alcohol. c. maintenance of a nutritious diet. d. long-term, low-dose corticosteroids. C Rationale: Spironolactone is a potassium-sparing 13. A patient with cirrhosis who is being diuretic and will help to increase the patient's treated with spironolactone (Aldactone) and potassium level. The nurse does not need to talk furosemide (Lasix) has a serum sodium level with the doctor before giving the spironolactone, of 135 mEq/L (135 mmol/L) and serum although the health care provider should be potassium 3.2 mEq/L (3.2 mmol/L). Before notified about the low potassium value. The notifying the health care provider, the nurse furosemide will further decrease the patient's should potassium level and should be held until the nurse a. administer the furosemide and withhold the talks with the health care provider. spironolactone. b. give both drugs as scheduled. c. administer the spironolactone. d. withhold both drugs until talking with the health care provider. C 14. When assessing the neurologic status of a Rationale: Extending the arms allows the nurse to patient with a diagnosis of hepatic check for asterixis, a classic sign of hepatic encephalopathy, the nurse asks the patient to encephalopathy. The other tests might also be done a. stand on one foot. as part of the neurologic assessment but would not b. ambulate with the eyes closed. be diagnostic for hepatic encephalopathy. c. extend both arms. d. perform the Valsalva maneuver.

15. When lactulose (Cephulac) 30 ml QID is D ordered for a patient with advanced cirrhosis, Rationale: The purpose for lactulose in the patient the patient complains that it causes diarrhea. with cirrhosis is to lower ammonia levels and The nurse explains to the patient that it is still prevent encephalopathy. Although the medication important to take the drug because the may promote fluid loss through the stool, prevent lactulose will constipation, and prevent bearing down during a. promote fluid loss. bowel movements (which could lead to esophageal b. prevent constipation. bleeding), the medication is not ordered for these c. prevent gastrointestinal (GI) bleeding. purposes for this patient. d. improve nervous system function. 16. A patient who is admitted with acute hepatic encephalopathy and ascites receives B instructions about appropriate diet. The nurse Rationale: The patient with acute hepatic determines that the teaching has been encephalopathy is placed on a low-protein diet to effective when the patient's choice of foods decrease ammonia levels. The other choices are all from the menu includes higher in protein and would not be as appropriate a. an omelet with cheese and mushrooms and for this patient. In addition, the patient's ascites milk. indicate that a low-sodium diet is needed and the b. pancakes with butter and honey and orange other choices are all high in sodium. juice. c. baked beans with ham, cornbread, potatoes, and coffee. d. baked chicken with french-fries, low-fiber bread, and tea.

B Rationale: Maintaining gas exchange has the 17. A patient with cirrhosis has a massive highest priority because oxygenation is essential hemorrhage from esophageal varices. In for life. The airway is compromised by the planning care for the patient, the nurse gives bleeding in the esophagus and aspiration easily the highest priority to the goal of occurs. The other goals would also be important a. controlling bleeding. for this patient, but they are not as high a priority b. maintenance of the airway. as airway maintenance. c. maintenance of fluid volume. d. relieving the patient's anxiety. Cognitive Level: Application Text Reference: pp. 1107, 1114 Nursing Process: Planning NCLEX: Physiological Integrity 18. During treatment of a patient with a C Minnesota balloon tamponade for bleeding Rationale: The most common complication of esophageal varices, which nursing action will balloon tamponade is aspiration pneumonia. In be included in the plan of care? addition, if the gastric balloon ruptures, the a. Encourage the patient to cough and deep esophageal balloon may slip upward and occlude breathe. the airway. Coughing increases the pressure on the

b. Insert the tube and verify its position q4hr. varices and increases the risk for bleeding. The c. Monitor the patient for shortness of breath. health care provider inserts the tube and verifies d. Deflate the gastric balloon q8-12hr. the position. The esophageal balloon is deflated every 8 to 12 hours to avoid necrosis, but if the gastric balloon is deflated, the esophageal balloon may occlude the airway.

19. A patient with severe cirrhosis has an episode of bleeding esophageal varices. To detect possible complications of the bleeding episode, it is most important for the nurse to monitor a. prothrombin time. b. bilirubin levels. c. ammonia levels. d. potassium levels. 20. The nurse identifies a nursing diagnosis of risk for impaired skin integrity for a patient with cirrhosis who has ascites and 4+ pitting edema of the feet and legs. An appropriate nursing intervention for this problem is to a. restrict dietary protein intake. b. arrange for a pressure-relieving mattress. c. perform passive range of motion QID. d. turn the patient every 4 hours.

C Rationale: The blood in the GI tract will be absorbed as protein and may result in an increase in ammonia level since the liver cannot metabolize protein well. The prothrombin time, bilirubin, and potassium levels should also be monitored, but these will not be affected by the bleeding episode.

B Rationale: The pressure-relieving mattress will decrease the risk for skin breakdown for this patient. Dietary protein intake may be increased in patients with ascites to improve oncotic pressure. Turning the patient every 4 hours will not be adequate to maintain skin integrity. Passive range of motion will not take pressure off areas like the sacrum that are vulnerable to breakdown.

21. A portocaval shunt is considered for a patient with cirrhosis following an episode of bleeding esophageal varices. The nurse plans B to teach the patient that this procedure Rationale: The risk for hepatic encephalopathy a. is likely to improve the patient's life increases after shunt procedures because blood expectancy. bypasses the portal system and ammonia is b. will increase the risk of hepatic diverted past the liver and into the systemic encephalopathy. circulation. Life expectancy is not improved. The c. will help to decrease the incidence of risk for peritonitis is not decreased by a surgical peritonitis. procedure, which will increase infection risk. Firstd. is a first-line therapy for portal line procedures for portal hypertension are hypertension. medications such as diuretics and albumin.

22. A patient with cancer of the liver has severe ascites, and the health care provider plans a paracentesis to relieve the fluid pressure on the diaphragm. To prepare the patient for the procedure, the nurse a. asks the patient to empty the bladder. b. positions the patient on the right side. c. obtains informed consent for the procedure. d. assists the patient to lie flat in bed.

A Rationale: The patient should empty the bladder to decrease the risk of bladder perforation during the procedure. The patient would be positioned in Fowler's position and would not be able to lie flat without compromising breathing. The health care provider is responsible for obtaining informed consent.

23. A patient with end-stage liver disease who is to undergo a liver transplant tells the nurse, "I have a friend who has already rejected two kidney transplants. I am concerned that I will reject this liver." The nurse's best response to the patient is a. "Perhaps your friend did not have a good C tissue match with the kidney transplants." Rationale: The liver is less susceptible to rejection b. "You would not be scheduled for a than the kidney. The other statements are transplant if there was a concern about inaccurate or will not decrease the patient's rejection." anxiety. c. "The problem of rejection is not as common in liver transplants as in kidney transplants." d. "It is easier to get a good tissue match with liver transplants than with kidney transplants."

24. A patient hospitalized with possible acute pancreatitis has severe abdominal pain and nausea and vomiting. The nurse would expect C the diagnosis to be confirmed with laboratory Rationale: Amylase is elevated early in acute testing that reveals elevated serum pancreatitis. Changes in bilirubin, calcium, and a. calcium. potassium levels are not diagnostic for pancreatitis. b. bilirubin. c. amylase. d. potassium.

25. In planning care for a patient with acute pancreatitis, the nurse assigns the highest priority to the patient outcome of a. developing no acute complications. b. maintenance of normal respiratory function. c. expressing satisfaction with pain control. d. having adequate fluid and electrolyte balance.

B Rationale: Respiratory failure can occur as a complication of acute pancreatitis, and maintenance of adequate respiratory function is the priority goal. The other outcomes would also be appropriate for the patient.

C Rationale: Pancreatic enzymes are released when the patient eats. NG suction and NPO status 26. A patient with acute pancreatitis has a decrease the release of these enzymes. Fluid and nasogastric (NG) tube to suction and is NPO. electrolyte imbalances will be caused by NG The nurse explains to the patient that the suction and require that the patient receive IV major purpose of this treatment is fluids to prevent this. The patient's nausea and a. control of fluid and electrolyte imbalance. vomiting may decrease, but this is not the major b. relief from nausea and vomiting. reason for these treatments. The pancreatic c. reduction of pancreatic enzymes. enzymes that precipitate the pancreatitis are not d. removal of the precipitating irritants. removed by NG suction.

27. The nurse identifies the collaborative problem of potential complication: electrolyte imbalance for a patient with A severe acute pancreatitis. Assessment Rationale: Muscle twitching and finger numbness findings that alert the nurse to electrolyte indicate hypocalcemia, a potential complication of imbalances associated with acute pancreatitis acute pancreatitis. The other data indicate other include complications of acute pancreatitis but are not a. muscle twitching and finger numbness. indicators of electrolyte imbalance. b. paralytic ileus and abdominal distention. c. hypotension. d. hyperglycemia.

28. When obtaining a health history from a patient with acute pancreatitis, the nurse asks the patient specifically about a history of a. cigarette smoking. b. alcohol use. c. diabetes mellitus. d. high-protein diet.

B Rationale: Alcohol use is one of the most common risk factors for pancreatitis in the United States. Cigarette smoking, diabetes, and high-protein diets are not risk factors.

29. The health care provider prescribes pancreatin (Viokase) for a patient with chronic pancreatitis. The nurse teaches the A patient that the drug is considered effective if Rationale: The patient's steatorrhea should improve the patient experiences if the pancreatic enzymes are effective. The a. normal-appearing stools. pancreatin will not decrease jaundice, improve b. decreased jaundice. appetite, or reduce abdominal pain. c. improved appetite. d. reduced abdominal pain.

30. When the nurse is caring for the patient with pancreatic cancer, which nursing diagnosis is a priority? a. Chronic pain related to tumor pressure on abdominal structures b. Imbalanced nutrition: less than required related to anorexia c. Impaired skin integrity related to itching secondary to jaundice d. Grieving related to potentially terminal diagnosis 31. A patient who is admitted to the hospital with a sudden onset of severe right upperquadrant pain that radiates to the right shoulder is diagnosed with cholecystitis. Which assessment information will be most important for the nurse to report to the health care provider? a. The patient has an increase in pain after eating. b. The patient needs 4 mg of morphine for pain relief. c. The patient's stools are clay colored. d. The patient's urine is bright yellow. 32. When caring for a patient following an incisional cholecystectomy for cholelithiasis, the nurse places the highest priority on assisting the patient to a. turn, cough, and deep breathe every 2 hours. b. choose low-fat foods from the menu. c. perform leg exercises hourly while awake. d. ambulate the evening of the operative day.

A Rationale: All of these nursing diagnoses are appropriate for a patient with pancreatic cancer, but treating the patient's pain is the priority because the patient will be unable to meet outcomes for the other nursing diagnoses unless the pain is controlled.

C Rationale: The clay-colored stools indicate biliary obstruction, which requires rapid intervention to resolve. The other data are not unusual for a patient with this diagnosis, although the nurse would also report the other assessment information to the health care provider.

A Rationale: Postoperative nursing care after a cholecystectomy focuses on prevention of respiratory complications because the surgical incision is high in the abdomen and impairs coughing and deep breathing. The other nursing actions are also important to implement but are not as high a priority as ensuring adequate ventilation.

D 33. An appropriate collaborative problem for Rationale: With obstruction of the common bile the nurse to include in the care plan for a duct, bile will back up into the liver and damage patient with cholelithiasis and obstruction of liver cells. Bleeding, gastritis, and the common bile duct is thromboembolism are not common complications a. potential complication: bleeding. of biliary obstruction. b. potential complication: gastritis. c. potential complication: thromboembolism. d. potential complication: biliary cirrhosis. 34. When providing discharge instructions to C a patient following a laparoscopic Rationale: After a laparoscopic cholecystectomy, cholecystectomy at an outpatient surgical the patient will have Band-Aids in place over the center, the nurse recognizes that teaching has incisions; patients are discharged the same (or been effective when the patient states, next) day and have few restrictions on activities of a. "I should plan to limit my activities and not daily living. Drainage from the incisions would be return to work for 4 to 6 weeks." abnormal, and the patient should be instructed to b. "I can expect some reddish yellow call the health care provider if this occurs. A lowdrainage from the incisions for a few days." fat diet may be recommended for a few weeks after c. "I can remove the bandages on my surgery but will not be a life-long requirement. incisions tomorrow and take a shower." d. "I will always need to maintain a low-fat diet since I no longer have a gallbladder." D 35. Which data obtained by the nurse during Rationale: The asterixis indicates that the patient the assessment of a patient with cirrhosis will has hepatic encephalopathy, and hepatic coma may be of most concern? occur. The spider angiomas and right uppera. The patient's skin has multiple spiderquadrant abdominal pain are not unusual for the shaped blood vessels on the abdomen. patient with cirrhosis and do not require a change b. The patient has ascites and a 2-kg weight in treatment. The ascites and weight gain do gain from the previous day. indicate the need for treatment but not as urgently c. The patient complains of right upperas the changes in neurologic status. quadrant pain with abdominal palpation. d. The patient's hands flap back and forth when the arms are extended. C 36. A patient with severe cirrhosis has a new Rationale: -blockers have been shown to decrease prescription for propranolol (Inderal). The the risk for bleeding in esophageal varices. nurse will teach the patient that the Although propranolol will decrease BP and prevent medication is ordered to cardiac ischemia, these are not the purposes for this a. decrease systemic BP. patient. Propranolol will not decrease fluid b. prevent the development of ischemia. retention or edema. c. lower the risk for bleeding varices. d. reduce fluid retention and edema.

37. A patient who was admitted with acute bleeding from esophageal varices asks the nurse the purpose for the ordered ranitidine (Zantac). Which response by the nurse is most appropriate? a. The medication will inhibit the development of gastric ulcers. b. The medication will prevent irritation to the esophageal varices. c. The medication will decrease nausea and anorexia. d. The medication will reduce the risk for aspiration.

B Rationale: The therapeutic action of H2 receptor blockers in patients with esophageal varices is to prevent irritation and bleeding from the varices caused by reflux of acid gastric contents. Although ranitidine does decrease the risk for peptic ulcers, reduce nausea, and help prevent aspiration pneumonia, these are not the primary purpose for H2 receptor blockade in this patient.

D Rationale: Providing oral hygiene is included in the 38. Which of these nursing actions included education and scope of practice of nursing in the plan of care for a patient with cirrhosis assistants. Assessments and assisting patients to can the nurse delegate to a nursing assistant? choose therapeutic diets are nursing actions that a. Assessing the patient for jaundice require higher-level nursing education and scope of b. Assisting the patient in choosing the diet practice and would be delegated to LPNs/LVNs or c. Palpating the abdomen for distention RNs. d. Providing oral hygiene before meals

39. When taking the BP of a patient with severe acute pancreatitis, the nurse notices carpal spasm of the patient's hand. Which action should the nurse take next? a. Notify the health care provider immediately. b. Retake the patient's blood pressure. c. Check the calcium level on the chart. d. Ask the patient about any arm pain.

C Rationale: The patient with acute pancreatitis is at risk for hypocalcemia, and the assessment data indicate a positive Trousseau's sign. The health care provider should be notified after the nurse learns the patient's calcium level. There is no indication that the patient needs to have the BP rechecked or that there is any arm pain.

B 40. A patient with acute pancreatitis has a Rationale: NG suction and NPO status will nasogastric (NG) tube to suction and is NPO. decrease the release of pancreatic enzymes into the Which information obtained by the nurse is pancreas and decrease pain. Although bowel the best indicator that these therapies have sounds may be hypotonic with acute pancreatitis, been effective? the presence of bowel sounds does not indicate that a. Bowel sounds are present. treatment with NG suction and NPO status have b. Abdominal pain is decreased. been effective. Electrolyte levels will be abnormal c. Electrolyte levels are normal. with NG suction and must be replaced by d. Grey Turner sign resolves. appropriate IV infusion. Although Grey Turner sign will eventually resolve, it would not be

appropriate to wait for this occur to determine whether treatment was effective.

D Rationale: A palpable abdominal mass may 41. When the nurse is caring for a patient indicate the presence of a pancreatic abscess, with acute pancreatitis, which of these which will require rapid surgical drainage to assessment data should be of most concern? prevent sepsis. Absent bowel sounds, abdominal a. Absent bowel sounds tenderness, and left upper quadrant pain are b. Abdominal tenderness common in acute pancreatitis and do not require c. Left upper quadrant pain rapid action to prevent further complications. d. Palpable abdominal mass

You might also like